Sunteți pe pagina 1din 308

MATEMTICA PASSO A PASSO

OPERAES FUNDAMENTAIS Sistema de Numerao Decimal

O Sistema de Numerao Decimal se baseia na posio que um algarismo tem no numeral.

As regras que definem ordens, classes e nomes que resumimos no seguinte quadro:

A Numerao Decimal
314 537 012 423

bilhes dezena de bilhes centena de bilhes

milhes dezena de milhes unidade

milhares dezena de milhares

unidade dezena de

centena de milhes

centena de milhares

centena de unidade

Classe dos bilhes Classe dos Milhes Classe dos Milhares

Classe das unidades

314
unid. de 10 orde Unid. de 11 ordem unid. de 12 ordem

537
unid. de 7 ordem unid. de 8 ordem unid. de 9 ordem

012
unid. de 4 ordem unid. de 5 ordem unid. de 6 ordem

423
unid. de 1 ordem unid. de 2 ordem unid. de 3 ordem

cada algarismo situado esquerda de outro tem um valor dez vezes maior que se estivessem no lugar desse outro.

MATEMTICA PASSO A PASSO

Tabuada da Adio
4+ 1= 5 4 + 2 = ___ 4+ 3= 7 4+ 4= 8 4 + 5 = ___ 4 + 6 = ___ 4 + 7 = 11 4 + 8 = ___ 4 + 9 = 13 4 +10= ___ 9 + 1 = ___ 9 + 2 = 11 9 + 3 = 12 9 + 4 = 13 9 + 5 = ___ 9 + 6 = ___ 9 + 7 = 16 9 + 8 = 17 9 + 9 = ___ 9 +10= 19 5 + 1 = ___ 5 + 2 = ___ 5+ 3= 8 5 + 4 = ___ 5 + 5 = 10 5 + 6 = ___ 5 + 7 = ___ 5 + 8 = 13 5 + 9 = 14 5 +10= ___ 10 + 1=___ 10 + 2= 12 10 + 3= 13 10 + 4=___ 10 + 5=___ 10 + 6= 16 10 + 7= 17 10 + 8=___ 10 + 9= 19 10 +10=___ 6 + 1 = ___ 6+ 2= 8 6+ 3= 9 6 + 4 = ___ 6 + 5 = 11 6 + 6 = 12 6 + 7 = ___ 6 + 8 = 14 6 + 9 = ___ 6 +10= 16 11 + 1= 12 11 + 2=___ 11 + 3=___ 11 + 4= 15 11 + 5= 16 11 + 6=___ 11 + 7=___ 11 + 8= 19 11 + 9= 20 11 +10=___ 7+ 1= 8 7 + 2 = ___ 7 + 3 = 10 7 + 4 = ___ 7 + 5 = ___ 7 + 6 = 13 7 + 7 = ___ 7 + 8 = 15 7 + 9 = 16 7 +10= ___ 12 + 1=___ 12 + 2= 14 12 + 3= 15 12 + 4=___ 12 + 5= 17 12 + 6= 18 12 + 7=___ 12 + 8= 20 12 + 9= 21 12 +10=___

3 + 1 = ___ 3+ 2= 5 3 + 3 = ___ 3 + 4 = ___ 3+ 5= 8 3 + 6 = ___ 3 + 7 = ___ 3 + 8 = 11 3 + 9 = ___ 3 +10= ___ 8 + 1 = ___ 8 + 2 = ___ 8 + 3 = 11 8 + 4 = 12 8 + 5 = ___ 8 + 6 = 14 8 + 7 = 15 8 + 8 = ___ 8 + 9 = ___ 8 +10= ___

Adio - Passo a Passo

Adio a operao onde juntamos quantidades Em adies usa-se o sinal de + (mais). Parcelas so os termos da adio. O resultado da adio chama-se soma ou total Ao efetuarmos uma adio, colocamos. unidade embaixo de unidade dezena embaixo de dezena centena embaixo de centena A soma sempre se inicia Pela direita. +
C D 2 4 1 3 3 7 U 2 5 7

parcela parcela soma ou total

PROF. WELLINGTON BRITO

Adio com reserva


C D U +1 +1 2 1 6 6 + 6=12 5 8 9 1 6 2

Soma-se as unidades: 6unidades + 6unidades = 12 unidades, que corresponde a 1 dezena e 2 unidades.Escreve-se o 2 na ordem das unidades e o 1 vai para a ordem das dezenas. O mesmo acontece com as centenas. Soma-se as dezenas:1 dezena + 1 dezena + 9 dezenas = 11 dezenas, que corresponde a :1 centena e 1 unidade.Escreve se o primeiro 1 na ordem das dezenas e segundo 1 vai para a ordem das centenas.

Prova Real da Adio Para sabermos se uma conta est correta usamos a operao inversa. A operao inversa da ADIO SUBTRAO. Prova Real Parcela 2 4 2 3 7 7 3 7 7 + ou parcela 1 3 5 1 3 5 2 4 2 soma 3 7 7 2 4 2 1 3 5 ou total A soma ou total menos uma das parcelas sempre igual a outra parcela. Tabuada da Subtrao
22= 32=1 4 2 = ___ 5 2 = ___ 62=4 72=5 8 2 = ___ 9 2 = ___ 10 2 = 8 11 2 = 9 7 7=___ 8 7= 1 9 7= 2 10 7= 3 11 7=___ 3 3= 0 4 3=___ 5 3= 2 6 3= 3 7 3=___ 8 3= 5 9 3= 6 10 3=___ 11 3=___ 12 3=___ 8 8= 0 9 8=___ 10 8=___ 11 8= 3 12 8= 4 4 4=___ 5 4=___ 6 4= 2 7 4= 3 8 4=___ 9 4= 5 10 4= 6 11 4=___ 12 4= 8 13 4=___ 9 9=___ 10 9= 1 11 9=___ 12 9=___ 13 9=___ 5 5=0 6 5=___ 7 5=___ 8 5= 3 9 5= 4 10 5= 5 11 5=___ 12 5=___ 13 5= 8 14 5=___ 10 1=___ 10 2=___ 10 3= 7 10 4= 6 10 5= 5 1 1 =___ 21=1 31=2 4 1 = ___ 51=4 61=5 7 1 =___ 81=7 91=8 10 1 = ___ 6 6=___ 7 6=___ 8 6= 2 9 6= 3 10 6= ___

10

11 6= ___ 12 6= 6 13 6= 7 14 6= ___ 15 6= ___

MATEMTICA PASSO A PASSO 12 7=___ 13 8=___ 14 9= 5 13 7= 6 14 8=___ 15 9= ___ 14 7=___ 15 8= 7 16 9= 7 15 7=___ 16 8= 8 17 9=___ 16 7= 9 17 8=___ 18 9= 9

10 6= 4 10 7=___ 10 8=___ 10 9= 1 10 10=___

Subtrao - Passo a Passo

Subtrao a operao onde retiramos uma quantidade menor de uma maior. O subtraendo no pode ser maior que o minuendo. Em subtraes usamos o sinal (menos). O minuendo e o Subtraendo so termos da subtrao. O resto ou diferena o resultado da subtrao. A subtrao sempre se inicia pela direita Na subtrao, colocamos: C D U unidade embaixo de unidade; 7 4 1 minuendo dezena embaixo de dezena; 3 2 1 subtraendo centena embaixo de centena. 4 2 0 resto ou diferena Subtrao com Recurso C D U No se pode tirar 8 unidade de 3 unidades, 3 1 pois o 8 maior que 3. Ento pedimos 1 Minuendo 4 3 dezena emprestada a ordem das dezenas subtraendo 1 8 e juntamos s unidades. resto ou 2 5 D U 3 diferena 4 3 +1 10 +3 = 13 Agora de 13, podemos tirar 8. 13 unidades - 8 unidades 5 unidades Em 4 dezenas, emprestamos 1 dezena. Ficaram 3 dezenas. 4 dezenas - 1 dezena 3 dezenas o 4 ficou valendo 3. Prova Real da Subtrao.

A operao inversa SUBTRAO a ADIO. Prova Real Minuendo 7 8 6 2 + 11

PROF. WELLINGTON BRITO

subtraendo resto ou diferena

1 6

6 2

1 7

6 8

Tabuada da Multiplicao
3 x 1=___ 3 x 2=___ 3 x 3=___ 3 x 4= 12 3 x 5=___ 3 x 6= 18 3 x 7=___ 3 x 8=___ 3 x 9= 27 3 x10=___ 8 x 1= 8 8 x 2=___ 8 x 3=___ 8 x 4=32 8 x 5=___ 8 x 6=___ 8 x 7=56 8 x 8=___ 8 x 9=___ 8 x 10=___ 4 4 4 4 4 4 4 4 4 4 9 9 9 9 9 9 9 9 9 9 x 1= 4 x 2= 8 x 3=___ x 4=___ x 5=___ x 6=24 x 7=28 x 8=___ x 9=___ x 10=40 x 1=___ x 2=18 x 3=___ x 4=36 x 5=___ x 6=___ x 7=63 x 8=___ x 9=___ x 10=90 5 x 1= 5 5 x 2=___ 5 x 3=___ 5 x 4=20 5 x 5=___ 5 x 6=30 5 x 7=___ 5 x 8=___ 5 x 9=45 5 x 10= __ 10 x 1=10 10 x 2=20 10 x 3=30 10 x 4=__ 10 x 5=50 10 x 6=__ 10 x 7=__ 10 x 8=80 10 x 9=90 10 x 10=__ 6 x 1= 6 6 x 2=___ 6 x 3=___ 6 x 4=24 6 x 5=30 6 x 6=___ 6 x 7=___ 6 x 8=48 6 x 9=___ 6 x10=___ 11 x 1=__ 11 x 2=__ 11 x 3=33 11 x 4=44 11 x 5=55 11 x 6=__ 11 x 7=__ 11 x 8=88 11 x 9=__ 11 x 10=__

2 x 1=___ 2 x 2= 4 2 x 3= 6 2 x 4=___ 2 x 5=___ 2 x 6=12 2 x 7=14 2 x 8=___ 2 x 9=___ 2 x10=___ 7 x 1=___ 7 2=___ 7 x 3=21 7 x 4=28 7 x 5=___ 7 x 6=___ 7 x 7=49 7 x 8=___ 7 x 9=___ 7 x 10=70

Multiplicao - Passo a Passo Multiplicao uma adio de parcelas iguais. Apresentamos a multiplicao com o sinal x (vezes). O multiplicando e o multiplicador so chamados fatores. O resultado chama-se produto. Observe quantas figuras h nos quadrados. So trs quadrados com quatro livros. ento: 4 + 4 + 4 = 12 ou 3 x 4 = 12 4 multiplicando Fatores x3 multiplicador 12 Produto

se multiplicarmos um nmero qualquer por 0 (zero) seu produto ser sempre zero. Veja: 9 x 0 = 0, pois 0 + 0 + 0 + 0 + 0 + 0 + 0 + 0 + 0 = 0

Multiplicao sem Reserva 12

MATEMTICA PASSO A PASSO

C D U 1 4 2 x 2 2 8 4

Multiplica-se: primeiro a ordem das unidades (U) segundo a ordem das dezenas (D) e por fim, a ordem das centenas (C).

Multiplicao com Reserva Multiplica-se as unidades: C D U 3 x 6 = 18 unidades ou 1 dezena e 8 unidades 1 6 Coloca-se o 8 na ordem das unidades e o 1 vai para X 3 a ordem das dezenas. 4 8 Multiplica-se as dezenas: 3 x 1 = 3 mais 1 que foi so 4. Resultado final: 48 Multiplicao com mais de um Algarismo no Multiplicador
1 1

2 x 5 26 32

6 1 3 9 2

9 2 8 8

multiplicando multiplicador 1 produto parcial 2 produto parcial produto final

Primeiro multiplica-se o 2 pelo 9, depois pelo 6 somando-se com o 1 que foi, em seguida multiplica-se o 2 pelo 2 somando-se com o 1 que foi. Achamos, assim, o primeiro produto parcial. Ao multiplica-se o 1 pelo 9 depois pelo 6 e depois pelo 2 encontrase o segundo produto parcial, que dever ser afastado uma casa para a esquerda.
Sempre iremos afastar uma casa para a esquerda para cada produto parcial:

Veja mais um exemplo: 1 4 3 x 1 3 22 81 6 42 9 6 1 43 2 1 89 0 2

2 multiplicando 2 multiplicador 4 produto parcial produto parcial produto parcial 4 produto final

Multiplicao por 10, 100 e 1000 Para multiplicar um nmero por 10 basta acrescentar um zero direita desse nmero. 13

PROF. WELLINGTON BRITO

Exemplos:

9 x 10 = 90 15 x 10= 150 130 x 10= 1.300

Se for multiplicar por 100 so acrescidos dois zeros direita do nmero: Veja: 8 x 100= 800 16 x 100= 1.600 200 x 100=20.000 E por 1000, acrescenta-se trs zeros direita do nmero: Confira: 7 x 1000= 7.000 40 x 1000= 40.000

Prova Real da Multiplicao

A operao inversa multiplicao a diviso. Prova Real 4 multiplicando 8 2 fatores 0 4 x 2 multiplicador 8 produto Divide-se o produto por um dos fatores e encontra o outro fator. Tabuada Da Diviso
2 2=__ 4 2=__ 6 2= 3 8 2= 4 10 2= 5 12 2= 6 14 2=__ 16 2=__ 18 2= 9 20 2=__ 7 7=__ 14 7= 2 21 7= 3 28 7= 4 35 7=__ 3 3=__ 6 3=__ 9 3= 3 12 3=__ 15 3=__ 18 3= 6 21 3=__ 24 3= 8 27 3=__ 30 3=10 8 8=__ 16 8= 2 24 8= 3 32 8= 4 40 8=__ 4 4= 1 8 4= 2 12 4=__ 16 4=__ 20 4= 5 24 4= 6 28 4=__ 32 4=__ 36 4= 9 40 4=10 9 9= __ 18 9= 2 27 9= 3 36 9= 4 45 9=__ 5 5=__ 10 5= 2 15 5= 3 20 5= 4 25 5= 5 30 5=__ 35 5=__ 40 5= 8 45 5= 9 50 5=10 10 10= 1 20 10= 2 30 10=__ 40 10= 4 50 10= 5

1 1=___ 2 1= 2 3 1=___ 4 1= 4 5 1= 5 6 1=___ 7 1=___ 8 1= 8 9 1=__ 10 1=10 6 6= 1 12 6= 2 18 6= 3 24 6=__ 30 6=__

14

36 6= 6 42 6= 7 48 6=__ 54 6=__ 60 6=10

MATEMTICA PASSO A PASSO 42 7= 6 48 8= 6 54 9=__ 49 7=__ 56 8= 7 63 9= 7 56 7= 8 64 8=__ 72 9= 8 63 7= 9 72 8= 9 81 9= 9 70 7=10 80 8=10 90 9=__

60 10=__ 70 10=__ 80 10= 8 90 10=__ 10010=10

Diviso - Passo a Passo Diviso a operao onde separamos uma quantidade em partes iguais. Representamos a diviso pelos sinais: ou :
dividendo

resto

4 2 0 2

divisor quociente

dividendo 4 2

- 4 2
resto

divisor quociente

0 4 dividido por 2 so 2. 2vezes 2 so 4. 4 para chegar no 4 no falta nada, ento O (zero). Diviso Exata Na diviso exata, o resto ser sempre zero. 12 3 veja na prtica 0 4 So 12 biscoitos, cercados de 3 em 3, pois a diviso por 3. Formamos 4 conjuntos e no sobrou nada do lado de fora. Por isso, a diviso exata! Prova Real da Diviso Exata Para tirarmos a prova real da diviso exata, s multiplicarmos o divisor pelo quociente. Prova Real 68 2 3 4 08 3 4 x 2 0 6 8 Diviso Inexata ou Aproximada Em uma diviso inexata o resto ser diferente de zero e sempre menor que o divisor.

7 2 Na prtica 1 3 resto 1 So 7 borboletas, cercadas de 2 em 2, pois a diviso por 2. 15

PROF. WELLINGTON BRITO

Formamos 3 conjuntos, mas 1 borboleta sobrou. Na tabuada de x 2 no existe um nmero que multiplicado por 2 d como resultado 7.Ento, procura-se o maior nmero que multiplicado por 2 d um resultado prximo (porm nunca maior) que 7. Esse nmero 3. Observe : 2 x 1= 2 2x2=4 2x3=6 o mais prximo e menor que 7 2x4=8 maior que 7 2 x 5 =10 Prova Real da Diviso Aproximada

Para tirarmos a prova real da diviso inexata, s multiplicarmos o divisor pelo quociente e somarmos este resultado com o resto

Vejamos: 72 22 2 5 14

Prova Real 14 x5 70 +2 72

resto :

Vamos entender o processo da diviso com 72 5 ?

7 dividido por 5 1, que multiplicado por 5 so 5. 5 para chegar no 7 faltam 2. Abaixa-se o outro 2 do dividendo. Tendo agora 22 para dividir por 5, que dar 4. 4 multiplicado por 5 so 20. 20 para chegar no 22 faltam 2. Como no h mais nmeros para abaixar, fecha-se a conta com resto 2. 16

MATEMTICA PASSO A PASSO

Obs1: Numa diviso o divisor deve ser diferente de zero. Obs2: O maior resto de uma diviso aproximada o divisor menos a unidade.

EXPRESSES ARITMTICAS O uso dos sinais auxiliares ( ) parntese, [ ] colchtes e { } chaves permite calcular em cada caso: a) 9 (5+3) ou 9 8 = 1 b) 15 [ 12 (7 + 2) ] Que indica: 15 [12-9] E ainda: 15 3 = 12 c) 12 { 10 [7 + (5 4) ] } indica: 12 { 10 [ 7 + 1] } Ou: 12 { 10 8} 12 - 2 = 10 EXERCCIO - EXPRESSES ARITMTICAS ( I ) Resolver as seguintes expresses aritmticas:

a) b) c) d) e) f) g) h)

100 80 + 40 (30+5) 45 (120 100) + 120 (100 + 10) (7 + 5) + [ 6 (9 5) + (15 + 1) 58 + [48 (31 10) + 15 ] 38 { (51 15) + [ 5 + (3 1)] 10} { 108 [ 15 + (13 10) ] } + 58 528 { 675 [ 255 (15 + 13 ) ] } { 57 [ 108 (71 + 26) ] } { 177 [ 96 + (51 16 ) ] } 17

PROF. WELLINGTON BRITO

Respostas a) 25 b) 35 c) 30

d) 100 e) f) 5 148

g) 80 h) Zero

Expresses Aritmticas contendo as Quatro Operaes: Nestas expresses obedece-se seguinte ordem de operaes: 1 As multiplicaes e divises; 2 As adies e subtraes A eliminao de parnteses, colchetes, chaves etc., feita a partir dos mais internos que, em geral, so os parnteses, seguidos dos colchetes, chaves etc. Exemplos: 1) [ ( 18 + 3 x 2 ) 8 + 5 x 3] 6 + (625 11 x 5 ) 10 = [ ( 18 + 6) 8 + 15] 6 + (625 55 ) 10 = = [24 8 + 15] 6 + 570 10= [ 3 + 15 ] 6 + 57 = = 18 6 + 57= 3 + 57 = 60 2){ 240 3 x [ 24 ( 2 + 5 ) x (9 6 ) ] 180 9 } x ( 2 + 36 3) = { 240 3 x [ 24 7 x 3 ] 20 } x ( 2 +12) = = { 240 3 x [ 24 21 ] 20 } x 14 = = { 240 3 x 3 20} x 14 = { 240 9 20 } x 14 = = { 231 20 } x 14 = 211 x 14 = 2.954 EXERCCIO EXPRESSES ARITMTICAS ( II ) Resolver as seguintes expresses aritmticas: a) 15 + 8 x ( 40 5 x 6 ) b) 28 5 x 4 (40 8 x 4 ) + 10 c) 105 6 x [ ( 12 5 ) x ( 11 9 ) + ( 3 + 2 ) x ( 4 3 ) ] d) 8 + [ ( 255 21 x 3 ) 6 ] e) f) [ 9 + ( 585 15 x 6 ) ] 56 100 25 + 58 x 3 + 20 x 5 1864 8

g) [ 30 ( 17 8 ) x 3 + 25 ] 7 h) [ ( 18 + 6 ) 3 + 4 x 5 ] ( 4 + 3 ) + ( 125 x 5 55 ) 10 i) j) { 48 [ 57 ( 2 + 15 ) x 3 ] + 12 } 4 720 { 3 x [ 67 ( 10 + 3 x 9 ) ] } ( 79 15 ) 8 18

k) { 11 x [ 2 x ( 42 + 37 ) 41 ] } [ 143 x ( 17 8 ) ] Respostas a) 95 e) 9 i) 5 b) 10 f) 45 j) zero c) - 9 g) 4 k) 1 d) 40 h)61 QUESTES DE CONCURSOS E VESTIBULARES: 1. (CEF) 1 2 X 5Y + Z 3 0 2 1 7 4 W1 Determinando-se esses algarismos para que a soma seja verdadeira, verifica-se que: a) Y W = X c) Y = 8 e) X + Z = W b) X = 2 d) Z = 4 1 a b c 3 a b c 4 Acima est representada uma multiplicao onde os algarismos a, b e c so nmeros desconhecidos. Qual o valor da soma a + b + c? a) 5 b) 8 c) 11 d) 14 e)17 3. (UNIFOR) O esquema abaixo apresenta o algoritmo da subtrao de nmeros inteiros, no qual alguns algarismos foram substitudos pelas letras x, y, z e t. 7 3 x 7 -y 4 9 z 2 t 4 9 Reconstituindo-se essa subtrao, a fim de torn-la verdadeira, obtm-se: a) x = y = 2 e z = 2t d) y = 2t e x = 2z b) x = z = 4 e y = 2t e) t = 2x e z = 2y c) y = z = 8 e x = 4t 4. (MPU) Numa diviso, o divisor 14 o quociente 26 e o resto o maior possvel.O dividendo igual a: a) 379 b) 378 c) 376 d)377 e) 375 5. (TRE) Dividindo-se um nmero natural X por 5, obtm quociente 33 e o resto o maior possvel. Esse nmero X : a) menor que (1) uma centena d) cubo perfeito b) maior que (2) duas centenas e) igual a (3) trs centenas c) quadrado perfeito 19 x 2. ( FUVEST)

MATEMTICA PASSO A PASSO

PROF. WELLINGTON BRITO

6. (UECE) Um certo inteiro n quando dividido por 5 deixa resto 3. O resto da diviso de 4n por 5 igual a: a) 1 b) 2 c) 3 d) 4 Respostas 1)E 2) D 3) E 4) D 5) C 6) B NMERO, NUMERAL E ALGARISMO


1 2

Os nmeros so representados por numerais. O nmero de borboletas representado pelo numeral "5". Para representar os numerais existem smbolos chamados algarismos 0123456789 Eles se chamam Algarismos indo-arbicos. Exemplos: 27 um numeral formado pelos algarismos 2 e 7. Um nmero pode ser representado de vrias formas. O nmero a idia da quantidade de elementos.
5

3+2 cinco 6-1

5x1
5:1

Questes comentadas: Problemas de Contagem 1.


(PRF) Para enumerar as pginas de um livro de 468 pginas, quantos algarismos so escritos? b) 936 c)1296 d) 1324 e) 1428

a) 468

comentrio: a quantidade de algarismos a soma de todos os smbolos usados at a ltima pgina do livro, (468 que um numeral de 3 algarismos) como constante o nmero de algarismos chegamos a seguinte relao: 1 etapa: de 1 at 9 temos 9 nmeros (9x1) = 9 algarismos; 2 etapa: de 10 at 99 temos 90 nmeros (90x2) = 180 algarismos; 3 etapa: de 100 at 999 temos 900 nmeros (900x30) = 2700 algarismos; etc.

20

MATEMTICA PASSO A PASSO

Soluo: conhecendo a relao, precisamos apenas calcular a quantidade de algarismos da 3 etapa somar com +9 +180 (que representam a quantidade de algarismos da 1 e 2 etapas, respectivamente), assim temos:

Pg. 100 at Pg. 468 temos 369 pginas (369 x 3) = 1.107 algarismos 9 + 180 + 1.107 = 1.296 algarismos: Resposta c) 2. (B) Para numerar seguidamente as cadeiras de um auditrio foram necessrios 1092 algarismos. Quantas cadeiras possui o auditrio? b) 400 c) 300 d) 401 e) 1092

a) 301

Comentrio: para encontrarmos o total de nmeros (cadeiras), usaremos a mesma resoluo, porm com as operaes inversas. Soluo: subtraindo do total de algarismos (1092) -9 -180, chegamos em 903 algarismos; agora s dividir por 3 e temos o nmero de cadeiras na 3 etapa. 903 / 3 = 301 cadeiras Agora, no podemos esquecer que o total de cadeiras do auditrio, ser: 9 + 90 + 301 = 400 cadeiras Resposta b)

21

PROF. WELLINGTON BRITO

QUESTES DE CONCURSOS E VESTIBULARES 1.(UECE) O nmero de algarismos, contados com as repeties, necessrios para numerar as 96 pginas de um livro igual a: a)180 b) 181 c)183 d) 185 2.(PUC) Para numerar as pginas de um livro, foram escritos 1359 algarismos. O nmero de pginas desse livro : a) 490 b) 489 c) 488 d) 487 e) 485 3. (BB) Para numerar as pginas de um livro, foram empregados 10681 algarismos. Determine quantas pginas tem o livro. a) 2947 b) 2951 c) 2955 d) 2959 e) 2963 4. (CEF) Um livro tem 300 pginas, numeradas de 1 a 300. A quantidade de vezes que o algarismo 2 aparece na numerao das pginas desse livro : a) 140 b) 142 c) 150 d) 154 e) 160 5. (PRF) Escrevendo-se os inteiros de 1 at 537,determine quantas vezes aparecer o algarismo 8. a) 101 b) 103 c) 105 d) 107 e) 109 6.(BB) A soma dos dois algarismos de um nmero 12. Se trocarmos a ordem desses algarismos, o nmero aumenta em 18 unidades. Determine a tera parte desse nmero. a) 15 b) 17 c) 19 d) 21 e) 24 7.(UECE) Dado um nmero de dois algarismos, forma-se um novo nmero de trs algarismos, colocando 1" direita do nmero original; o novo nmero assim formado : a) Dez vezes o nmero original, mais um. b) Cem vezes o nmero original, mais um. c) Cem vezes o nmero original d) Dez vezes o nmero original. 8.(UECE) Sejam ab e ba dois nmeros de dois algarismos. Se a mdia aritmtica entre estes dois nmeros 66, o valor de a + b : 22

MATEMTICA PASSO A PASSO

a) 10

b) 11

c) 12

d) 13

9. (UFC) Um nmero positivo N, de dois algarismos, tal que, ao inverter-se os dois algarismos, o novo nmero assim formado excede N em 27 unidades. Se a soma dos algarismos de N igual a 11, qual o valor de N ? a) 29 b) 38 c) 47 d) 56 e) 65 10.(UNIFOR) Seja o nmero AB, onde A e B so os algarismos das dezenas e das unidades, respectivamente. Se BA o nmero obtido permutando-se os algarismos A e B, ento AB BA sempre: a) Zero b) Nmero Primo c) Quadrado perfeito d) Divisvel por 5 e) Mltiplo de 9

11. (BNB) Do maior nmero possvel de ser digitado em uma calculadora com lugar para oito algarismos foi subtrado o nmero de habitantes de um dos estados do Nordeste, obtendo-se como resultado, 92.582.597. somando-se uma nica vez os nmeros de um algarismo obtidos dos algarismos que compem o nmero de habitantes desse estado obtm-se: a) 16 b) 41 c) 14 d) 51 e) 15 12. (AMC) Quantos nmeros de dois algarismos distintos existem cuja soma dos algarismos 8? a)6 Respostas 1. C 2. B 3. A 4. E 5. B 6. C 7) A 8. C 9. C 10) E 11) C 12) B b) 7 c) 8 d) 9 e) 10

23

PROF. WELLINGTON BRITO

EXERCCIOS Resolver as seguintes expresses aritmticas:


1) 2) 3) 4) 5) 6) 7) 8) 9) 31 + {18 + [ ( 7+ l5 ) + 2 + (3+1) ] + 15} 7+5x82x4 8+7x43x5+7 28 5 x 4 (40 8 x 4) + 10 ( 5 x 7 + 3 ) x 6 + ( 12 3 x 2) x 5 123 { 150 + [ 36 ( 7 x 4 + 3 x 2 ) + 5 ] } x 2 285 3 x { 25 + 2 x [18 3 x (15 2 x 5 ) ] } [ 12 ( 3 + 2 x 3 ) ] + 15 (2 + 6 : 2) (13 + 7) : 5 + 24 : [ 12 ( 3 + 2 x 3) ] 15 : (2 + 6 : 2)

10) [40 (11 6) x 2 + 15 ] : [ 3 + 3 x (12 5 x 2)] 11) { 16 + 8 x [ 28 (15 3) : (5 + 1) ] 24 : 3 } : (14 2 x 3) 12) { 230 3 x [ 24 6 x (11 2 x 4) : (5 x 4 11)] : 11} x 3 + 4 13) [ 60 : (5 x 12 50) ] : { 55 : [ 40 : 2 : ( 4 + 8 x 2) ] 52 } 14) { 120 : [ 72 : ( 53 x 13 680 ) + 22 ] } + (10 + 5 )

15) Observe a soma abaixo: 1 a 3 A soma dos algarismos representados 1 7 b por a, b, e c igual a: + c 1 9 2 3 8 a) 10 b) 11 c) 12 d) 13 e) 14 9 5 7 16) 2 x y z x 4 1 0 1 z 8 Determinando-se os algarismos x, y e z para que a multiplicao seja verdadeira, verifica-se que: a) z = 7 b) x = y + z c) x = 2z d) y = x e) x + y z = 0 17) Escrevendo de 385 a 829 includos esses nmeros, quantos nmeros inteiros existem? a) 440 b) 442 c) 443 d) 444 e) 445 24

MATEMTICA PASSO A PASSO

18) Calcular o nmero de algarismos necessrios para escrever todos os nmeros naturais de 1 at 88. a) 167 b) 168 c) 169 d) 170 e)171

19) Quantos algarismos so necessrios para numerar as 934 pginas de um livro? a) 2684 b) 2690 c) 2692 d)2694 20) Determinar o nmero de algarismos necessrios para se escrever os nmeros pares de 6 at 281, inclusive. a) 361 b) 363 c) 365 d) 367 e)369 21) Para enumerar as pginas de um livro foram necessrios 270 algarismos. Quantas pginas tem esse livro? a) 126 b) 125 c) 124 d) 123 e) 120 22) Uma pessoa, para numerar as pginas de um lbum, cobrou R$ 15,30. Quantas pginas tem o lbum, sabendo-se que ela cobra R$ 0,05 por algarismos? a) 160 b) 157 c) 138 d)153 e)155 23) Determine o nmero de vezes que o algarismo 8 aparece na sucesso dos nmeros de 1 at 1.000? a) 280 b) 300 c) 350 d) 380 e) 400 24) Escrevendo-se os inteiros de 1 at 537, quantas vezes aparecer o algarismo 8? a) 101 b) 102 c) 103 d) 104 e) 105 25) Determine o nmero de vezes que o algarismo 4 aparecer quando se escreve de 1 at 327. a) 65 b) 64 c) 63 d) 62 e) 60 26) Escrevendo-se a srie natural dos nmeros inteiros, sem separar os algarismos, obtm-se: 1234567891011121314... Assim, o algarismo que ocupa o 1173 lugar : a) 3 b) 4 c) 5 d) 6 e) 7 27) Escrevendo-se a srie natural dos nmeros inteiros, sem separar os algarismos. Determine o algarismo que ocupa 0 1200 lugar. a) 5 b) 6 c) 7 d)8 e) 9 28) Escrevendo os nmeros inteiros, a partir da unidade e sem separar os algarismos, que algarismo ocupar a 1536 posio? a) 5 b) 6 c) 7 d)8 e) 9 25

PROF. WELLINGTON BRITO

Respostas 1) 92 8) 13 2) 39 9) 9 3) 28 10) 5 4) 258 11) 27 5) 192 12) 676

13) 2 14) 19 15) D 16) B 17) E

18) A 19) D 20) C 21) A 22) C

23) B 24) C 25) D 26) E 27) B

28) D

MLTIPLOS E DIVISORES Mltiplo e Divisor de um nmero Consideremos os termos divisvel, divide, divisor e fator. Nos seguintes produtos observemos que:
0 divisvel por 3 3x0 =0 03 =0 3 divisor de 0 3 divisvel por 3 3x1= 3 33 = 1 3 divisor de 3 6 divisvel por 3 3x2= 6 63= 2 3 divisor de 6 3x2 =6 63 = 2 3x1 = 3 33 = 1 Tambm poderamos dizer: 0 mltiplo de 3 3x0 = 0 03 =0 3 fator de 0 3 divide o 0 3 mltiplo de 3 3 fator de 3 3 divide o 3 6 mltiplo de 3 3 fator de 6 3divide o 6

De um modo geral, consideremos o conjunto: IN = { 0,1,2,3,4......n } e os subconjuntos de IN que indicaremos por M(6), ou conjunto dos mltiplos de 6, e D(6), ou conjunto dos divisores de 6. Assim: M(6) = { 0,6,12,18,24....} Podemos, pois, dar as definies: Mltiplo de um nmero o produto desse nmero por um natural qualquer. D(6) = {1,2,3,6}

Um nmero b divisor de um nmero a,se existir um natural c tal que b.c = a

Ento, quando b . c = a, podemos afirmar equivalentemente: b divisor de a ou c divisor de a a divisvel por b ou a divisvel por c a mltiplo de b ou a mltiplo de c 26

MATEMTICA PASSO A PASSO

Observaes: Divisores

b fator de a ou c fator de a. Mltiplos

a) Zero mltiplo de qualquer nmero. a)Zero no divisor de nmero algum b) Todo nmero mltiplo de si mesmo b)Todo nmero divisor de si mesmo c)O conjunto dos mltiplos de um c) O conjunto dos divisores de um nmero infinito. nmero finito.

Princpios Gerais de Divisibilidade.


5 divide 50 5 divide (50 + 20) ou e 5 divide 70 5 divide 20 5 divide (50 - 20) ou 5 divide 30. 3 divide 6 12 mltiplo de 6 18 mltiplo de 6 3 divide 12 ou 3 divide 18

1 Princpio: 2 Princpio: Se um nmero a divide outros dois, Se um nmero a divide um b e c, entoa divide a soma e a nmero b ento a divide diferena destes nmeros. tambm os mltiplos de b. Divisibilidade - Passo a Passo Para se verificar se um nmero divisvel por outro, no necessrio, em todos casos efetuar-se a diviso. Deduz-se um conjunto de regras que permitem verificar quando um nmero divisvel por um segundo. Essas regras constituem o que se chama os Caracteres de Divisibilidade. DIVISIBILIDADE POR 10, 2 E 5 I) Divisibilidade por 10 Um nmero divisvel por 10 quando termina em zero. Exemplos: 160,120,31.200 etc. II) Divisibilidade por 2 Um nmero divisvel por 2, quando o algarismo das unidades for par. III) Divisibilidade por 5 Um nmero divisvel po 5 quando o algarismo das unidades for zero ou 5. Exemplos: 405, 310, 1.100 etc. DIVISIBILIDADE POR 4 E 25. 27

PROF. WELLINGTON BRITO

Um nmero divisvel por 4 ou por 25, quando terminar em 00, ou quando os algarismos dasdezenas e unidades formarem um nmero divisvel por 4 ou 25. Exemplos: 1016 divisvel por 4 porque 16 tambm o . 204150 divisvel por 25 porque termina em 50 que divisvel por 25.

DIVISIBILIDADE POR 8 E 125 Um nmero divisvel por 8 ou por 125, quando os algarismos das centenas, dezenas e unidades forem 000, ou, nessa ordem, formarem um nmero divisvel por 8 ou 125. Exemplos: 24 000 divisvel por 8 e por 125. 54 104 divisvel por 8 porque 104 o . 321 250 divisvel por 125 porque 250 o . DIVISIBILIDADE POR 3 E 9 I) Divisibilidade por 9 Um nmero divisvel por 9 quando a soma dos valores absolutos dos seus algarismos for um nmero divisvel por 9. Tomando-se um nmero qualquer como exemplo: 7 434 podemos decomp-lo em suas unidades, ou seja: 7 434 = m . 9 + ( 7 + 4 + 3 + 4) II) Divisibilidade por 3 Um nmero divisvel por 3 quando a soma dos valores absolutos de seus algarismos der um nmero divisvel por 3. Exemplo: 1) 57 divisvel por 3 porque 5 + 7 = 12 .E 12 tambm divisvel por 3. 2) 5014 no divisvel por 3 porque 5 + 0 + 1 + 4 = 10. E 10 no divisvel por 3. DIVISIBILIDADE POR 7 E POR 11. I) Divisibilidade por 7 Vamos verificar se o nmero 343 divisvel por 7. Procedese do seguinte modo: a) Separa-se, do nmero dado, o algarismo das unidades. O dobro deste subtrai-se do nmero que se obteve com essa separao. Esquematicamente: 3 4 3 34 - 6 dobro de 3 28

MATEMTICA PASSO A PASSO

2 8

diferena

b) Se a diferena obtida for um mltiplo de 7 (no caso obtivemos 28 que mltiplo de 7), ento, o nmero dado tambm ser mltiplo de 7. Conclumos que 343 mltiplo de 7. Exemplo: 1) Verificar se 4 802 divisvel por 7. a) Separa-se o algarismo das unidades e dobra-se o valor desse nmero. 480 2 E o dobro de 2 4. b) Subtrai-se esse dobro, do nmero que ficou aps retirado o algarismos das unidades: 4804=476 Como ainda no se sabe se 476 divisvel por 7, repete-se o processo, agora com o nmero 476. 4 7 6 O dobro de 6 12. 47 12 = 35 Como 35 divisvel por 7, ento, o nmero 4 802 tambm o . II) Divisibilidade por 11 A divisibilidade por 11 semelhante divisibilidade por 7 e mais simples ainda. Basta obedecer regra: a) Separa-se, do nmero dado, o algarismo das unidades. b) Subtrai-se esse nmero, que representado pelo algarismo das unidades, do nmero que ficou aps sua retirada. Se a diferena for um nmero divisvel por 11, ento, o nmero dado tambm ser divisvel por 11. Exemplos: 1) Verificar se 121 divisvel por 11. a) Separa-se o ltimo algarismo da direita: 1 2 1 (separamos o nmero 1) b) Subtrai-se 12 1 = 11 Como 11 divisvel por 11, ento, 121 tambm o . 2) Verificar e 7 425 divisvel por 11 Aplicando-se sucessivamente a regra anterior: a) 7 4 2 5 (separamos o 5) b) 742 5 = 737 (subtramos o 5). 29

PROF. WELLINGTON BRITO

Deve-se verificar, pelo mesmo processo, se 737 divisvel por 11. a) 7 3 7 (separamos o 7). b) 73 7 = 66 (subtramos o 7 e obtivemos 66). Como 66 divisvel por 11, ento 7 425 tambm o .

EXERCCIO: MLTIPLOS E DIVISORES 1) O menor nmero de dois algarismos que se deve colocar direita do nmero 356 para que o mesmo seja divisvel por 2, 3 e 5 : a) 10 b) 20 c) 40 d) 22 2) O menor nmero que se deve adicionar a 58315 para se obter um nmero divisvel por 6 : a)1 b) 5 c) 15 d) 2

3) O menor nmero que se deve subtrair de 3101 para se obter um nmero divisvel por 8 : a) 3 b) 23 c) Zero d) 5 4) Qual das afirmaes abaixo falsa: a) Todo nmero par divisvel por 2. b) Todo nmero impar divisvel por 3. c) Todo nmero terminado em 0 divisvel por 5. d) Todo nmero terminado em 5 divisvel por 5. 5) Qual das afirmaes abaixo verdadeira: a) 15 divisor de 5 c) 13 divisor de 39 b) 2 divide 15 d) 15 divide 3 6) Se um nmero divisvel por 2 e 3, ento ele divisvel por: a) 5 b) 12 c) 6 d) 9 7) Se um nmero divisvel por 9, ento ele: a) sempre divisvel por 3 c) divisvel por 3, algumas vezes b) nunca divisvel por 3 d) divisvel por 6 8) Se um nmero divisvel por 3 e por 4, ento, ele: a) divisvel por 18 c) nunca divisvel por 12 b) sempre divisvel por 7 d) sempre divisvel por 12 9) O nmero 3 divide 12 e tambm divide 15. Ento: a) 3 divide 15 + 12 c) 3 no divide 15 x 12 30

MATEMTICA PASSO A PASSO

b) 3 no divide 15 12

d) 3 divide 15 : 12

10) O nmero 12 divisvel por 4 e por 6 (dentre outros nmeros). ento, podemos dizer que: a) 12 divisvel por 4 x 6 b) 12 divisvel por 6 4 c) 12 divisvel por 6 : 4 d) 12 divisvel por 6 + 4 nmero nmero nmero nmero

11) Se 2 o resto da diviso de um nmero por 3, ento: a) adicionando-se 2 ao dividendo, obtm-se um divisvel por 3. b) subtraindo-se 1 do dividendo obtm-se um divisvel por 3. c) adicionando-se 1 ao dividendo obtm-se um divisvel por 3. d) dividindo-se o dividendo por 2 obtm-se um divisvel por 3. 12) O resto da diviso de um nmero por 5 2 ento: a) (n+2) divisvel por 5 b) (n2) divisvel por 5 c) (n+1) divisvel por 5 d) (n1) divisvel por 5

13) Colocar V ou F nas seguintes afirmaes, conforme elas sejam verdadeiras ou falsas: a) 4 314 divisvel simultaneamente por 2 e por 3 b) 5 314 divisvel simultaneamente por 2 e por 5 c) 2 130 divisvel simultaneamente por 6 e por 5 d) 43 186 divisvel por 11 e) 20 010 divisvel por 6 e por 9 f) 41 310 divisvel por 2 e por 9 g) 37 212 divisvel por 2 e por 9 h) 32 715 divisvel por 5 e por 9 i) 5 101 350 divisvel por 5 e por 6 j) 5 002 446 divisvel por 2, 3 e 9 Respostas 1) A 4) B 7)A 31 10)B ( ( ( ( ( ( ( ( ( ( ) ) ) ) ) ) ) ) ) )

PROF. WELLINGTON BRITO

2) B 3) D 13) a) V f) V

5) C 6) C b) F g) F

8)D 9)A c) V h) V

11)C 12)B d)V i) V e) F j) F

NMEROS PRIMOS E NMEROS COMPOSTOS Definies Na sucesso IN = { 0, 1, 2, 3.....n} verifica-se que: O divisvel por qualquer nmero O 1 divisvel apenas por 1 2 divisvel por 1 e 2 3 divisvel por 1 e 3 4 divisvel por 1, 2, e 4 5 divisvel por 1 e 5 6 divisvel por 1,2,3 e 6 7 divisvel por 1 e 7

Categoria P

Categoria C

Ora, fcil ver que, com exceo da unidade, os nmeros se dividem em duas categorias: Nmeros Primos: aqueles que somente so divisveis por si mesmo e pela unidade. Nmeros Compostos ou Mltiplos: aqueles que admitem outros divisores alm deles prprios e da unidade. Logo,se P o conjunto dos nmeros primos,ento: P = { 2,3,5,7,11,13....} Nmeros Primos - Passo a Passo O reconhecimento dos nmeros primos se faz por um processo prtico que se baseia no fato que: Todo nmero mltiplo admite pelo menos um divisor primo O reconhecimento se baseia na regra prtica: 32

MATEMTICA PASSO A PASSO

Divide-se o nmero dado pelos nmeros da sucesso dos nmeros primos: 2,3,5,7,11,13,...obtendo-se um quociente e um resto. Se o resto for diferente de zero at o instante em que o quociente se torna menor ou igual ao divisor, pode-se afirmar que o nmero primo.

Exemplos: 1) Verificar se o nmero 47 primo ou composto: Pela regra, faz-se: 47 3 47 5 47 7 17 15 2 9 5 6 2 Neste instante, obtivemos o quociente 6 e o divisor 7, isto , o quociente menor que o divisor.Afirmamos: o nmero 47 primo 2) Verificar se 289 primo. 289 no divisvel por 2, 3, 5, 7, 11, vejamos por 13, 17, 19... 289 13 29 22 3 289 17 119 17 00

Como o resto zero, ento 289 mltiplo de 17. Fatorao Decomposio de um Nmero em Fatores Primos Veja: 8 = 2 x 2 x 2 = 23 12 = 2 x 2 x 3 = 22 x 3 Todo nmero mltiplo pode ser 15 = 3 x 5 decomposto de um s modo no 28 = 2 x 2 x 7 produto de vrios fatores primos. Basta usar o clssico processo e fazer: 8 2 12 2 15 3 4 2 6 2 5 5 2 2 3 3 1 1 1 33 28 2 14 2 7 7 1

PROF. WELLINGTON BRITO

8 = 23

12 = 22 x 3

15 = 3 x 5

28 = 22 x 7

Divisores de um Nmero - Passo a Passo

Um Processo prtico consiste em se fazer como no exemplo que segue, para o nmero 90. a) Decompe-se o nmero em seus fatores primos e direita da decomposio obtida traa-se um segmento de reta vertical. 1 90 2 45 3 15 3 5 5 1 b) Uma linha acima do 1 fator primo e direita do segmento vertical coloca-se o nmero 1. Efetua-se o produto do 1 fator primo (2) pelo nmero 1, colocando-se o produto (2) direita do trao. Multiplicam-se os seguintes fatores primos pelos nmeros que estiverem direita do trao vertical e acima desse fator. Os nmeros direita do trao vertical so os divisores do nmero pedido. No se repetem na multiplicao os divisores iguais. 1 90 2 2 45 3 3 6 15 3 9 18 5 5 5 10 15 30 45 90. 1 Os divisores de 90 so: 1, 2, 3, 6, 9, 18, 5, 10, 15, 30, 45 e 90. Questes Comentadas 1) Verificar se, sendo 60 um divisor de 180, todos os fatores primos de 60 tambm so fatores primos de 180. Decompondo-se em fatores primos, vem: 180 2 60 2 90 2 30 2 45 3 15 3 34

MATEMTICA PASSO A PASSO

15 3 5 5 1 180 = 2 x 2 x 3 x 3 x 5 Ou: 180 = 22 x 32 x 5

5 5 1 60 = 2 x 2 x 3 x 5 Ou: 60 = 22 x 3 x 5

2) Verificar que fator falta ao 504 para que se tenha um nmero divisvel por 210. 504 2 210 2 252 2 105 3 126 2 35 5 63 3 7 7 21 3 1 7 7 1 504 = 2 3 x 32 x 7 210 = 2 x 3 x 5 x 7 Com exceo do fator 5, todos os fatores de 210 esto contidos nos fatores de 504, portanto falta ao 504 o fator 5. De fato, multiplicando-se 504 por 5, obtm-se 2 520 que mltiplo de 210. 3) Qual o menor nmero pelo qual se deve multiplicar 210 a fim de se obter um nmero divisvel por 84? Decompondo-se em fatores primos, vem: 210 = 2 x 3 x 5 x 7 84 = 22 x 3 x 7 Ao 210 falta apenas o fator 2, pois 2 1 fator de 210 e 22 fator de 84. Todos os outros fatores de 84 pertencem a 210.

EXERCCIO: NMEROS PRIMOS E FATORAO


1) Reconhecer se so primos os seguintes nmeros: a) 289 e) 521 b) 343 f) 421 c) 731 g) 997 d) 1.111 h) 409 2) Decompor em fatores primos os seguintes nmeros: a) 160 f) 1024 b) 210 g) 729 c) 250 h) 1728 35

PROF. WELLINGTON BRITO

d) 289 e) 243

i) 11907

3) Decompor em fatores primos os seguintes nmeros, sem efetuar a multiplicao indicada: a) 504 x 240 b) 720 x 243 4) Sem efetuar as seguintes potncias, decomposio em fatores primos: dar a sua

a) 8403 b) (2432)3

c) (1202)3 d) (10243)4

5) Dizer quantos divisores possui cada um dos nmeros seguintes, sem dizer quais so: a) 420 a) 105 b) 960 b) 240 c)1260 c) 840 6) Dizer quais so os divisores dos nmeros seguintes: 7) Pela decomposio em fatores primos, verificar, sem efetuar a diviso, se 4374 divisvel por 686. 8) Pela decomposio em fatores primos, determinar qual o menor nmero pelo qual se deve multiplicar 3 675 a fim de se obter um nmero divisvel por 490.

9) Sendo A = 23 x 3 x 52 e B = 2n x 5, determinar o maior


valor possvel de n, de modo que B seja divisor de A.

10) Sendo A = 3x x 52 x 7 e B = 35 x 7, determinar o menor


valor possvel de x, de modo que A seja mltiplo de B.

11) Se: A = 23 x 52 x 11 e B = 22 x 3 x 52, qual o maior divisor


de A e de B simultaneamente?

12) Sendo A = 23 x 52 x 7n determinar n, de modo que A tenha


60 divisores.

13) Sendo A = 2 x 3x determinar x, de modo que A tenha 18


divisores. Respostas
1) a) b) c) d) Compostos 2) e) f) g) h) Primos 3) a) 27 x 33 x 5 x 7 b) 24 x 37 x 5 7)No 8) 2 9) 3 10) 5

36

MATEMTICA PASSO A PASSO

4) a) 29 x 33 x 53 x 73 b) 330 c) 218 x 36 x 56 d) 2120 5) a) 24 b) 28 c) 36

11) 100 12) 4 13) 8

MXIMO DIVISOR COMUM E MNIMO MULTIPLO COMUM Mximo Divisor Comum ( m.d.c) Sejam os nmeros 12, 18 e 30 e os conjuntos D(12), D(18) e D(30) de seus respectivos divisores, que so finitos e ordenados. D (12) = { 1,2,3,4,6,12} D (18) = { 1,2,3,6,9,18} D (30) = { 1,2,3,5,6,10,15,30} Consideremos agora o conjunto dos divisores comuns, isto , o conjunto interseo de D(12), D(18) e D(30). D(12) D(18) D(30) = {1,2,3,6} Ento, definimos: Chama-se Mximo Divisor Comum de dois ou mais nmeros, ao maior valor da interseo dos conjuntos dos divisores dos nmeros dados. Logo: m.d.c (12, 18, 30) = 6

Clculo do m.d.c - Passo a Passo 1 Processo: Decomposio em Fatores Primos Para se calcular o m.d.c. de vrios nmeros, conclui-se a regra: a) Decompe-se os nmeros dados em seus fatores primos. b) Toma-se o produto dos fatores primos comuns a essas decomposies, cada um deles tomado com o menor dos expoentes que esse fator possui nas decomposies. Exemplo: Calcular o m.d.c (720, 420, 540): 720 = 24 x 32 x 5 420 = 23 x 3 x 5 x 7 37

PROF. WELLINGTON BRITO

540 = 2 2 x 33 x 5 m.d.c. (720,420,540) = 22 x 3 x 5 m.d.c. (720,420,540) = 60

Numeros Primos Entre Si

Procuremos o m.d.c entre 25 e 36. Sabe-se que: 25 = 52 e 36 = 2 2 x 32 Neste caso, os nmeros no tm fatores primos comum com exceo da unidade. Dizemos que o mximo divisor comum o nmero 1. Estes nmeros so chamados primos entre si, definindo-se, pois: Nmeros primos entre si divisor comum a unidade. so aqueles cujo nico

2) Processo: Mtodo das divises sucessivas (I) O nmero maior divisvel pelo menor. Seja calcular o m.d.c. entre 30 e 6. Como 6 divide 30 e ele prprio, ento 6 o maior divisor comum podendo-se escrever. m.d.c. (6, 30) = 6. E conclumos: Se o maior nmero divisvel pelo menor, ento, este menor o m.d.c de ambos. (II) O nmero maior no divisvel pelo menor. Para se achar o m.d.c. de dois nmeros, divide-se o maior pelo menor. A seguir, divide-se o menor pelo resto da diviso entre o maior e o menor. A seguir divide-se o 1 resto pelo 2 resto e assim sucessivamente. Quando se obtiver um resto zero,o ltimo divisor o m.d.c. procurado. Exemplo: Calcular o m.d.c (45, 36) Na prtica, faz-se 45 38 1 4 36 9 quociente divisores

MATEMTICA PASSO A PASSO

00

resto

Isto , quando o resto zero, o ltimo divisor (9) o m.d.c.

Mximo Divisor Comum de Mais de Dois Nmeros

Calcular o m.d.c. (240, 180, 72, 54). Neste caso, basta usar qualquer um dos esquemas seguintes, onde chamamos de R 1 e R2 os resultados parciais e R o resultado final. ESQUEMA 240 180 72 54 R1 R 1 3 240 180 60 060 00 R1 = 60 3 60 18 6 00
R=6
final

R2

1 3 72 54 18 18 00 R 2 = 18 3 6
ou m.d.c (240, 180, 72, 54) = 6

EXERCCIOS: MXIMO DIVISOR COMUM 1) Calcular o mximo divisor comum, pelo processo das divises sucessivas dos seguintes nmeros: a) 576 e 96 c) 168, 252 e 315 b) 576 e 708 d) 192, 256 e 352 e)1 980, 2 700 e 3 060 2) No Clculo do m.d.c. de dois nmeros, pelas divises sucessivas, obteve-se o seguinte esquema. Preencher com nmeros os lugares assinalados com x. 2 6 1 2 x x x x 6 x x x 0 39

PROF. WELLINGTON BRITO

3 )No m.d.c. de dois nmeros, pelas divises sucessivas, obtevese como quociente os nmeros 3, 6, 1 e 3. Sabendo-se que o m.d.c. 4, determinar os nmeros 4) O m.d.c. de dois nmeros 12 e os quocientes obtidos no esquema das divises sucessivas so 1, 3 e 2. Quais so os nmeros ? 5) Calcular o mximo divisor comum, pelo processo da decomposio em fatores, dos seguintes nmeros: a) 1414, 910, 700 b) 264, 360, 432 e 378 e) 625,1331,343 e729 6) Sendo A = 23 x 32 x 5x e B = 2y x 37 x 53 e sendo C = 22 x 32 o m.d.c. de A e B, determinar os valores de x e y 7) Sendo A = 32 x 5m x 74 e B = 54 x 73 x 11 e sendo C = 7n o m.d.c. de A e B, determinar m e n. 8) Quais so os menores nmeros pelos quais devemos dividir, respectivamente, 12 e 15 a fim de obter quocientes iguais? 9) Quais so os menores nmeros pelos quais devemos dividir, respectivamente, 216 e 168 a fim de obter quocientes iguais? 10) Calcular, pela decomposio em fatores primos o m.d.c. das potncias seguintes, sem efetu-las: (72.4 e (324)3 11) Calcular, sem efetuar as potncias, o m.d.c. dos seguintes nmeros: (350)2 e (450)4 12) Comprei uma partida de feijo de trs qualidades A, B, e C. A rimeira qualidade veio em sacas de 60 kg; a segunda qualidade em sacas de 72kg e a terceira em sacas de 42kg. Desejo vende-las a varejo em sacas de igual peso, sem misturar as qualidades e sem perder com restos. Devo acondicion-los em sacos de quantos quilogramas? Respostas 1) a) 96
b) 12 c) 21

c) 441, 567, 630 e 1029 d) 363, 2541, 3993

5) a) 14 b) 6 c) 21 d) 363 e) 1 so primos entre si

40

MATEMTICA PASSO A PASSO

d)32 2) 2 6 258 120 18 18 12 6 3) 340 e 108 4) 108 e 84

1 2 12 6 0

6) x = 0 y = 2 7) m = 0 n=3 8) 4 e 5 9) 9 e 7 10) 26 x 38 11) 22 x 54 12) 6 kg

Mnimo Mltiplo Comum - (m.m.c)

Consideremos os nmeros 3, 4, e 6 e o conjunto dos seus mltiplos, que chamaremos M(3), M(4) e M(6). M(3) = { 0, 3, 6, 9, 12, 15,18,......} M(4) = { 0, 4, 8, 12, 16, 20, 24,.........} M(6) = { 0, 6, 12 , 18, 24, 30,............} Cada um desses conjuntos infinito. O conjunto interseo tambm ser infinito, como se v: M(3) M(4) M(6) = { 12, 24, 36,.....} Chama-se Mnimo Mltiplo Comum de dois ou mais nmeros dados ao menor valor da interseo dos conjuntos dos mltiplos desses nmeros. Logo: m.m.c. (3,4,6) = 12 Clculo do m.m.c - Passo a Passo 1 Processo: Pela decomposio em fatores primos a) Decompem-se os nmeros em fatores primos. b) Toma-se o produto dos fatores primos comuns e no comuns a essas decomposies, cada um deles tomado com o maior dos exponentes que esse fator possui nas decomposies. Exemplo: Calcular o m.m.c. dos nmeros 105, 625 e 343 Decompondo-se, vem: 105 = 3 x 5 x 7 625 = 5 4 343 = 7 3 m.m.c. (105,625,343) = 3 x 54 x 73 = 3 x 625 x 343 m.m.c (105,625,343) = 643125 41

PROF. WELLINGTON BRITO

Na prtica, pode-se realizar a decomposio num nico dispositivo, conde os fatores primos comuns e no comuns ficam dispostos direita de um trao vertical que separa os nmeros dados desses fatores, como segue:

Calcular o m.m.c. de 90, 105 e 135: 90 105 135 45 105 135 15 35 45 5 35 15 5 35 5 1 7 1 1 1 1

2 3 3 3 5 7

m.m.c.(90,105,135) = 2 x 3 x 3 x 3 x 5 x 7= 2 x 33 x 5 x 7. Ou: m.m.c. (90,105,135) = 1890. Propriedades do m.m.c. 1 Propriedade No m.m.c. de dois ou mais nmeros, se o maior deles o mltiplo dos outros, ento o maior o mnimo mltiplo comum de todos. Exemplo: m.m.c.(60,12,15,10) 60 mltiplo de si mesmo e tambm de 12, 15 e 10 Logo: Ou: 60 o m.m.c. dos nmeros: 60,12,15 e 10 m.m.c. (60,12,15,10) = 60

2 Propriedade O produto de dois nmeros, A e B, igual ao produto do m.d.c. pelo m.m.c desses nmeros A e B. Sejam os nmeros: A = 15 e B = 18. Teremos: m.d.c (15, 18) = 3 m.m.c(15, 18) = 90 Representando-se o m.d.c. (15,18) por (15,18) e o m.m.c (15,18) por (15,18), vir: 42

MATEMTICA PASSO A PASSO

(15,18) x (15,18) = 3 x 90 = 270 E: Donde: 15 x 15 x 18 = 270 18 = (15, 18) x (15, 18).

Questes Comentadas: 1) Determinar o m.m.c. entre os nmeros 12 e 13. Como 12 e 13 so consecutivos e todos os consecutivos so primos entre si, pela primeira propriedade: m.m.c. (12,13) = 12 x 13 = 156. 2) Determinar os menores nmeros pelos quais se devem multiplicar 50 e 75, a fim de se obter produtos iguais. Basta determinar o m.m.c.(50,75) que 150 e depois efetuar as divises: 150 50 = 3; 150 75 = 2

Ento, deve-se multiplicar 50 por 3 e 75 por 2, obtendo-se o produto 150 em ambos os casos. 3) Numa avenida que mede 4500 metros, a partir do incio, a cada 250m h uma parada de nibus e a cada 225 metros uma de bonde. Pergunta-se: a) A que distancia do incio coincide a primeira parada de nibus com a de bonde? b) Quantos so os pontos comuns de parada de nibus e bonde? Raciocinando: 1) A primeira parada comum de bonde e nibus o menor mltiplo comum de 250m e 225m. Logo: m.m.c. (250, 225) = 2250m. Portanto: A primeira parada comum est a 2250m do inicio. 43

PROF. WELLINGTON BRITO

As outras paradas sero mltiplas de 2250m. A 2 x 2250m estar a segunda parada comum, onde termina a avenida (4500m).

EXERCCIOS: MNIMO MLTIPLO COMUM 1) Calcular o m.m.c. dos seguintes nmeros pela decomposio em fatores primos: a) 18, 30 e 48 d) 200, 40, 50 e 20 b) 120, 300 e 450 e) 60, 84, 132 e 120 c) 18 e 108 f) 1024, 512, 729 e 81 2) Sendo A = 25 x 3a x 5b e B = 2c x 37 e sendo C = 27 x 38 x 52, o m.m.c de A e B, determinar a, b e c. 3) Sendo A = 33 x 5x x711 e B = 25 x 38 e sendo C = 2y x 3z x 711 x 54, determinar, x, y, z 4) Sendo A = 22 x 3 x 53 e B = 23 x 52 x 11, determinar o quociente da diviso do seu m.m.c. pelo seu m.d.c. 5) Calcular o m.m.c. dos nmeros seguintes pela decomposio simultnea em fatores primos: a) 42, 72 e 108 c) 1225, 1715 e 70 b) 160, 64 e 512 d) 121, 110, 66 e 363 6) Quais os nmeros compreendidos entre 100 e 1000, mltiplos ao mesmo tempo de 12, 9 e 30? 7) Quais os nmeros compreendidos entre 100 e 2000, que so mltiplos de 36, 45 e 54? 8) Quais so os menores nmeros pelos quais se devem multiplicar respectivamente 63 e 42 a fim de se obter produtos iguais? 9) O m.d.c. de dois nmeros 20 e o seu m.m.c. 120. Um dos nmeros 20. Determinar o outro. 10) O produto de dois nmeros 1470 e o seu m.d.c. 7. Calcular o m.m.c. 11) O m.m.c. de dois nmeros primos entre si 221. Um deles 13. Quanto vale o outro? 12) O m.d.c. de dois nmeros a unidade e o mnimo mltiplo comum deles 29403. Um dos nmeros 112. Qual o outro? 44

MATEMTICA PASSO A PASSO

Respostas
1)a) 720 2) a = 8 b)1800 b=2 c) 342 c)= 7 d) 200 e)9240 f )746496 3) x = 4 y=5 z=8 5) a) 1512 7) 540,1080,1620 b) 2560 8) 2 e 3 c) 17 150 9) 120 d) 3 630 10) 210 11) 17 12) 243 4) 330 6) 180,360,540,720,900

QUESTES DE CONCURSOS E VESTIBULARES 01). (TRT) Seja A7B um nmero inteiro e positivo de trs algarismos, no qual B e A representam os algarismos das unidades e das centenas, respectivamente. Para que esse nmero seja divisvel por 15, calcule quantas possibilidades de escolha temos para A7B. a) 6 b) 7 c) 8 d)9 e)10 02). (UECE) Quantos nmeros naturais existem entre 10 e 100, divisveis simultaneamente por 2, 5 e 9? a) Nenhum b) um c) dois d) trs 03). (UNIFOR) Se o mximo divisor comum dos nmeros inteiros A=23 x 33, B= 23 x 3s x 7 e C= 2t x 34 igual a 12, ento: a) t=3 b) t=2 c) s=0 d)s=2 e)t=1 04). (UNIFOR) Seja n a diferena entre o maior nmero inteiro com 6 algarismos distintos e o maior nmero inteiro com 5 algarismos distintos. A soma dos algarismos de n um nmero: a) Primo c) divisvel por 11 e) mltiplo de 5 b) Par d) quadrado perfeito 05.(TRE) Sabe-se que o M.D.C. dos nmeros A= 2x x 33 x 54, B = 23 x 3y x 52 e C = 24 x 34 x 5z igual a 180. Nessas condies x+ y + z igual a: a) 2 b) 3 c) 4 d)5 e)6 06.(TRT) A associao de funcionrios de certa empresa promove palestras regulares: uma a cada 3 meses, outra a cada 6 meses e outra a cada 8 meses. Se, em 2000, as trs palestras foram dadas em julho, a prxima coincidncia de poca das palestras ser em: a) Junho de 2001 c) Julho de 2001 e) Julho de 2003 b) Junho de 2002 d) Julho de 2002 07.(CEF) Numa pista circular de autorama, um carrinho vermelho d uma volta a cada 72 segundos e um carrinho azul d uma volta a cada 80 segundos. Se os dois carrinhos partiram 45

PROF. WELLINGTON BRITO

juntos, quantas voltas ter dado o mais lento at o momento em que ambos voltaro a estar lado a lado no ponto de partida? a) 7 b) 8 c)9 d)10 e)11

08.(TRT) Trs funcionrios fazem plantes nas sees em que trabalham: um a cada 10 dias, outro a cada 15 dias, e o terceiro a cada 20 dias, inclusive aos sbados, domingos e feriados. Se no dia 18/05/02 os trs estiveram de planto, a prxima data em que houve coincidncia no dia de seus plantes foi: a) 18/09/02 c))18/08/02 e)18/07/02 b) 17/09/02 d)17/07/02 09.(UECE) Dois relgios tocam uma msica periodicamente, um deles a cada 60 segundos e o outro a cada 62 segundos. Se ambos tocaram (simultaneamente) s 10 horas, que horas estaro marcando os relgios quando voltarem a tocar juntos (simultaneamente) pela primeira vez aps s 10 horas? a) 10 horas e 31 minutos b) 10 horas e 41 minutos c) 10 horas e 51 minutos d) 11 horas e 01 minuto

10.(TRT) No almoxarifado de certa repartio pblica h trs lotes de pastas iguais: o primeiro com 60, o segundo com 105 e o terceiro com 135 pastas. Um funcionrio deve empilh-las, colocando cada lote de modo que, ao final de seu trabalho, ele tenha obtido pilhas com igual quantidade de pastas. Nestas condies, o menor nmero de pilhas que ele obter : a) 10 b) 15 c)20 d)60 e)120 11.(BB) Uma pessoa tem duas folhas de cartolina, ambas quadradas e com superfcie de 2.304cm2 e 1296cm2.Ela deseja recort-las em pequenos quadrados, todos iguais e de maior rea possvel. O lado de cada quadradinho, em centmetros, medir: a) 11 b) 12 c)13 d)14 e)15 12.(TRT) Uma enfermeira recebeu um lote de medicamentos com 132 comprimidos de analgsico e 156 comprimidos de antibitico. Ela dever distribu-los em recipientes iguais, contendo, cada um, a maior quantidade possvel de um nico tipo de medicamento. Considerando que todos os recipientes devero receber a mesma 46

MATEMTICA PASSO A PASSO

quantidade de medicamento, o nmero de recipientes necessrios para essa distribuio : a) 24 01)A 02)B 03)B 04)D b) 20 05)D 06)D 07)C 08)D c) 18 Respostas 09)A 11)B 10)C 12)A d)16 e)12

NMEROS INTEIROS Introduo LUCROS E PREJUZOS Os resultados financeiros de uma empresa, nos dois semestres, foram:
1 Semestre Prejuzo de R$ 40.000,00

+R$ 20.000,00 1 Sem. 2 Sem.

2 Semestre

Lucro de R$ 20.000,00

- R$ 40.000,00 Para diferenciar essas duas situaes, podemos indicar o Lucro com o sinal de + e o Prejuzo com o sinal de . O Conjunto dos Nmeros Inteiros

O conjunto formado por todos os nmeros inteiros Negativos, pelo Zero e por todos os nmeros inteiros Positivos chamado de Conjunto dos Nmeros Inteiros Relativos. O conjunto dos nmeros inteiros relativos indicado pela letra . Assim: = { ...., -3,-2,-1,0, +1, +2, +3, ....} Observaes: Todo elemento do conjunto dos nmeros naturais (IN) tambm elemento do conjunto dos nmeros inteiros relativos () Da: IN Representao Geomtrica dos Nmeros Inteiros - Passo a Passo: 47

PROF. WELLINGTON BRITO

Marcamos arbitrariamente sobre uma reta um ponto 0, que chamamos de origem. Esse ponto representa o nmero zero. A partir de 0, estabeleceremos um sentido Positivo (+) e um sentido Negativo(). () Negativo 0 (+) Positivo

Escolhemos uma medida conveniente ( 1cm, por exemplo) e marcamos direita de 0 pontos consecutivos, distantes entre si 1cm. Para cada um desses pontos faremos corresponder um nmero inteiro Positivo. 0 A B C D 0 1 2 3 4 De maneira anloga, representamos esquerda de 0 os nmeros inteiros Negativos. D' C' B' A' 0 A B C D -4 -3 -2 -1 0 1 2 3 4 A reta assim marcada chamada Reta Numrica Inteira. Temos: O ponto B a imagem geomtrica do Nmero Inteiro 2. O ponto C' a imagem geomtrica do Nmero Inteiro 3. Valor Absoluto ou Mdulo

Valor absoluto ou mdulo de um nmero inteiro relativo, o nmero sem o sinal, ou seja, a distncia do ponto correspondente a um nmero inteiro at o referencial zero. Indica-se o mdulo, escrevendo-se nmero entre duas barras I n I (l-se: mdulo de n). Exemplos: -5 -4 -3 -2 -1 0 1 2 3 4 5 3 I3I=3 I3I=3 I0I=0 I1I=1 3 I 10 I = 10

Nmeros Opostos ou Simtricos 48

MATEMTICA PASSO A PASSO

Os pontos que representam os nmeros inteiros 3 e 3 esto a mesma distncia da origem. Por esse motivo, dizemos que 3 e 3 so Nmeros Opostos ou Nmeros Simtricos. Exemplos: 5 o oposto de 5 -n o oposto de n - 4 o oposto de 4 x o oposto de -x

Comparao de Nmeros Inteiros - Passo a Passo Comparar dois nmeros inteiros, a e b, significa verificar se: a=b ou a>b ou a<b

1) O zero maior que qualquer nmero inteiro negativo. 0 > -1 0 > -13 0 > -20 2) O zero menor que qualquer nmero inteiro positivo. 0<1 0<3 0 < 10 3) Qualquer inteiro positivo maior do que qualquer inteiro negativo. 1 > -8 2 > -2 5 > -10 4) Entre dois inteiros positivos, o maior o que possui o maior mdulo. 3>1 5>3 10 > 5 5) Entre dois inteiros negativos, o maior o que possui o menor mdulo. -1 > -3 -5 > -10 -3 > -5 EXERCCIO NMEROS INTEIROS Diga quantas unidades aumentamos ( ou diminumos) ao passar de: a) 4 para +4 d) 5 para 0 b) +4 para -1 e) 6 para -2 c) +1 para -3 f) 3 para -10 Observe a figura e responda: D C P -4 -2 0 49

1)

2)

A 1

B 4

PROF. WELLINGTON BRITO

a) b) c) d) e)

Qual o nmero inteiro cuja imagem geomtrica o ponto A? Qual o nmero inteiro cuja imagem geomtrica o ponto D? O nmero inteiro 2 abscissa de qual ponto? O nmero inteiro 4 abscissa de qual ponto? Qual o ponto da abscissa zero ?

3)

Calcule o mdulo: a) I 5 I c) I 3I e) I15 I b) I 10I d) I10 I f) I 20I 4) Encontre o oposto: a) (-2) c) (-4) e) - (+9) b) (+12) d) [- ( - 3) ] f) -[ - ( +5)] 5) Complete usando > ou <: a) 15 _____ -12 d) 8 _____ -4 g) 4 _______0 b) 5 _____ 0 e) 0 _____ -10 h) 2 _______11 c) 10 _____ 2 f) 10_____ -3 i) -1 _______-8 6) Escreva em ordem crescente: a) 4, -1, 5, -3, 0, 1, -2 b) 1, -5, -10, 9, 18, -30, -20, 8 7) a) 8) Escreva em ordem decrescente: 3, 1, 5, 4, -5, 0, -1,10 b) 1, -5, -3, -15, 0, -18 Quais os trs prximos nmeros de cada seqncia? a) -105, -104, -103, -102, ______, ______, ______. b) 90, -80, -70, ______, ______, ______. c) 20, -15, -10, ______, ______, ______. Respostas
01)

a) Aumentamos 8 unidades b) Diminumos 5 unidades c) Diminumos 4 unidades


02) a) 1 03) a) 5 04) a) 2 b) 4 b) 10 b) 12 c) C c) 3 c) 4 d)B d) 10 d) 3

d) Aumentamos 5 unidades e) Aumentamos 4 unidades f) Diminumos 7 unidades e)P e)-15 e) 9 f) 20 f) 5

50

MATEMTICA PASSO A PASSO

06) a) 3,-2,-1,0,1,4,5 b) 30,-20,-10,-5,1,8,9,18 07) a) 10,5,4,1,0,-1,-3,-5 b) 0,-1,-3,-5,-15,-18 08) a) 101,-100,-99 b) 60, -50, -40 c) 5, 0, 5

OPERAES COM NMEROS INTEIROS Adio - Passo a Passo 1 Caso) Os nmeros possuem o mesmo sinal. D-se o sinal comum e soma-se os valores absolutos Exemplos: a) (+2) + (+5) = + (2 + 5) = + 7 c) ( 4) + ( 2) + ( 3) = b) ( 1) + ( 4) = (1 + 4) = 5 = (4 + 2 + 3) = 9 2 Caso) Os nmeros possuem sinais contrrios. D-se o sinal do maior mdulo e subtra-se Exemplos: a) (+ 7) + ( 2) = +( 7 2 )=+ 5 c) (12) + (+5) = - (12 5) = - 7 b) (+ 3) + ( 4) = ( 4 3 )= 1 d) ( 6 ) + (+8) = +( 8 6 ) = + 2 Subtrao - Passo a Passo Para subtrair Nmeros Inteiros Relativos, soma-se ao primeiro,o simtrico do segundo. Exemplos: a) c) (+9) ( 2) = + 9 + 2 = + 11 (+3) (+4) = + 3 4 = 1 b) (- 5) (+5) = 5 5 = 10

Multiplicao - Passo a Passo 51

PROF. WELLINGTON BRITO

A multiplicao de nmeros inteiros segue os critrios: Sinais iguais, resultado Positivo Sinais diferentes resultado Negativo

Exemplos: a) (+ 2) x (+3) = + ( 2 . 3) = +6 c) (+ 5) x (2) = ( 5 . 2) = 10 b) ( 4) x ( 2) = + ( 4 . 2) = +8 d) ( 3) x (+4) = ( 3 . 4) = 12 Diviso - Passo a Passo A diviso de nmeros relativos, segue os mesmos critrios da multiplicao, ou seja, sinais iguais resultado positivo e sinais diferentes resultado negativo. Exemplos: a) ( 9 ) ( 3) = + ( 9 : 3) = + 3 c) ( 8) (+2) = ( 8 : 2) = 4 b) (+10) (+2) = + ( 10 : 2) = +5 d) (+12) (-4) = (12: 4) = 3 EXPRESSES ARITMTICAS Acompanhe os exemplos: a) 20 + 15 18 + 37 Termos Positivos: +15 + 37 = +52 Termos Negativos: - 20 18 = - (20+18) = 38 Resultado: + 52 38 = + (52 38 ) = + 14 b) 17 5 8 + 5 17 + 3 = 17 17 5 + 5 8 + 3 =8+3=5 c) 7 ( 8 5 + 12) = 7 ( 3 + 12) = 7 ( +15) = 7 15 = 8 52 A soma de dois nmeros opostos sempre zero

MATEMTICA PASSO A PASSO

d) 50 { 18 + [ 7 ( 8 15) ] } =50 { 18 + [ 7 ( 7) ] } =50 { 18 + [ 7 + 7] } =50 { 18 + 14 } = 50 { 4} = 50 + 4 = 54 EXERCCIO EXPRESSES ARITMTICAS 1. Resolva as expresses: a) 5 + 3 1 b) 10 3 7 c) 15 18 + 5 3 d) 38 15 + 12 5 e) 104 30 10 + 16 f) 108 + 40 108 30 X Y X+Y 2 7 6 5 5 9 1 9 0 8 4 8 2. Complete a tabela abaixo: g) 4 + 5 + 3 7 2 8 h) 7 + 15 3 + 9 4 1 i) 53 + 79 18 7 + 15 39 + 18 j) 43 + 13 104 + 300 148 + 31

3. Calcule o valor das expresses: a) 2 ( 5 + 3 1) d) 35 [ 4 + (18 15) 3 ] b) 3 ( 5 4) + ( 2 17) e) { 12 + [ 5 10 (3 25) 37 ] } 4. Sendo x = 3 e Y = 2, calcule: a) x ( y + 4) b) 15 + ( x + y) c) y ( x 4 ) d) 8 ( y x )

5. Escreva o dobro, triplo, qudruplo e o quntuplo de: a) 4 b) 4 c)10 d) 10 6. a) b) c) Determine A metade de 50 A tera parte de 243 A quarta parte de 1200 53

PROF. WELLINGTON BRITO

d) A quinta parte de 175 7. Sendo a = 30, encontre o valor das expresses: a) 10a c) 3a : 2 b) a : 3 1 8. Resolva as Expresses a) 20 : 5 3 b) 5 + 7 . 3 4 : 2 a) x : ( 30) = 3 b) ( 100) : y = 1 c) z : 153 = 0 c) ( 5) : 5 (5) : (5) d) 5 . 8 (4) : 4 + 3 ( 5) + 12 : ( 4) d) t : ( 8) = 7 e) (f + 1) : ( 5) = 1 f) (30 + m ) : 16 = 2 d) (a + 5 ) : ( 7) 2

9. Qual o nmero inteiro que cada letra est representando:

10. Determine os prximos trs nmeros inteiros de cada seqncia abaixo: a) 2, 4 , 8, 16, _____, _____, _____. b) 1, 2, 6, 24, _____, _____, _____. c) 128, 64, 32, 16, _____, _____, _____. d) 5040, 720, 120, -24, _____, _____, _____. Respostas 01.
a) b) 7 0 c) 1 d) 30 b) 13 b)16 e) 80 f) 10 c) 31 c) 1 c) 20, 30, 40, 50 d) 20, 30, 40, 50 c) 300 d) 35 b) 9 b)14 c) 45 c) 2 d) 7 d)23 g) 5 h) 9 d) 32 d)13 i) 5 j) 49 03. a) 5 04. a) 1 05. a) 8, 12, 16, 20 b) 8, 12, 16, 20 06. a) 25 b) 81 07. a) 300 08. a) 1 09.

54

MATEMTICA PASSO A PASSO

a) x = -90 b) y = 100 10. a) 32, 64, -128 b)120, -720, 5040

c) z = 0 d) t = -56 c) 8, 4, 2 d) 6,-2, 1

e) f= 4 f) m = 2

EQUAES DO 1 GRAU Introduo EQUAO toda sentena matemtica expressa por uma igualdade ( = ), onde os nmeros desconhecidos so representados por Letras ( incgnitas). Exemplos: 2x 3 = 15 3x 4y = 6 Membros e Termos Equao na incgnita X. Equao nas incgnitas X e Y

Numa equao, a expresso situada esquerda do sinal = chamada de 1 membro da equao, e a expresso situada direita do sinal = chamada de 2 membro da equao. Exemplo: 2x + 10 1 membro = 3x 5 2 membro

Cada uma das parcelas que compem um membro de uma equao chamada de Termo da Equao. Exemplo: 5x 1 = 2x + 8 Termos Resoluo de uma equao do 1 grau - Passo a Passo

1 Caso) Resolver a equao 4x 3 = 2 (2x + 1) 5 soluo 4x 3 = 2 (2x + 1) 5 4x 3 = 4x + 2 5 4x 3 = 4x 3 Prop. Distributiva

55

PROF. WELLINGTON BRITO

4x 4x = 3 + 3 0 = 0 A igualdade se faz verdadeira, a equao chamada Indeterminada, e seu conjunto- soluo ser o Universo. (S = U)

2 Caso) Resolver a equao Soluo x + x = 5 + 5x 3 2 6 2x + 3x = 30 + 5x 6 6

x + x = 5 + 5x 3 2 6

Reduzimos as fraes ao mesmo denominador comum(m.m.c)

2x + 3x = 30 + 5x 5x 5x = 30 0 = 30 A igualdade no se faz verdadeira, a equao chamada Impossvel, e seu conjunto soluo ser Vazio. (S = ) 3 Caso) Resolver a equao 2x - 1 - x + 1 = 1 3 2 2 Soluo 2x - 1 - x + 1 = 1 3 2 2 2(2x 1) 3(x + 1) = 3 6 6 6 Reduzimos ao menor denominador comum 2 (2x 1) 3 ( x + 1) = 3 Prop. Distributiva 4x 2 3x 3 = 3 x = 3 + 5 x = 8 A equao chamada Determinada, e seu conjunto soluo a raiz encontrada. (S={8})

56

MATEMTICA PASSO A PASSO

EXERCCIO EQUAO DO 1 GRAU Resolva as equaes ( U = IR)


1) 3x + 5 = 20 2) 2x = - 6 3 15) x + x + 3x = 18 2 4 4 2 2 16) 3x = 5x 7

3) 5x 2 ( 3x + 2) = 7x 2 ( 4x + 3) 4) 5 ( x + 12) = x 5) 4 ( x 1 ) = 2 (x 4 )
6) 7) 2x = 5 ( x + 3 ) 5 (1 x ) 2x + 1 = - 3 ( 2 + x ) 2 5 3

17) x + x = 7 + 2x 3 2 18) 7x + 4 x = 3x 5 19) 4x 6 3x 8 = 2x 9 x 4 2 12 4 6

8) x + x = 15 3
8 9) x 4 = 8 12 5 4 x 5 8 6 10

20) 4x 5x + 18 = 4x + 1 5 2 4 3 4 3 2 4 9 6 3 2 21) 3x + 1 2x = 10 + x 1 22) 3x 2 4 x = 2x 7x 2 23) x + 2 x 3 = x 2 x 1

10) 3x 7 + x 1 = 2x 3 11) 2 + 2( x 3 ) = x x 3 12) 2 ( 5 + 3x ) = 5 ( x + 3 ) 5 13) 7 ( x 3 ) = 9 ( x + 1 ) - 38 14) x + x x = 14 2 3 4 6) S = {- 5 } 7) S = { 3 } 8) S = { 18 } Respostas:

1) S= { 5 } 2) S= { - 9 } 3) S=
{7}

11) S = { - 7/2} 12) S = {0} 13) S = { 4 }

16) S = { 2 } 21) S = { 14 } 17) S = { 14 } 22) S = { 2 } 18) S = { 3 } 23) S =

57

PROF. WELLINGTON BRITO

4) S = { -15}

9) S = { - 20 / 3} 14) S = { 24 } 15) S = { 8 }

19) S = { 4 } 20) S = { 20 }

5) S = { - 2 } 10) S = { 5 }

SISTEMA DE EQUAES DO 1 GRAU Introduo Equaes do tipo ax + by = c, isto , do primeiro grau com duas variveis, possuem uma infinidade de solues. Resolver um sistema de duas equaes achar os valores das variveis x e y, que satisfaam, ao mesmo tempo, cada uma das equaes. Logo, as equaes que constituem um sistema devero admitir a mesma soluo. Equaes desse tipo so chamadas de equaes simultneas. Na resoluo de um sistema de duas equaes simultneas do primeiro grau, empregamos os processos de Adio e Substituio, os quais passaremos a estud-los separadamente. Adio - Passo a Passo a) Multiplicam-se, ambos os membros de uma ou de cada uma das equaes, por nmeros, tais que, a incgnita que se deseja eliminar tenha, nas duas equaes o mesmo coeficiente, porm de sinais contrrios; b) Somam-se, membro a membro, as duas equaes, resultando, assim, uma nica equao com uma incgnita; c) Resolve-se esta equao, obtendo-se, assim, o valor de uma incgnita; d) Substitui-se o valor dessa incgnita em qualquer uma das equaes obtendo-se, assim, o valor da outra incgnita. Exemplos: 01) Resolver o sistema x + 2y = 11 x y= 5 Soluo: Como a varivel y j possui sinais contrrios, basta multiplicarmos a segunda equao por 2, no que resulta 58

MATEMTICA PASSO A PASSO

x + 2y = 11 2x 2y = 10 Somando, membro a membro, as duas equaes, vem: 3x = 21, logo: x = 7. Substituindo o valor de x na primeira equao, resulta: 7 + 2y = 11, que resolvida dar: Y = 2. Logo, S = { (7,2) } que o conjunto verdade da equao. 02) Resolver o sistema : 2x + 3y = 8 5x 2y = 1 Soluo: Multiplicando-se a primeira equao por 2 e a segunda por 3, temos: 4x + 6y = 16 15x 6y = 3

Somando, membro a membro, teremos: 19x = 19, onde x =1. Substituindo na primeira equao, o valor de x, vem: 2 + 3y = 8, que resolvida dar: y = 2. Ento, o conjunto soluo ser: S = { (1,2) }. Substituio - Passo a Passo a) Resolve-se uma das equaes, em relao incgnita que se deseja eliminar; b) Substitui-se, na outra equao, a incgnita pelo seu valor obtido na primeira; c) Resolve-se a equao resultante dessa substituio; encontrando-se, dessa forma, o valor dessa incgnita; d) Substitui-se o valor dessa incgnita em qualquer uma das equaes do sistema obtendo-se, assim, o valor da outra incgnita e, em conseqncia, a soluo do sistema. Exemplos: 1) Resolver o sistema Soluo: Resolvendo a primeira equao, em relao a x, temos: x = 1 2y. 59 x+ 2y = 1 2x y = 7

PROF. WELLINGTON BRITO

Substituindo na segunda equao, o valor de x, isto , 1 2y, vem: 2x y = 7. 2 (1 2y) y = 7 que resolvida, dar: y = 1. Substituindo o valor de y em x = 1 2y, temos: x = 3. Logo: S = { (3, -1)} que o conjunto verdade da equao.

x + y = 10 2) Resolver o sistema: xy=2 Soluo: Tirando o valor da varivel x na primeira equao, temos: x = 10 y. Substituindo, na segunda equao, o valor de x, vem: 10 y y = 2, que resolvida, resulta: y = 4. Substituindo o valor de y em qualquer uma das equaes do sistema ou na expresso x = 10 y, encontraremos o valor da varivel x que ser: x = 6. Ento, o conjunto soluo ser: S = { ( 6, 4) }. EXERCCIO EQUAES DO 1 GRAU 1. Resolver os sistemas abaixo, pelo mtodo da ADIO. a) b) c) x + 2y = 3 3x + y = 4 x + 3y = 4 2x y = 6 2x + 5y = 17 3x 2y = 16 a) x + y = 11 xy=1 b) f) e) d) 2x + 3y = 7 4x + y = 9 x+y=5 xy=1 x + 2y = 7 x 2y = 3 x + y = 46 x y = 14 c) x + y = 3 xy=1

2. Resolver os sistemas abaixo, pelo mtodo da SUBSTITUIO.

60

MATEMTICA PASSO A PASSO

d) 2x + y = 12 y = 2x g) 2x + y = 11 2x 3y = 1

e)

x + 2y = 7 x 2y = 3

f)

2x + y = 4 x y = -1

h)

3x 7y = 13 4x 5y = 13

i)

2x + 5y = 17 3x 2y = 16

3. Resolver os sistemas abaixo a) 2x + y = 13 x -y=8 c) x + 2y = 1 2x y = 7 e) x y=2 3 2 x y=3 2 3 g) x+y=7 3x y = 5 j) i) 2x + 4y = 16 5x y = 7 Respostas: 1.a) S = {(1,1)} e) S = {(3,2)} 2. a) S= {(6,5)} e) S= {(5,1)} b) S = {( 2, 2)} f) S = {(5,1)} b) S= {(30,16)} f) S = {(1,2)} c) S = {(2,1)} g) S = {(4,3)} 61 d) S= {(3,6)} h) S={(2, -1)} c) S = {(6, 1)} d) S = {( 2,1)} h) f) b) x + 2y = 9 x 2y = 1 d) 3 (x y ) + 5 ( y x ) = 18 2x + 3y = 37

2x + 3y = 23 5x - 3y = 5

2x + 3y = 5 7x 3y = 4 x+y = y+2 3 2 xy = x1 2 3

PROF. WELLINGTON BRITO

i) S= {(6,1)} 3. a) S= {(7,-1)} e) S= {(6,0)} i) S= {(2,3)} b) S= {(5,2)} f) S = {(4,5)} j) S= {(4,2)} c) S= {(3,-1)} g) S = {(3,4)} d) S={(2,11)} h) S={(1,1)}

PROBLEMAS COM NMEROS INTEIROS Introduo Antes de resolver um problema devemos obter uma forma de representao para o que ele prope. Vejamos: 1) Representar um nmero ou uma quantia, e a seguir, o seu dobro, seu triplo etc. Forma: x = o nmero 2x = o seu dobro 3x = o seu triplo

2) Representar duas quantidades, onde uma tem cinco unidades mais que a outra. Forma: x a quantia menor x + 5 = a quantia menor mais cinco unidades 2 forma x = a idade maior x 10 = a idade menor

3) Representar duas idades que diferem 10 anos. 1 forma x = a idade menor x + 10 = a idade maior

4) So dados trs nmeros: o 1 5 unidades maior que o 2 e este tem trs unidades menos que o 3. Forma: x : o segundo nmero x + 5 : o primeiro nmero x + 3 : o terceiro nmero

Questes Comentadas 1) Determinar dois nmeros cuja soma 40, sendo o maior o qudruplo do menor. x = o menor Donde: x = 8 62

MATEMTICA PASSO A PASSO

4x = o maior 4x + x = 40 5x = 40 x = 40 5 = 8

e 4x = 32

8 + 32 = 40

2) A diferena entre dois nmeros 18 e o maior o triplo do menor. Determin-los. x = o menor nmero 3x = o maior nmero 3x x = 2x a diferena entre o maior e o menor. 2x = 18 x = 18 2 = 9 x=9 3x = 27 3) Um pai diz a seus trs filhos: Conceio, Lus e Duda: ---- Vou repartir entre vocs a importncia de $ 90,00 de modo que Conceio, que a mais velha, receba $ 12,00 mais que Lus e este receba $ 6,00 mais que Duda que a mais nova. x = a quantia de Duda x + 6 = a quantia de Luis x + 6 + 12 = a quantia de Conceio Ou: x x+6 = 90 x + 18 3x + 24 = 90 Pela inversa da adio: 3x = 90 24 Duda: $ 22,00 Lus: $ 22,00 + $ 6,00 = $ 28,00 Conceio:$ 28,00+$12,00=$40,00 3x = 66 pela inversa da multiplicao x = 66 3 = 22

4) A soma das idades de um pai e um filho hoje 54 anos. H 6 anos a idade do pai era o quntuplo da idade do filho. Quais as idades de cada um hoje? H seis anos 54 2 X 6 = 54 12 = 42 anos x a idade do filho 5x a idade do pai 5x + x = 6x 6x = 42 anos 63 x = 42 anos 6 = 7 anos o filho, 7 anos o pai, 35 anos Hoje 7 + 6 = 13 anos o filho.

PROF. WELLINGTON BRITO

35 + 6 = 41 anos o pai

5) Cndida faz problemas. Ganha $ 0,10 por problema certo e paga multa de $ 0,07 por problema que erra. Fez 20 problemas e recebeu $ 1,32. Quantos problemas acertou e quantos errou? Vejamos: Neste problema deve-se raciocinar: Se Cndida acertasse todos os problemas ganharia: 20 problemas x $ 0,10 = $ 2,00. Entretanto recebeu apenas $ 1,32. Significa que deixou de ganhar a diferena, isto : $ 2,00 $ 1,32 = 0, 68. Entretanto, em cada problema errado, Cndida deixou de ganhar: $ 0,17. Pergunta-se: errado?
___

Por que deixou de ganhar $ 0,17 por problema

___ Claro, primeiro no ganhou $ 0,10 que seria o prmio do acerto e segundo pagou $ 0,07 de multa. Logo, so $ 0,17 de prejuzo por problema errado.

Como o prejuzo total foi de $ 0,68 e o prejuzo por problema foi de $ 0,17 basta efetuar a diviso: $ 0,68 $ 0,17 = 4 problemas. Isto , Cndida errou 4 problemas e, logicamente, acertou 16 problemas. Problemas certos: Prova: Problemas errados: $ 0,07 x 4 = $ 0,28 Recebeu $ 1,32 64 $ 0,10 x 16 = $ 1,60

MATEMTICA PASSO A PASSO

6) Cndida foi a Bahia e deixou $ 1,00 de bolo em cada igreja que visitou e, ao fim das visitas, sobraram $ 4,00 do dinheiro destinado s esmolas. Se tivesse deixado $ 1,50 em cada igreja, teria gasto $ 8,00 mais do que esperara gastar com os bolos. Quantas igrejas Cndida visitou e quanto pensara gastar com as esmolas? Vejamos: I) Deixando $ 1,00 em cada igreja sobram $ 4,00 Deixando $ 1,50 em cada igreja faltam $ 8,00

V-se assim, que um acrscimo de 0,50 em cada bolo provoca um acrscimo de $ 12,00 nas despesas de Cndida.(De fato, a sobra de $ 4,00 e mais os $ 8,00 que despenderia.) Logo as igrejas so: $ 12,00 $ 0,50 = 24 igrejas. II) Cndida destinara aos bolos: 24 X $ 1,00 = $ 24,00 distribudos + $ 4,00 $ 28,00 Prova: De fato, se desse $ 1,50 em cada igreja, gastaria $ 36,00, isto , $ 8,00 mais do que previra. 7) Um boiadeiro comprou 25 bois e 8 novilhos pela importncia de $ 1.820,00. Determinar o preo de cada animal, sabendose que um boi e um novilho juntos custam $ 100,00 Vejamos: Como um boi e um novilho juntos valem $ 100,00, ento os oito novilhos e os oito bois custaram $ 800, 00. Desse modo, a importncia restante, isto , $ 1.820,00 $ 800,00 = $ 1.020,00 foi necessria para adquirir os restantes 17 bois, ou seja:$ 1.020,00 17 = 60,00 Logo: 65

PROF. WELLINGTON BRITO

$ 60,00 o preo de cada boi; e $ 100,00 - $ 60,00 = $ 40,00 o preo de cada novilho. Prova: Um boi + um novilho = $ 60,00 + $ 40,00 = $ 100,00 8 novilhos + 25 bois = $ 320,00 + $ 1.500,00 = $ 1.820,00. 8) De duas cidades A e B, cuja distncia 315km, partem simultaneamente dois trens. O que parte de A se dirige em direo a B com a velocidade mdia de 60km por hora e o que parte de B se dirige para A com a velocidade mdia de 45km por hora. Pergunta-se: depois de quanto tempo se cruzaro e a que distncia de A? Vejamos: teremos: Representando-se esquematicamente o problema,
P
1 trem 315 km 2 trem

V1 = 60 km/h

V2 = 45 km/h

Como os trens se deslocam em sentidos contrrios, ao se cruzarem, a soma dos percursos realizados igual distncia entre A e B que de 315 km. Para realizar o mesmo percurso, no mesmo tempo, um nico trem deveria ter uma velocidade V igual soma entre as velocidades V1 e V2 . Isto . V = 60 km/h + 45 km/h = 105 km/h Ora, um trem com 105 km/h demoraria 3 horas para percorrer os 315 km. De fato: 315km 105 km/h = 3 horas. Da mesma forma, os dois trens que partem de A e B. Para isso, basta, ento dividir o percurso de 315 km pela soma de suas velocidade e, como acima, se encontrar 3 horas. 60km/h x 3 horas = 180 km; 45 km/h x 3 horas = 135km. P
180 km 315 km 135 km

66

MATEMTICA PASSO A PASSO

EXERCCIOS NMEROS INTEIROS 1) A soma de dois nmeros 52 e um deles o triplo do outro. Quais so os nmeros? 2) A diferena entre dois nmeros 45 e o maior deles igual ao sxtuplo do menor. Determin-los. 3) A minha idade o quadruplo da idade de meu filho e juntos temos 45 anos. Quais so as nossas idades? 4) A soma de dois nmeros 43 e um deles excede o outro de 5 unidades. Quais so os nmeros? 5) A diferena entre dois nmeros 7 e a soma deles 29. Determin-los. 6) Quando Joo nasceu, Cndida tinha 5 anos. A soma das idades hoje 31 anos. Quais as idades? 7) O produto de dois nmeros 6.800 e um deles 170. Determinar o outro . 8) Pensei em um nmero; multipliquei-o por 3; somei 12 ao resultado; dividi esse resultado por 3 e obtive 19. Qual o nmero pensado? 9) Um recipiente alimentado por duas torneiras: a primeira despeja 64 litros de gua por minuto e a segunda despeja 48 litros de gua por minuto. Pergunta-se: ao fim de quantos minutos o recipiente ficar cheio, sabendo que sua capacidade de 1.904 litros? 10) O terreno de meu vizinho 80m 2 maior que o meu. A fim de ficarmos com terrenos iguais, ele vai vender-me a parte necessria razo de $ 180,00 o metro quadrado. Quanto deverei pagar? 11) Se eu tivesse $ 14.640,00 mais do que tenho, poderia comprar um terreno que tem 320m2 cujo valor $ 120,00 o metro quadrado. Quanto eu possuo? 12) Lus diz a Marcos: "Se eu lhe der 14 figurinhas das que eu tenho, ento voc ficar com tantas figurinhas quanto "eu". 67

PROF. WELLINGTON BRITO

Sabendo que juntos possuem 120 figurinhas, pergunta-se: quantas tm cada um? 13) A soma das idades de Cristina, Marcelo e Frederico 20 anos. Cristina nasceu 6 anos antes que Marcelo e este 4 anos mais velho que Frederico. Quais so as idades dessas crianas? 14) Quando Fbio nasceu, Clara tinha 4 anos e Lourdes tinha 6 anos. Hoje, a soma das idades dos trs 22 anos. Determin-las. 15) Um pai deseja distribuir a quantia de $ 115,00 entre seus trs filhos. Quer dar $ 10,00 mais a Sidnio do que a Roberto e $15,00 mais a Roberto do que a Francisco. Quanto deve receber cada um? 16) A soma das idades de um pai e de um filho hoje 72 anos. H 12 anos passados, a idade do pai era 7 vezes a idade do filho. Quais so as idades hoje? 17) A soma das idades de um pai e um filho hoje 30 anos. Daqui a 12 anos a idade do pai ser o dobro da idade do filho. Quais as idades hoje? 18) Um pai diz a seu filho: "A soma de nossas idades hoje 36 anos. Entretanto, h trs anos passados, minha idade era o qudruplo da sua". Quais so essas idades? 19) Um av tem 74 anos e seus 4 netos, 5, 7, 11 e 12 anos. No fim de quantos anos ser a idade do av igual soma das idades dos netos? 20) A soma de quatro nmeros inteiros consecutivos 86. Calcul-los. 21) Aumentando-se um certo nmero de 126 unidades, obtmse o quadruplo do nmero. Calcul-lo. 22) Num quintal existem perus e coelhos, ao todo 62 cabeas e 148 ps. Quantos so os perus e quantos so os coelhos? 23) Um aluno ganha $1,50 por problema que acerta e paga, a ttulo de multa, $ 0,90 por problema errado. Faz 20 problemas e recebe $ 20,40. Quantos acertou e quantos errou? 24) Uma pessoa d esmolas s igrejas que visita. Tendo deixado $ 0,20 em cada igreja, ainda lhe sobraram $ 1,80. 68

MATEMTICA PASSO A PASSO

Entretanto, se desse $ 0,30 de esmola a cada igreja, ter-lheiam sobrado apenas $ 0,70. Quantas foram as igrejas visitadas e quanto levava essa pessoa no bolso? 25) Dei trs laranjas a cada menino e fiquei com vinte. Se tivesse dado 5 a cada menino, teria ficado com 8. Quantos meninos eram? 26) Uma pessoa querendo distribuir laranjas entre vrios meninos, calculou que poderia dar a cada um, 11 laranjas e ainda lhe restariam 4 laranjas. Mas tendo um menino recusado a sua parte, cada um dos outros recebeu 14 laranjas, sobrando ainda 3 laranjas. Quantos meninos havia e quantas laranjas a pessoa possua? 27) Joo diz a augusto: "Eu tenho $ 3,15 no bolso com igual nmero de notas de $0,05, $ 0,10 e $ 0,20. Quantas notas eu tenho de cada espcie? 28) Augusto diz a Joo: "Eu tenho $1,80 em notas de $0,05, $0,10 e $0,20. As importncias em dinheiro que possuo de cada espcie so iguais. Quantas notas eu tenho de cada espcie? 29) Uma pessoa compra 12 frangos e 20 perus pela importncia de $ 124,00. Determinar o preo de cada ave, sabendo que um frango e um peru custam juntos $ 7,00. 30) Um criador compra 40 burros e 52 cavalos pagando $ 31.600,00 pelo lote. Determinar o preo de cada animal, sabendo que um burro e um cavalo custam juntos $ 700,00. 31) So dados trs nmeros: a soma dos dois primeiros 20, a soma dos dois ltimos 15 e a soma do primeiro com o ltimo 19. Quais so esses nmeros? 32) Um ciclista parte da cidade A em direo a B, ao mesmo tempo que o outro parte de B em direo a A, cuja distncia 120 km. O primeiro desenvolve uma velocidade de 24 km por hora e o segundo 16 km por hora. Pergunta-se: a) Ao fim de quanto tempo se encontraram? b) A que distncia da cidade A se d o encontro? 33) Durante uma viagem, uma caravana pousou num motel. Os homens pagaram o dobro que as senhoras e estas pagaram o triplo que as crianas. Sabendo-se que a despesa foi de $ 1.950,00 e que existiam 20 homens, 15 senhoras e 30 crianas, pergunta-se: quanto pagou cada um? 69

PROF. WELLINGTON BRITO

34) De uma estao parte um trem que desenvolve 50km por hora. Aps 3 horas, parte outro trem no mesmo sentido, que alcana o primeiro quando decorreram 5 horas da partida do segundo. Pergunta-se: qual a velocidade mdia do segundo trem? Respostas 1) 13 e 39 2) 9 e 54 3) Pai: 36anos Filho: 9 anos 4) 19 e 24 5) 11 e 18 6) Cndida: 18 anos Joo:13 anos 7) 40 8) 15 9) 17 minutos 10) R$ 7.200,00 11) R$ 23.700,00 12) Lus: 74 Marcos: 46 13) Cristina:12 anos Marcelo: 6 anos Frederico: 2 anos 14) Lourdes: 10 anos Clara: 8 anos Fbio:4 anos 15) Sidnio: R$ 50,00 Roberto:R$ 40,00 Francisco: R$ 25,00 16) Pai: 54 anos Filho: 18 anos 17) Pai: 24 anos Filho 6 anos 18) Pai: 27 anos Filho 9 anos 19) 13 anos 20) 20,21,22,23 70 23) Acertou:16 problemas Errou: 4 problema 24 11 Igrejas e R$ 4,00 25) 6 meninos 26) 5 meninos-59 laranjas 27) 9 notas 28) 12 de $ 0,05 6 de $ 0,10 3 de $ 0,20 29) Frango: R$ 2,00 Peru: R$ 5,00 30) Burro: R$ 400,00 Cavalo: R$ 300,00 31) 1 nmero: 12 2 nmero: 8 3 nmero: 7 32) a) 3 horas b) 72 km 33) Crianas:$ 10,00 Senhoras:$ 30,00 Homens: $ 60,00 34) 80 km por hora

MATEMTICA PASSO A PASSO

21) 42 22) Perus: 50 Coelhos 12

QUESTES DE CONCURSOS E VESTIBULARES 01.(CMF) Um pai tem 36 anos e seus trs filhos 3, 5 e 8 anos. No fim de quanto tempo a idade do pai ser igual a soma das idades dos filho? a) 10 anos b) 12 anos c) 15 anos d) 18 anos e) 20 anos 02.(BB) Hoje eu tenho a idade que meu amigo Paulo tinha quando eu nasci. Daqui a 15 anos terei 3/5 da idade de Paulo. Qual a idade atual de Paulo? a) 45 anos b) 50 anos c) 55 anos d) 60 anos e) 65 anos 03.(CEF) H 8 anos, a idade de "A" era o triplo da de "B", e daqui a 4 anos a idade de "B" ser os 5/9 da de "A". Achar a razo entre as idades de A e B. a) 1/2 b) 2 c) 3/2 d) 2/3 e) 3 04.(UFC) Quando Jos nasceu, Bruno tinha 4 anos de idade. Decorridos 17 anos, qual diferena, em anos, entre as idades de Bruno e Jos? a) 13 b) 4 c) 21 d) 5 e) 17 05.(BNB) O sistema de equao abaixo: 2x + y + z + w = 1 x + 2y + z + w = 2 x + y + 2z + w = 3 x + y + z + 2w = 4 Possui uma nica soluo x, y, z, w. Pode-se afirmar que a soma S= x + y + z + w igual a: a) 1 b) 2 c) 3 d) 4 e)5 06). (BB) Em um ptio existem automveis e bicicletas. O nmero total de rodas 130 e o nmero de bicicletas o triplo do nmero de automveis. Assim, o nmero total de veculos igual a: 71

PROF. WELLINGTON BRITO

a) 39

b)42

c) 49

d) 52

e)59

07).(TJ) Em um terreiro havia um certo nmero de bpedes e de quadrpedes, num total de 900 patas. As patas dos bpedes so a metade das dos quadrpedes. O nmero total de animais que havia no terreiro : a) 150 b) 300 c) 350 d) 450 e)600 08.(TTN) Dois professores "A" e "B", do aulas particulares e sabe-se que "A" cobra R$ 2,00 a mais que "B" por hora de trabalho. Se, por 30 horas de aula, "A" receber R$ 210,00 a mais que "B" receberia ministrando 20 horas aula. A soma das quantias que cada um cobra por hora de trabalho : a) R$ 32,00 b) R$ 35,00 c) R$ 40,00 d) R$42,00 e) R$ 45,00 09.(CEF) O Sr. J.R.N. foi contar seu patrimnio e encontrou apenas moedas de 1 centavo, 5 centavos, 10 centavos, 25 centavos, e 50 centavos, todas em quantidades iguais, totalizando RF$ 15,47. Qual a importncia que o desafortunado tem em moedas de 25 centavos? a) R$ 3,00 b) R$ 3,50 c) R$ 3,75 d) R$ 4,00 e) R$ 4,25 10.(UFC) Uma dona de casa programou uma recepo no aniversrio de seu marido e solicitou a um buffet que fizesse 7 salgadinhos de um certo tipo para cada convidado. No dia da recepo, ao receber os salgadinhos, notou que havia 2 a mais do que o encomendado. Por outro lado, compareceram recepo 3 convidados a mais que o esperado. A dona de casa resolveu o imprevisto, distribuindo exatamente 6 salgadinhos para cada convidado presente. Com base nessas informaes, assinale a opo que contm o nmero de salgadinhos preparados pelo buffet. a) 108 b)114 c)120 d)126 e) 132 Respostas 01.A 02. D 03. B 04. B 05.B 06.D 72 07.B 08.A 09.E 10.B

MATEMTICA PASSO A PASSO

O CONJUNTO DOS NMEROS RACIONAIS A Idia de Nmero Racional

2 4

( 2,4 )

2 3

( 2,3 )

Para se representar numericamente uma ou mais partes de um inteiro, so necessrios dois nmeros naturais: a) O primeiro, indicando quantas partes tomamos do inteiro; b) O segundo indicando em quantas partes, de igual valor, o inteiro foi dividido; Nesses novos smbolos
3 2

ou
5

etc., o primeiro elemento chama-se

Numerador e o segundo elemento denominador dos nmeros fracionrios ou fraes. De modo geral, chamam-se de Termos da frao ao conjunto do numerador e denominador. Fraes Equivalentes

fcil ver que um mesmo nmero fracionrio pode ser representado por vrios smbolos ou vrios numerais. Vejamos: I) II)
2 4 2 6 1 2 1 3 22 42 22 62 1 2 1 3

III) IV)

3 4 1 1

12 16 2 2

3x 4 4x 4 1x 2 1x 2

12 16 2 2

73

PROF. WELLINGTON BRITO

Dizemos ento: 2 4 2 6 3 4 1 1 e 1 so fraes equivalentes ou 2 ~ 1 2 4 2 e 1 so fraes equivalentes ou 2 ~ 1 3 6 3 e 12 so fraes equivalentes ou 3 ~ 12 16 4 16 e 2 so fraes equivalentes ou 1 ~ 2 2 1 2

Propriedade Fundamental Multiplicando-se ou dividindo-se os termos de uma frao por um nmero natural 0, obtm-se uma frao equivalente frao dada. Classificao

Frao Imprpria aquela cujo numerador maior que o denominador. Frao aparente aquela cujo numerador mltiplo do denominador. As fraes restantes se dizem prprias. Exemplos: Os nmeros fracionrios Imprprias. Os nmeros fracionrios 4 3 e 7 , chamam-se 2 Fraes

4 e 12 que equivalem, 2 4 respectivamente, a 2 e 3, so fraes aparentes.

74

MATEMTICA PASSO A PASSO

EXERCCIO DE FIXAO Colocar V ou F conforme cada uma das seguintes sentenas matemticas sejam verdadeiras ou falsas:
a) 2 e 4 so fraes equivalentes ( 3 6 b) 3 5 6 5 ( ). h) 2 < 1 5 5 ( )

). I) Toda frao imprpria maior que toda frao prpria. ( ) ). J) Qualquer frao prpria menor que o inteiro. ( ). ). ). ). )

c)Toda frao aparente imprpria ( d)Toda frao imprpria aparente ( e) 7 > 1 2 f) 2 < 1 3. g) 3 > 2 2 2 ( ( (

Resposts: a) V

b) F c) F d) F e) V f) V g) V h) F i)V

j)V

Simplificao de Fraes.

Obter uma frao equivalente a 36 de modo que os termos sejam primos entre si. 48 Basta aplicar a propriedade fundamental e dividir sucessivamente os termos da frao por um fator comum. Exemplo: 36 2 18 2 93 3 36 3 48 2 24 2 12 3 4 48 4 Tambm se pode dividir os termos de 36 pelo seu mximo divisor comum: 48 m.d.c. (36,48) = 12 36 12 3 36 3 48 12 4 48 4 Assim conclumos: Simplificar uma frao a significa transform-la numa b frao c de modo que c e d sejam primos entre si. d 75

PROF. WELLINGTON BRITO

Conjunto dos Nmeros Racionais Q Vimos que, de um modo geral, todo nmero que pode ser representado na Forma de Frao um Nmero Racional. Portanto, temos as seguintes propriedades:
I - Todo nmero natural racional II Todo nmero inteiro racional

De fato: 0 =

=.......

De fato: -1= -1 = 1 = 1 -1 2 =
-2

-2

= ...... 2 = ...... 2

1 = 1 = 2 = 3 =....... 1 2 3 =....... 1 2 3 Assim, podemos concluir que: IN Q Z Q ou, ou, Q IN Q Z 2=


2

= 2 = 1 -1

-4

Exerccio de Fixao Colocar V ou F em cada uma das sentenas matemticas seguintes, conforme elas sejam verdadeiras ou falsas: a) se a IN b) se a Q c) IN = Q d) Q IN e) 2/3 Q aQ ( ) f) 3/5 Q g) Q 0 h) IN 0 i) 0 Q j) 0 IN ( ) ( ) ( ) ( ) ( ) a IN ( ) ( ) ( ) ( )

Respostas: a)V b)F c)F d)F e)V f)F g)F h)F i)V j) V Fraes Homogneas e Fraes Heterogneas Duas ou mais fraes que tm denominadores diferentes, se dizem heterogneas. Exemplos: 3, 1, 1 so heterogneas. 5 4 2

76

MATEMTICA PASSO A PASSO

Quando duas ou mais fraes tm denominadores iguais, elas se dizem homogneas. Exemplos: 3, 1, 5 so homogneas. 7 7 7 Portanto, reduzir fraes ao mesmo denominador, significa tornlas homogneas. Sejam as fraes: 1 , 2 , 3 e 5. 2 3 4 6 a) Determina-se o m.m.c. dos denominadores: m.m.c. (2,3,4,6) = 12 b) Divide-se o m.m.c. achado (12) pelos denominadores das fraes. O quociente obtido em cada caso deve ser multiplicado pelos termos da frao. Pela propriedade fundamental obter-se- uma frao equivalente primeira. Assim, teremos: 1, 2, 3, 5 2 3 4 6 6 , 8 , 9 , 10 12 12 12 12 Comparao de Fraes Passo a Passo Comparar duas ou mais fraes significa determinar uma relao de igualdade ou desigualdade entre elas. Temos 3 casos a examinar. 1) As fraes so homogneas. Comparar: 2, 3 e 5 7 7 7

Quando vrias fraes so homogneas, a maior delas a que tem maior numerador. 2 < 3 < 5 7 7 7 2) As fraes tm numeradores iguais. 3, 3 e 3 5 4 7 Quando vrias fraes tm o mesmo numerador, a maior delas a que tem menor denominador. 3 < 3 < 3 7 5 4 77 Comparar:

PROF. WELLINGTON BRITO

3) As fraes so heterogneas. 2, 3, 1, 5 3 4 2 6 Neste caso deve-se reduzir as fraes a um denominador comum. Mais simplesmente ao menor denominador comum. 8 , 9 , 6 , 10 12 12 12 12 Como as fraes resultantes so homogneas, recamos no primeiro caso, ou seja: 6 < 8 < 9 < 10 12 12 12 12 Ou, ordenando-se crescentemente: 1 < 2 < 3 < 5 2 3 4 6 Questes Comentadas 1) Obter trs fraes equivalentes a 3/5. Basta tomar os termos da frao 3/5 e multiplic-los por um mesmo nmero diferente de zero. 3 x 2 = 6 ; 3 x 3 = 9 ; 3 x 10 = 30 5x2 10 5 x 3 15 5 x 10 50 2) Obter uma frao equivalente a 3/4 cujo denominador seja 60. Como 60 = 4 x 15, isto , mltiplo de 4, o problema possvel. Basta multiplicar numerador pelo mesmo fator 15. 3 = 45 4 60 3.) Obter duas fraes equivalentes a 12/18 cujos denominadores sejam 30 e 42. Reduz-se 12/18 sua forma mais simples e equivalente: 12 = 2 ; 18 3 2 = x 3 30 Comparar :

78

MATEMTICA PASSO A PASSO

Para determinar o valor desconhecido (x) do numerador da nova frao faz-se: 30 : 3 = 10; 10 x 2 = 20. Logo: Analogamente: 2 = 20 3 30 2 = y 3 42 Donde: 2 = 28 3 42

Para se determinar o o numerador y, faz-se: 42 : 3 = 14; 14 x 2 = 28.

4.) Escrever uma frao equivalente a 5/6 cuja soma dos termos seja 88. Como a soma dos termos de 5/6 11, para se determinar o fator pelo qual se multiplicaro os termos de 5/6, faz-se: 88 11 = 8 8 o fator procurado. Logo: 5 x 8 = 6x8 =

40 48

De fato: 40 + 48 = 88 EXERCCIO NMEROS RACIONAIS 1) Escrever fraes equivalentes a:


a) 3/4, cujo denominador seja 56 b) 8/16, cujo denominador seja 36 c) 15/45, cujo denominador seja 27 d) 2/5, cujo numerador seja 64 e) 255/315, cujo denominador seja 21 f) 255/315, cujo numerador seja 51

2) Simplificar as seguintes fraes: a) 180 240 b) c) 625 2500 121 2057 d) 343 490 e) 1920 2520 f) 1728 2880 g) 729 1728 h) 289 2057 i) 1350 2800

79

PROF. WELLINGTON BRITO

3) Reduzir ao menor denominador comum os seguintes grupos de fraes: a) 1 , 1 , 1 , 1 2 3 4 6 b) 3 , 4 , 5 , 3 4 5 6 8 c) 3 , 1 , 5 , 8 7 12 42 21 d) 4 , 5 , 11 , 7 15 12 18 30

e) 11 , 17 , 121, 19 120 540 600 270 f) 3 , 2 , 121 49 5 , 77 3 7

4) Escrever em ordem decrescente de seus valores cada um dos seguintes grupos de fraes: a) 3 , 5 , 1 , 7 d) 4 , 3 , 5 , 6 8 8 8 8 3 4 6 5 b) 8 , 8 , 8 , 8 3 5 11 7 c) 11 , 11 , 11 , 11 2 5 13 8 Respostas
1) a) 42/56 b) 18/36 2) a) 3/4 g) 27/64 b) 1/4 c) 9/27 c) 1/17 d) 64/160 d) 7/10 e) 17/21 e) 16/21 f) 51/63 f) 3/5

e) 5 , 8 , 9 , 11 18 12 24 30

h) 17/121 i) 27/56 c) 36 , 7 , 10 , 32 84 84 84 84 e) 495, 170 ,1089 , 380 5400 5400 5400 5400

3) a) 6 , 4, 3 , 2 12 12 12 12

b) 90 , 96 , 100 , 45 d) 48 , 75, 110 , 42 f) 147 , 242 , 385 , 2541 120 120 120 120 180 180 180 180 5929 5929 5929 5929 4) a) 7 > 5 > 3 > 1 8 8 8 8 b) 8 > 8 > 8 > 8 3 5 7 11 c) 11 > 11 > 11 > 11 2 5 8 13 d) 4 > 6 > 5 > 3 3 5 6 4 e) 8 > 9 > 11 > 5 12 24 30 18

80

MATEMTICA PASSO A PASSO

Operaes Com Nmeros Racionais Adio e Subtrao Passo a Passo Distinguiremos trs casos: 1) As fraes so homogneas. 3 + 2 = 3 + 2 = 5 ou: 3 - 2 = 3-2 = 1 7 7 7 7 7 7 7 7 Conclui-se: Se as fraes so homogneas somam-se os numeradores e d-se ao resultado o denominador comum. Se as fraes so homogneas subtraem-se os numeradores e d-se ao resultado o denominador comum. 2) As fraes so heterogneas. Seja: seja: 2 + 1 2 - 1 3 4 3 4 Reduzem-se as fraes ao menor denominador comum. m.m.c. (3,4) = 12 8 + 3 = 8 + 3 = 11 12 12 12 12 8 - 3 = 83 = 5 12 12 12 12

E procede-se como no primeiro caso. 3) Inteiro e Frao. Seja: 2+ Seja:


1 2- 1 4 4 Basta tomar o nmero 2 e escrev-lo sob a forma racional 2 teremos: 1 2 + 1 2 - 1 1 4 1 4 Reduzindo-se ao menor denominador comum: 8+1 = 9 81 = 7 4 4 4 4 Nota-se em ambos os casos que se fez: 2 + 1 = 2x4+1 = 9 2 - 1 = 2x4-1 = 7 4 4 4 4 4 4

que uma forma prtica de se efetuar tais clculos.

81

PROF. WELLINGTON BRITO

Nmero Misto O nmero expresso por 2 + 1 , (inteiro e frao) chamado 4 misto e costuma ser representado por 2 1 , que se l: dois inteiros e 4 um quarto. Assim: 2 1 = 2 x 4 + 1 = 9
4 4 4

a forma habitual de se transformar o nmero misto em frao imprpria. Extrao de Inteiros de uma Frao Imprpria regra: Para se extrair os inteiros de uma frao imprpria, dividese o numerador pelo denominador da mesma. O quociente indicar a parte inteira do nmero misto e o resto ser o numerador da parte fracionria que conserva o denominador primitivo. Extrair os inteiros: 2 2
2 3 5 5

Exemplo: Transformar em frao imprpria: 1) 3 1 = 3 x 5 + 1 = 16


5 5 5

1) 12 5

12 5 2 17 2

2) 4

1 3

4x3+1 3

13 3

2)

17

5
3

Questes Comentadas
Efetuar as seguintes operaes:

1) 3

+ 1

4 3 2 Reduzindo-se os nmeros mistos a fraes vem: 13 + 4 + 1 4 3 2 Reduzindo-se ao menor denominador comum : 39 + 16 + 6 = 61 = 5 1 12 12 12 12 12

82

MATEMTICA PASSO A PASSO

2)

1 3

(1

1 8 13 3 13 3 13 3

1 2

) _ (2 (
8

1 4

_2

1 5

)
4

= 11
5

63 40

9 + 1 2

)( 9
20

)
=

( 9 + 4 ) ( 45 - 44 )
8 13 8

20

= 13
3

65 40

2 =
2 91 120

13 3

520 120

189 120

331 120

A Multiplicao e Diviso de Nmeros Fracionrios Multiplicao - Passo a Passo O produto de duas fraes uma frao, onde o numerador o produto dos numeradores e o denominador o produto dos denominadores das fraes dadas.

Exemplos: 1) Efetuar os produtos: 3 x 1x 5 = 3 x 1 x 5 5 4 6 5 x 4 x 6 Antes de efetuar a multiplicao convm realizar a simplificao pelo cancelamento. No produto anterior ficaria: 1 1 3 x 1 x 5 = 1 5 4 6 8 1 2 2) Efetuar: 1 x1 1 x 1 3
4 13 6

Em primeiro lugar reduzem-se os nmeros mistos a fraes imprprias, resultando: 13 x 14 x 1 4 13 6 A simplificao dar: 1 x 7 x 1 = 7 2 1 6 12

83

PROF. WELLINGTON BRITO

Diviso - Passo a Passo 2 3 = 2 x 5 3 5 3 3 Do que se conclui:

Note a equivalncia:

Para se dividir uma primeira frao por uma segunda, Multiplica-se a primeira pela frao inversa da segunda. Exemplos: 1 2)2 3) Frao de Frao Seja calcular: 2 de 1 3 2 pela operao
1 3 5 1 2

3 6 10 5 31 =
2 7 3

= 3 x 5 = 1 10 6 4
2 1 = 7 2

= 10

7 3

2 7

=
3

5 x

Na prtica substitui-se a preposio "de" multiplicao. Assim: 1) 2 de 1 = 2 x 1 = 2 = 1 3 2 3 2 6 3 2) 1 de 5 4 6 = 1 x 4 5 = 5 6 24

3) 3 de 2 = 3 x 2 = 6 = 3 8 8 1 8 4

84

MATEMTICA PASSO A PASSO

EXERCCIO OPERAES COM FRAES I) 1) 2) 3) 4)

) ( )( )( ( ) ) (
4 + 1 7

Resolver as seguintes expresses fracionrias 2


3 +2

21

7+

+3

1 3

3 14

) (
3

2 +

11 3

63

7 12

1 3

1 4

5 6

+ 5

3 8 19 8 3 8
2 5

)
+ 5

77 12
2 1

5) 6) 7) 8) 9)

17 5

2 15

3 5

( ) (
3 3 8 + 1+

+ 1+ 2

10) 11) 12)

( ) ( ) ( ) ( ) ( ) ( ) ( ) ) ( ) ( )( )( ( )( ) (
8
2 +

11 5
3 11 17 2

33

11

11

+4

89

1+

19

13

26

13

13

( ) ( ) ) ( )
3 + 3+ 7 + 70

7+

1+ 33

+ 3

11

15

26

+1

2 7

27

16

42

11

X 4

15

11

+ 2

20 9

4 9

12

15

21

31

2 3

12

20

10

2 +

28

94

124 77

55 62

4 7

17

35

47

21

)
X

1 90

4 X

15

14 111

70

15

12

13)

2 11

143 6

40 21

5 7

X 7 4

85

14) 15) 16) 17) 18) 19)

( (

13 84

56

)
25 9

205 48

( )
5 2 3 + 1

PROF. WELLINGTON BRITO


: 65 12 + 2 3 + 1 21

20) 21) 22) 23)

24)

)( ) ( )( ) ( ) ( ) ( ) ( )( )( ) ( ) ) ( ) ( ) ( ) ( ) ( )( ) ( )( ) ( ( ) ( ) ( )( ) ( ( )) ) ( ) ( )
14 : X

5 : 21 6 5
: 5

3 10

5 +

126 35
X

18 43
3

14

65

14
X

12

29

10

21

23

35

21

11

10

25 14
1 4

11

55

7 3

15 4

1 + 7

+4

13

27

1+

25

14

25

40

2 :

11

1 + X 3 33 X

24

55

88

4 3

5 2

21 16

14 30

5 8

847 96 8 9

5 21 1 3
2

7 9

+ 1

10 9

3 + 72
3

5 X

3 35

1 49

X 9

2 +

1 3

1 2

1 4

: 11

5 7 4 6
x

14

11 5

7 3

3 2

: 5 +

1 7

1 + 1 5 2

3 5

: 3

Respostas 1) 1 5 7) 4 1 7 2 2) 2 2 8) 1 3 3) 1 8 9) 21 9 2 4) Zero 10) 1/6 5) 4 4 11) 1 5 10 6) 3 1 12) 2 3 9

13) 9 2 14) 11 7 15) 39 86 16) 1 17) 2 1 11 18) 7 8

19) 1 35 20) 2 10 11 21) 1 1 14 22) 1 23) 2 9 24) 1 3

PROBLEMAS COM NMEROS RACIONAIS

86

MATEMTICA PASSO A PASSO

Questes Comentadas 1) Se 2/3 de um muro custaram $ 48,00, pergunta-se qual o preo do muro todo? Raciocinando: Nestes problemas deve-se sempre encontrar em primeiro lugar a unidade em relao qual o problema se identifica. Aqui a unidade 1/3. Se: 2 $ 48,00 ento: 1 $ 48, 00 : 2 $ 24,00. 3 3 Sabendo-se 1/3, saber-se- o custo do muro todo que representado por 3/3. Ento: 3 3 x $ 24,00 = $ 72,00 3

2) Um auto percorre 2/9 de uma estrada e depois percorre mais


1/3 da mesma e desse comprimento da estrada? Raciocinando: modo rodou 300 km. Qual o

Devem ser somados os percursos que o auto realizou, ou seja: 2 + 1 = 2 + 3 = 5 9 3 9 9 9 Percorreu o carro 5/9 e desse modo 5/9 300 km. Donde: 1/9 E: 9/9 300km : 5 = 60 km. 9 x 60 km = 540 km. Logo: a entrada toda tem 540km.

3) Um ciclista percorre 1/4 do percurso entre duas cidades e depois mais 3/8 dessa estrada e ainda faltam 7 200 m para percorrer. Qual a distncia entre as cidades? Raciocnio: Somamos inicialmente os percursos realizados: 1 + 3 = 2 + 3 = 5 4 8 8 8 8 Logo, faltam 3/8 para percorrer. Ainda faltam 7200 metros a percorrer. Teremos ento: 3/8 7200 m 1/8 7200 3 = 2400m E: 8/8 8 x 2400m = 19 200m. Portanto: A estrada toda tem 19 200 metros.

87

PROF. WELLINGTON BRITO

4) Uma senhora vai feira e gasta, em frutas, 2/9 do que tem na bolsa. Gasta depois 3/7 do resto em verduras e ainda lhe sobram $ 8,00. Quanto levava ao sair de casa? Podemos usar o seguinte raciocnio: Se uma pessoa gasta 2/9 do que tem, ento fica com 7/9. De fato, representado-se por 9/9 a quantia toda, tem-se: 9 2 = 7 7 = 1 resto 9 9 9 9 Em segundo lugar, gasta 3/7 deste primeiro resto, ou seja: 3 de 7 = 3 x 7 = 1 7 9 7 9 3 Ora, quem tinha 7/9 e gasta 1/3 fica com: 7 1 = 7 3 = 4 4 = 2 resto 9 3 9 9 9 9 Estes 4/9, que representam o 2 resto, correspondem ao que sobrou em dinheiro. E dessa forma: 4/9 $ 8,00 1/9 $ 8,00 : 4 = $ 2,00 9/9 $ 9 x $ 2,00 = $ 18,00 Portanto : A senhora levava $ 18,00 ao sair de casa.

1) 2) 3) 4)

EXERCCIO PROBLEMAS COM NMEROS RACIONAIS

Eu tenho hoje $ 72,00. Minha irm Lcia tem 2/3 do que possuo. Quanto ela tem? Um automvel percorreu 3/5 de uma estrada toda que mede 120 km. Quanto percorreu? Os 3/4 do que eu possuo eqivalem a $ 180,00. Quanto eu tenho? Em virtude da paralisao da energia eltrica, uma fbrica de automveis que trabalha 9 horas por dia, trabalhou apenas 7/9 do tempo habitual. Qual foi esse tempo? Marcelo estuda para os exames durante 4 horas por dia. Hoje esteve ocupado com sua me e por esse motivo estudou apenas 1/8 do tempo habitual. Quantos minutos estudou?

5)

88

MATEMTICA PASSO A PASSO

6)

Se Maria Cndida estudar 3 horas por dia ela ter feito apenas 3/4 de sua obrigao diria. Quanto tempo levar para fazer a obrigao? Joaquim j fez 3/7 do muro e Geraldino 2/7 apenas. O muro todo dever medir 84 metros. Pergunta-se: a) Quantos metros cada um j construiu?b)Quantos metros faltam ainda para construir?

7) Joaquim e Geraldino so pedreiros que constroem um muro.

8)

Eu moro numa rua que mede 3 240 metros. O nmero de minha casa os 2/3 da metragem da rua. Qual o nmero de minha casa? Eu possuo hoje $ 72,00 e essa quantia os 3/4 do que eu possua ontem. Quanto eu tinha ontem?

9)

10) Salim herdou 5/8 de uma herana e depois deu para uma instituio de caridade 3/8 do que recebeu. Ficou ainda com a importncia de $2 400,00. Qual foi a herana de Salim?

11) O numerador da frao 2/9 foi duplicado. Pergunta-se: a)A frao


aumentou ou diminuiu? b)De quanto foi o acrscimo ou decrscimo?.

12) O numerador da frao 9/15 foi dividido por 3. Pergunta-se: a) A


frao aumentou ou diminuiu? b) De quanto foi o decrscimo ou acrscimo?

13) O denominador da frao 3/8 foi dividido por 2. Pergunta-se: a) A


frao aumentou ou diminuiu? b) Quanto aumentou ou quanto diminuiu?

14) Duplicou-se o numerador da frao 3/4 e depois dividiu-se o


denominador da mesma por 2. Pergunta-se: a) A frao final ficou aumentada ou diminuda? b) Se aumentou ou diminuiu, diga quantas vezes ela ficou aumentada ou diminuda. 15) O numerador da frao 18/21 foi dividido por 3 e do mesmo modo, o denominador foi tambm dividido por 3. Pergunta-se: o que aconteceu com o valor da frao? 16) Num dia, um tear tece 1/4 de uma encomenda de pano. No dia seguinte, tece mais 3/8 da encomenda. Desse modo, o teor completou 540 metros. Qual a encomenda toda? 17) Uma estante tem quatro prateleiras. A primeira mede 1/8 da altura da estante. A segunda mede 1/4 da altura da estante. Que frao da estante medem as outras duas prateleira juntas?

89

PROF. WELLINGTON BRITO

18) Conceio nadou os 2/3 do comprimento de uma piscina e seu irmo nadou 3/4 da mesma. Que frao da piscina nadaram juntos? Quanto isso representa, em metros, se o comprimento da piscina igual a 60 m? 19) O ingresso de um cinema custa 1/8 da minha mesada. Fui trs vezes ao cinema e gastei $ 8,40. Qual a minha mesada? 20) Maria Cndida diz a Luis: "Eu tenho hoje 2/3 de sua idade e juntos temos 30 anos. Quais as nossas idades". 21) Maria Clara diz a Conceio: "Nosso primo Marcelo tem 3/5 da minha idade e juntos temos 16 nos". Quais so as idades de Maria Clara e Marcelo? 22) Marcos Aurlio depositou uma certa quantia num banco. Quando retirou $ 120,00 do banco, verificou que essa importncia correspondia a 5/7 do que havia depositado. Qual foi o depsito inicial? 23) Uma pea de fazenda custou $ 540,00. Determinar qual o preo de 5/6 da pea. 24) Uma pea de 18 metros de brim custou $ 216,00. Dar o preo dos 3/4 de um metro dessa fazenda. 25) 5/8 da rea de um terreno foram ocupados com uma construo que usou 400 metros quadrados do mesmo. Pergunta-se: Que rea tinha o terreno? 26) Meu terreno e do meu vizinho tm juntos 3 200 metros quadrados. O terreno de meu vizinho 9/7 do meu. Dar a rea de cada um. 27) Lus e Osvaldo obtiveram notas em matemtica que, somadas, resultaram 16. A nota de Lus 7/9 da nota de Osvaldo. Quanto obteve cada um? 28) Os 5/8 dos 4/6 de minha mesada so $ 60,00. Qual a minha mesada? 29) Gastei 2/7 do que tinha na carteira em livros, e 1/3 em roupas. Tenho ainda $ 72,00. Quanto possua na carteira antes das compras? 30) Uma senhora faz compras. Gasta 3/10 do que levava em tecidos e 2/7 do que lhe restara em sapatos e desse modo gastou ao todo $ 253,75. Quanto levava em dinheiro? 31) Um auto percorre inicialmente os 3/11 de uma estrada. Numa segunda etapa roda 3/8 do que resta do percurso. Aps essa 2

90

MATEMTICA PASSO A PASSO

etapa ainda lhe faltam 340 km para percorrer. Qual a medida da estrada toda? 32) A diferena das quantias de Nair e Lcia $ 68,00. Nair possui os 15/13 do que possui Lcia. Quanto tem cada uma?

33) Um trem percorre 4/13 de seu percurso e depois roda mais 112 km
e assim chega aos 2/3 de seu caminho. Pergunta-se:a) Quanto percorreu at esse momento? b) Quanto mede o seu percurso todo? 34) Numa indstria 2/3 dos trabalhadores so homens e 1/4 so mulheres. Os restantes 30 so meninos. Quantos so os homens e as mulheres? 35) Numa sala de aula 3/8 das carteiras individuais esto ocupados por rapazes, 1/2 por moas e ainda existem 6 carteiras vazias. Qual a capacidade dessa classe? 36) Um pintor pode pintar 3/8 de uma parede em uma jornada de 9 horas de trabalho. Quanto tempo levar para pintar duas paredes iguais a essa? 37) Uma sociedade formada por 3 pessoas. A primeira entra com 2/5 do capital; a segunda com 1/3 do capital; e a terceira com $12.800,00. Qual a parte de cada scio e qual o capital da sociedade?

38) Se
de 5

1 kg de uma substncia custam $ 14,00, qual o preo

3 3 kg da mesma substncia? 5

39) Um pai repartiu $ 20.000,00 a seus trs filhos. Ao primeiro deu 5/3 do que deu ao segundo e a este 3/2 do que deu ao terceiro. Quanto recebeu cada um? 40) Uma cpsula espacial lanada para ser colocada em rbita aps percorrer determinada distncia. Aos 3/8 desse percurso o engenho solta seu primeiro foguete propulsor. Aos 2/5 do resto a percorrer o engenho abandona o segundo estgio propulsor e nesse instante ainda restam para alcanar a rbita 153 km. Pergunta-se: a que altura a cpsula entrou em rbita?.

91

PROF. WELLINGTON BRITO

Respostas 1) 2) 3) 4) 5) 6) 7) 8) 9) 10) 11)


$ 48,00 72 km $ 240,00 7 horas 30 min 4h a) Joaquim: 36m ---Geraldino: 24m b) 24 m 2 160 $ 96,00 $ 9.600,00 a) aumentou b) o acrscimo foi de 2/9 b) o decrscimo foi de 6 = 2 15 5 21) Maria Clara: 10 anos Marcelo: 6 anos 22) $ 168,00 23) $ 450,00 24) $ 9,00 25) 640 m2 26) Meu: 1 400m2 do vizinho: 1 800m2 27) Lus: 7 Osvaldo:9 28) $ 144,00 29) $ 189,00 30) $ 507,50 31) $ 748 km 32) Nair: $ 510,00 Lcia: $ 442,00 33) a) 208 km b) 312 km 34) Homens: 240 Mulheres: 90 35) 48 alunos 36) 48 horas 37) 1 pessoa - $ 19.200,00 2 pessoa- $ 16.000,00 Total: $ 48.000,00 85 m 38) $ 33,60 39) O 1 - $ 10.000,00 O 2 - $ 6.000,00 O 3 - $ 4.000,00 40) 408 km

12) a) diminuiu 13) a) aumentou


b) o aumento foi de 3/8

14) a) aumentou
b) a frao ficou aumentada 2 vezes

15) O valor da frao no se alterou. 16) 864 m 17) 5


8

18) 17
12 ;

19) $ 22,40 20) $ Maria Cndida: 12 anos


Lus: 18 anos

92

MATEMTICA PASSO A PASSO

QUESTES DE CONCURSOS E VESTIBULARES 1)(BB) Qual o nmero cuja oitava parte multiplicada por 12 e dividida po 5/6 resulta 144. a) 50 b) 60 c) 70 d) 80 e) 90 2)(TTN)Os 5 de 2 do preo de uma geladeira equivalem a 2 de 3 3 3 5 2 do preo de um freezer que custa R$ 2.400,00. Ento o preo da geladeira, em reais, : a) 2.000 b) 1.296 c) 1.440 d) 2.160 e) 1.300

3)(BB) Alberto comprou 1/6 de certo terreno, Bento e Carlos compraram, respectivamente, 2/5 e 2/15 do resto. Calcular a rea primitiva desse terreno, sabendo que dele sobraram 1.470m 2, , aps a ltima venda. a) 3.870m2 b) 3.087m2 c) 3.708m2 d) 3.780m2 e) 3.807m2

4)(CEF) Retirei, inicialmente, uma quinta parte de minha conta bancria. Depois, saquei uma quarta parte do resto e ainda sobraram R$ 7.500,00. Qual era, em reais, o saldo inicial? a) 12.750 b) 12.500 c) 12.250 d) 10.200 e) 10.500

5)(UNIFOR) Se o triplo de um nmero 18 , ento: 5 a) seu quntuplo 18 d) sua metade 2 b) seu quadruplo 4 5 c) seu dobro 12 e) sua tera parte 1 5 5 6)(UECE) Considere a expresso algbrica
x+ 1 x 1

1
seu valor numrico para x =

,x 0 e x 1. x+1 1x

a) 5 1

5 b) negativo

c) 2,5 93

d) 5,2

PROF. WELLINGTON BRITO

7)(UFC) Trs irmos, Maria, Jos e Pedro receberam, respectivamente, 1/2, 1/3, e 1/9 de uma determinada herana. A frao desta herana que no foi distribuda entres estes irmos foi de: a) 2/3 b) 8/9 c) 1/2 d) 1/18 e) 5/6 8)(UFC) Se 3 a) 15 1 1 + 1 4 b) 16
2

p q

onde p e q so nmeros inteiros primos entre si, determine p + q. c) 17 d) 18 e)19

9)(UFC) Determine o valor de S, onde: S= 8 a) 36

() ( )
1 1 2 x 1 2 c) 34

4 + 5 d) 33 e) 32

b) 35

10)(TRT) O primeiro andar de um prdio vai ser reformado e os funcionrios que l trabalham sero removidos. Se 1 do total dos funcionrios devero ir para o segundo andar, 3 2 do total para o terceiro andar e os 28 restantes para o quarto 5 andar, o nmero de funcionrios que sero removidos : a) 50 b) 84 c) 105 d) 120 e) 150 11)(TJ) Rita sai de casa para fazer compras com certa quantia. Na primeira loja gastou 2 do que possua; na segunda loja gastou 3 R$ 30; na terceira R$ 10,00 e 2 do que restou. 5 Sabendo-se que no final das compras ficou com R$ 60,00, ao sair de casa, Rita tinha a importncia de: 94

MATEMTICA PASSO A PASSO

a) R$ 420,00 b) R$ 300,00

c) R$ 360,00 d)R$ 330,00

e)R$ 450,00

12)(TRT) Uma pessoa saiu de casa para o trabalho decorridos um dia e retornou sua casa decorridos 13 do mesmo dia. 16 Permaneceu fora de casa durante um perodo de: a) 14 horas e 10 minutos. b) 13 horas e 50 minutos. c) 13 horas e 30 minutos.

de

18

d) 13 horas e 10 minutos e) 12 horas e 50 minutos.

13)(TTN) Que horas so agora, se 1 do tempo que resta do dia igual ao tempo j decorrido? 4 a) 8 horas b) 4 horas c) 4 horas e 48 minutos d) 6 horas e 38 minutos e) 5 horas e 15 minutos

Respostas

01. D 02. B 03. D

04. B 05. C 06. C

07. D 08. E 09. A

10. C 11. A 12. E

13.C

95

PROF. WELLINGTON BRITO

NMEROS DECIMAIS Representao Decimal Os nmeros fracionrios


3 10

, 17 , 121 cujos denominadores so


100 1000

potncias de 10 so chamados fraes decimais. Como j se viu: 3 representam 3 partes de um inteiro que se dividiu 10 em 10 partes iguais; 3 representam 3 partes de um inteiro que se dividiu 100 em 100 partes iguais ; Assim: 3 e 0,3 so numerais do mesmo nmero; e 10 3 e 0,03 so tambm numerais do mesmo 100 nmero. 0,3 e 0,03 so chamados numerais decimais. Algumas vezes utiliza-se simplesmente a forma nmeros decimais. Leitura dos Nmeros Decimais As ordens decimais recebem nomes especiais. O esquema seguinte d uma idia: Unidade

4 8 3 1, 4

2 5

Convm, ento, observar os nomes que recebem as diferentes ordens decimais menores que a unidade. 0,1 ----- um dcimo

96

MATEMTICA PASSO A PASSO

0,01 ----- um centsimo 0,001 ----- um milsimo 0,0001 ----- um dcimo de milsimo 0,00001 ----- um centsimo milsimo 0,000001 ----- um milionsimo 0,0000001 ----- um dcimo milionsimo

-------------------------------------------------------0,000000001 ----- um bilionsimo

Para se ler, por exemplo: 2,437, diz-se: "2 inteiros e 437 milsimos". Ou: 2,437 so: "2 inteiros, 4 dcimos, 3 centsimos e 7 milsimos". Transformao de Frao Decimal em Nmero Decimal Passo a Passo. Regra: Para se transformar uma frao decimal, em nmero decimal, basta dar ao numerador tantas casas decimais quantos forem os zeros do denominador. Exemplos: Transformar: 1)

31 100

= 0,31

3)

47 10

= 4,7

2)

5 1000

= 0,005

4)

1231 100

= 12,31

Transformao de Nmeros Decimais em Fraes Decimais Passo a Passo. Regra:

Todo nmero decimal igual a uma frao, onde o numerador o nmero decimal sem a vrgula e o denominador a unidade, seguida de tantos zeros quantos forem as ordens decimais do nmero dado. Exemplos:

97

PROF. WELLINGTON BRITO

Transformar, em fraes decimais, e simplificar quando for o caso: 15 3 1) 0,015 = = 1000 200 125 100 5 4 1 4

2) 1,25 =

= 1

Propriedades de Nmeros Decimais. 1 . Propriedade: Um nmero decimal no se altera quando se acrescenta um ou mais zeros direita de sua parte decimal.

Ora, pela propriedade fundamental das fraes, podemos multiplicar os termos de 3 por 10, por 100, por 1000 etc., que 10 sempre obteremos fraes equivalentes a 3 assim: 10 3 = 30 = 300 = 3000 10 100 1000 10 000 Ou, na forma de nmeros decimais: 0,3 = 0,30 = 300 = 3 000 o que prova a propriedade 2. Propriedade: Para se multiplicar um nmero decimal por 10, por 100, por 1000... etc., basta afastar a vrgula para a direita uma, duas, trs...etc. casas decimais. Exemplos: 1,24 x 10 = 12,4 1,24 x 100 = 124 1,24 x 1000 = 1 240

3 Propriedade: 98

MATEMTICA PASSO A PASSO

Para se dividir um nmero decimal por 10, 100, 1000 etc., basta afastar a vrgula para a esquerda uma, duas, trs... etc. casas decimais. Exemplos: 387,2 : 10 = 38,72 387,2 : 100 = 3,872 387,2 : 1000 = 0,3872

Adio de Decimais - Passo a Passo Efetuar a soma: 4,2 + 15,27 + 0,375. Escrevendo-os sob a forma de fraes, teramos 42 10
+

1527 100

375 1000

Ou, reduzindo-as ao mesmo denominador comum: 4200 + 15 270 + 375 + 19845 = 19,845 1000 1000 1000 1000 Evidentemente no se far a transformao em fraes todas as vezes. O exemplo acima serve para justificar os seguintes fatos tradicionais: 1) Igualam-se as casas decimais o que eqivale a homogeneizar as fraes. 2) Coloca-se vrgula debaixo de vrgula o que eqivale a somar apenas as unidades de uma mesma ordem entre si. Na prtica faremos: 4,2 + 15,27 0,375 Igualando-se as "casas" (ordens) decimais Que l-se: "19 inteiros e 845 milsimos Subtrao de Decimais - Passo a Passo Segundo as consideraes anteriores, efetuar: 14,25 8,075 Igualando-se as ordens decimais, vem: 14,250 99 4,200 15,270 + 0,375 19,845

PROF. WELLINGTON BRITO

8,075 6,175 Multiplicao de Decimais - Passo a Passo Seja o produto: 1,3 x 42,71. Colocando-se (para fins de justificar as concluses ) sob a forma de frao decimal,vem: 13 x 4271 = 10 100 55 523 = 55,523. 1000

Conclui-se pois: Multiplicam-se os nmeros decimais como se fossem nmeros Inteiros e d-se ao produto tantas casas decimais quantas unidades somarem as casas decimais do multiplicando e do multiplicador. Exemplo: 8,45 1,5 x 4225 845 12,675 Que se l: "12 inteiros e 675 milsimos".

Diviso de Decimais Passo a Passo Estudaremos os seguintes casos: 1) Diviso de inteiro por decimal Seja: 5 : 0,25 Cortam-se as vrgulas Justificativa: Eliminar as vrgulas, aps o acerto das ordens, significa multiplicar dividendo e divisor por 100 ( no caso presente). O quociente no se altera. Resulta, ento: 500 25 000 20 Logo: 5 : 0,25 = 20 2) Diviso de decimal por inteiro Dividir: 0,25 : 5 Acertados as ordens decimais, vem: 0,25 5,00 100

MATEMTICA PASSO A PASSO

Suprimidas as vrgulas, fica: 25 500 Acrescenta-se ao dividendo quantos zeros forem necessrios para tornar a diviso possvel. O mesmo nmero de zeros colocar-se- no quociente, com a vrgula aps o primeiro zero. 2 500 500 0,0 A diviso se processa normalmente como diviso de inteiros a partir desse instante. 25 5 00 0,05 Portanto: Quocientes Aproximados. 1 . 0,25 : 5 = 0,05

Exemplos:

1) Dividir 31 por 7 com erro a menos de 100 Faz-se o produto do dividendo por 100:

3100 7 30 442 20 6 D-se duas ordens decimais ao quociente (4,42). Logo: 31:7 = 4,42 a menos de 1 100 1 . 1000

2) Dividir 2,5 por 41 a menos de

Multiplica-se o dividendo por 1000. Dividir-se- o quociente por 1000 ao final. 2,5 x 1000 = 2500 2500 41 040 60 (60 : 1000 = 0,060). Dividindo-se o quociente por 1000, o resultado : 2,5 : 41 = 0,060, a menos de 1 1000 CONVERSO DE FRAES ORDINRIAS EM NMEROS DECIMAIS ----DIZIMAS PERODICAS ----101

PROF. WELLINGTON BRITO

Sejam as fraes seguintes, que desejamos transformar em nmeros decimais: I) 3 ; 4 II) 4 ; 11 III) 5. 6

Como uma frao indica a diviso do numerador pelo denominador, vem: I) 3 4 30 4 20 0,75 0 3 = 0,75 uma decimal exata. 4

II) 4 11

40 11 70 0,3636... 40 70 4.... 4 = 0,363636... uma decimal no exata.

Mais precisamente: uma dzima peridica simples. Diz-se que o perodo 36. Isto : o nmero, formado por mais de um algarismo, que se repete na parte decimal, chamado perodo. A dzima peridica simples quando o perodo tem incio logo aps a parte inteira. Exemplos: a) 0,414141... = 0,41 dzima peridica simples; b) 2, 333... = 2,3 dzima peridica simples. III) 5 6 50 6 20 0,8333... 20 20 2... 5 6 = 0,8333...= 0,83 uma dzima peridica composta

Fraes Geratrizes das Dzimas Peridicas

As geratrizes se determinam segundo as regras seguintes:

102

MATEMTICA PASSO A PASSO

A frao geratriz, de uma dzima peridica simples, uma frao que tem como numerador o perodo e como denominador tantos noves quantos algarismos tiver esse perodo. Exemplos: Achar as geratrizes das seguintes dzimas: 1) 0,3 = 3 9 2) 0,36 = 36 99 = 1 3 = 4 11 4) 2,5 = 2 ; 3) 0,423 = 423 999 5 9 = 47 111

A geratriz, de uma dzima peridica composta, uma frao, onde o numerador formado pela parte no peridica, seguida do perodo, menos a parte no peridica. O denominador possui tantos noves quantos so os algarismos do perodo, seguidos de tantos zeros quantos forem os algarismos da parte no peridica. Exemplos: Determinar as geratrizes de: 1) 2,4222... = 2,42 = 2 42 4 90 2) 5,32121.... = 5,321 = 5 321 3 990 = 5 = 2 38 90 318 990 = 5 = 2 19 ; 45 53 165

EXERCCIOS NMEROS DECIMAIS 1) Transformar, em nmeros decimais, as seguintes fraes decimais: a) 3 d) 51 g) 137 10 100 1000 b) 1 e) 71 h) 15 431 1000 10000 100 c) 5731 f) 5731 i) 5731 10 100 1000 2) Efetuar as seguintes operaes, apenas com a mudana da vrgula: 103

PROF. WELLINGTON BRITO

a) 12,4x100 b) 137,421 x 1000 c) 0,0001 x 10000 d) 0,0005 x 104 e) 103 x 12,1

f) 5,05 x 105 g) 0,0003 x (102)3 h) 0,0000001x (10 ) i) 345,21: 10 j) 345,21 : 100


4 2

k) 345,21 : 1000 l) 5000,0001:1000 m) 72 : 102 n) 540,25 : 10 3 o) 853,2314 : (10 2)3 p) 1672,435 : (104)2

3) Calcular os seguintes quocientes aproximados por falta a menos de 1 : 100 a) 7 : 11 d) 0,005 : 15 g) 0,0003 : 0,00035 b) 15,2 : 7,1 e) 35,2 : 0,08 h) 7,42 : 2,1 c) 16,42 : 5 f) 123,4 : 2,45 4) Converter em nmeros decimais as seguintes fraes ordinrias: a) 2 d) 11 g) 4 i) 1 5 18 11 125 b) 5 e) 3 h) 4 k) 213 6 8 33 200 c) 7 f) 6 i) 4 l) 134 4 7 99 135 5) Obter as fraes geratrizes das seguintes dzimas peridicas: a) 0,414141..... d) 0,0102 g) 0,4215 i) 1,8 b) 2,54 e) 2,22 h) 0,0001 j) 3,40501 c) 5,4231 f) 0,12 6) Efetuar as seguintes expresses: a) 53,17 + 6,0008 + 2,752 + 0,2 b) 35,8 (11,111 + 0,0005) + 0,1 c) 3,7 x 0,005 + 7,506 7,384 x 0,92 d) 5,022 : 2,79 0,1 x [ ( 17,83 + 32,17) x 0,05 + 15,5] e) 0,1 + 1 + 3

2 + 34

f) ( 5,7 5,6 ) +

15

) ( ( ( )
1 15 + 1 3

2 25

+ 0,4 0,12 1 18 104

17 50

0,3

+ 0,5

MATEMTICA PASSO A PASSO

g) (3,8 3,7)+

4 + 2 + 3 15 5 + 3

3 10 5 x 18 7 11 6

h) 8,1 x 0,2 + i)

4,41 1,93 +

( )
4 9 +

3,7 + 2,5 7 3 7 11

29 +

25

0,01

50

j)

3,47 2 +

4 33

0,09

+ 1,2 0,3 + 0,41

7 330

QUESTES DE CONCURSOS E VESTIBULARES 1. (BB) Valor de x nas expresses x = (4,8 1,02) 0,4 : a) 2,25 2. b) 4,15 c) 5,75 d) 9,45 e) 9,7

(TRT) Simplificando-se a expresso: 0,015 + 0,01 0,5


obtm-se:

0,0003 0,002 0,01

a) 0,025 3.

b) 0,11

(TRE) Efetuando-se a) 25 8 b) 15 4

c) 0,25

1 0,025 5 c) 35 8

d) 1,1

e) 2,5

0,04 obtm-se d) 25 4 e) 35 4

4.

(PRF) O valor de a) 0,75 b) 1,245

1,5 0,01 de: 0,12 0,2 c) 1,25 d) 12,45


2

e) 12,5

5.

(TRT) Resolver a seguinte expresso:

( ) (
3

2 1 +1 3 + 1 1 6 2 4 2

a) 3 b) 4 6.

c) 4 11

d) 5 3

e) 3 16

(CEF) O valor da expresso x3 3x2y + 3xy2 y3 , para x = 1/2 e y = 1/2 : a) 1 b) 1/5 c) 0 105 d) 1/8 e) 1

PROF. WELLINGTON BRITO

7.

(TRT) Na expresso abaixo, o trao horizontal sobre o nmero indica o perodo da dzima peridica: 0,222. : 0,0404.. 2,166..+ 0,4166. x 0,2666... resolvendo essa expresso, obtm-se: a) 3 4/9 b) 29/9 c) 0,84 c) 29 180 d) 3,2 d) 29 198 e) 0,1666... e) 145 999 (DNER) A dzima peridica 0,1454545... igual a: a) 5 11 b) 8 55

8.

9.

(BNB) A expresso decimal 0,011363636... uma dzima peridica composta. Se a geratriz desta dzima for escrita sob a forma irredutvel m / n, ento m + n igual a: a) 88 b) 89 c) 90 d) 91 0,333.... x 3 2 1,98 d) 51 e) 92 1 : e)50

10. (TJ) O valor da expresso 0,6 x 1 + 4 3 5 a) 54 b)53 c)52

11. (PRF) 0 1994 algarismo, aps a virgula, na representao decimal de 12/37 : a) 1 b) 2 c) 3 d)4 e)5 12. (UECE) Na seqncia SPMSQSPMSQSP.... que letra ocupa a 90 posio? a) S b) P c) M d) Q 13. (UECE) Se contarmos 2000 dias a partir de amanh (Terafeira), qual o dia da semana que encontramos? a) Quarta-feira b) Quinta-feira c)Sexta-feira d) Sbado 1 ,x0 x+1 1-x 106

x+1 14. (UECE) Considere a expresso algbrica x - 1 1

MATEMTICA PASSO A PASSO

e x 1 seu valor numrico para x = 2 5 a) 5-1 b) negativo c) 2,5

: d) 5,2

15. (UNIFOR) Na "Notao cientfica, os nmeros so escritos como produto de um nmero x, por uma potncia de 10. Por exemplo, 1000 = 1 x 103 e 0,02 = 2 x 10-2. O valor de 0,00015 x 24000 x 0,0003 :
a) 1,08 x 10-3 b) 3,6 x 10-2 c) 4,5 x 10-7 d)9,08 x 10-4 e) 3,08 x 10-2

Respostas Questes de Concursos e Vestibulares


1) 2) 3) D D C 4) D 5) A 6) E 7) A 8) B 9) B 10) C 11) B 12) D 13) D 14) C 15) A

Respostas Exerccios Nmeros Decimais 1) a) 0,3


b) 0,001 c) 573,1 d) 0,51 e) 0,0071 f) 57,31 g) 0,137 h) 154,31 i) 5,731 3) a) 0,63 b) 2,14 c) 3,28 d) 0,00 e) 440 f) 50,36 g) 0,85 h) 3,53 4) a) 0,4 b) 0,83 c) 1,75 d) 0,61 e) 0,375 f) 0,857142 g) 0,36 h) 0,12 f) 5) a) 41 99 b) 229 90 c) 54 177 9 990 d) 101 9 900 e) 220 99 12 99 g) 4173 9900 h) 1 9000 i) 17 j) 1 6) a) 62,1228 b) 24,7885 c) 0,73122 d) zero e) 2 f)1 43 45 g) 3 h) 10 i) 1 7 33 3 11

2) a) 1240
b) 137421 c) 1 d) 5 e) 12100 f) 505000 g) 300 h) 10

107

PROF. WELLINGTON BRITO

i) 34,521 j) 3,4521 k) 0,34521 l) 5,0000001 m)0,72 n) 0,54025 o) 0,0008532314 p) 0,00001672435

i) 0,04 j) 0,008 k) 1,065 l) 0, 9925

9 j) 340 161 99 900

SISTEMA MTRICO DECIMAL Definio Chama-se metro linear ao comprimento equivalente frao1/10 000 000 da distncia que vai de um polo at a linha do equador, medida sobre um meridiano Esse comprimento, aps calculado , encontrase assinalado sobre uma barra de metal nobre (platina e irdio ) que est depositado no Equador Museu Internacional de pesos e medidas em Svres, na Frana. O Museu Nacional, no Estado da Guanabara, tem uma cpia do metro padro.

Polo

Mltiplos e Submltiplos do Metro Linear Os mltiplos e submltiplos do Sistema Mtrico Decimal tm seus nomes formados com os seguintes prefixos de origem grega: quilo que significa mil vezes hecto que significa cem vezes deca que significa dez vezes deci que significa dcima parte centi que significa centsima parte mili que significa milsima parte Desse modo temos o seguinte quadro:
Mltiplos Unidade Submltiplo decmetro centmetro milmetro

quilmetro hectmetro decmetro metro

108

MATEMTICA PASSO A PASSO

1 000m km

100m hm

10m dam

1m m

0,1m dm

0,01m cm

0,001m mm

Exemplos: 1) 4,52 km, l-se sob uma das seguintes formas: "4 quilmetros e 52 centsimos de quilmetros", ou: "4 quilmetros e 52 decmetros". 2) 123,425 m "123 metros e 425 milsimos do metro", ou: "123 metros e 425 mlmetros".

Transformaes de Unidades Passo a Passo As mudanas de unidades do sistema linear de medidas (medidas de comprimento) fazem-se com base no fato seguinte: Cada unidade de comprimento 10 vezes maior que a unidade imediatamente inferior. Assim, para se passar de km para hm multiplica-se por 10 e para passar de dm para m deve-se dividir por 10. Exemplos:
1) 0,02 hm em metros 2) 54,36 dm em dam 3) 0,425 km em cm 0,02hm = (0,02 x 100) m = 2m 54,36dm = (54,36 100) dam = 0,5436dam 0,425 km = (0,425 x 100 000) cm = 42 500 cm

Na prtica, cada mudana de vrgula para a direita ( ou multiplicao por 10) transforma uma unidade na imediatamente inferior a cada mudana da vrgula para a esquerda ( ou diviso por 10) transforma uma unidade na imediatamente superior. Permetro de um Polgono 1) O permetro de um polgono a soma dos comprimentos de seus lados. D No polgono ABCDE o permetro a soma das medidas dos lados,ou seja: C med AB + med BC + med CD+ med DE + med EA. E Representando-se o permetro por 2P teramos:
2P=med AB+med BC+ med CD + med DE + med EA

B 109

PROF. WELLINGTON BRITO

2) Num retngulo, como o da figura seguinte, o permetro 2P fica: W Z med base = b med altura = h h X b 2P = 2b + 2h O que significa: O Permetro do retngulo igual a duas vezes Y a medida da base mais duas vezes a medida da altura.

3) No quadrado o permetro quatro vezes o lado. D C OU 2P = 4 . L A B

4) No tringulo eqiltero o permetro trs vezes a medida do lado. A OU 2P = 3 . L B C 5) Na circunferncia o permetro pode ser calculado fazendo-se o seguinte: Chamando-se o comprimento da circunferncia r de C e o seu dimetro de 2 r (dois raios = 1 dimetro), teremos: C C = 3,14 2r E,na definio de diviso exata, vem: C = 2 x 3,14 x r Costuma-se representar o nmero 3.14 pela letra (pi) do alfabeto grego. Assim: C=2xxr Exemplos: 110

MATEMTICA PASSO A PASSO

1) Calcular qual a medida do contorno de um aqurio de forma circular, sabendo que o dimetro do mesmo 6 metros. Temos: o raio vale 3 metros. Logo: C = 2 x x r Donde: ou: C = 2 x 3,14 x 3m C = 18,84m

2) A medida que vai do centro da roda de minha bicicleta at a face externa do pneu 28 cm. Quando a roda d uma volta, quantos centmetro percorri? Sendo r = 28 cm, vem: C = 2 x x r C = 2 x 3,14 x 28 C = 175,84 cm

1,7584 metros.

EXERCCIOS SISTEMA MTRICO DECIMAL - I : I) Expressar em metros as seguintes grandezas: 1) 0,005 hm 2) 1,2 km 3) 134,2 dm 4) 1/4 hm 5) 5/6 km 6) 3/8 dam 7) 1 8) 3 9) 4 1 4 3 8 1 6 II) Expressar em dm os seguintes resultados: 1) 2,5 m + (5,4 hm 48 dam) 2) 5,28 dm + [ 85 dam (4,5 km 42 hm)] 3) 4,2 km [( 65 dm + 8,5 m) + ( 25 dam 240 m)] 4) 0,08 hm + [ 0,05 km + (120 hm 11,2 km)] 5) 120 hm [ 10 dam (120 m 1120 dm)] III) Resolver os problemas: 1) Calcular o permetro de um polgono de 5 lados, onde o menor lado vale 4dm e os outros so nmeros consecutivos a este. D E C 111 cm hm dm

PROF. WELLINGTON BRITO

A B 2) Determinar o permetro de um retngulo, onde um dos lados vale 12 cm e o outro os 5/4 do primeiro. . h b

3) O permetro de um retngulo 60 cm e a base o dobro da altura. Calcular base e altura. h b 4) Um tringulo issceles (tem dois lados iguais). A soma da base (lado desigual) com um dos lados 28 cm. Calcular o permetro sabendo que o lado o triplo da base. Z L L X b Y 5) Num tringulo retngulo, os catetos (lados que formam o ngulo reto) somam 16 cm e um deles 5/3 do outro. Calcular esses catetos. C x y A z B

6) O comprimento de uma circunferncia 18,84 cm. Calcular o raio da mesma . r 112

MATEMTICA PASSO A PASSO

7) Calcular o comprimento de uma circunferncia cujo raio vale 10cm. Respostas I) II) III) 1) 0,5 m 1) 625 dm 1) 30 dm 2)1200 m 2) 5 505,28 dm 2) 54 cm 3)13,42 m 3) 41750 dm 3) b = 20 cm; h= 10 cm 4) 25 m 4) 8580 dm 4) 49 cm 5) 833,3 m 5) 119080dm 5) b= 6cm ; c = 10cm 6) 3,75 m 6) r = 3 cm0,125 m7) 62,8 7) 337,5 m 7) 62,8 8) 0,0416 m

Superfcie Da rea

A Idia de superfcie conhecida. uma noo que se diz intuitiva porque a conhecemos sem necessidade de defini-la. Assim, a superfcie da mesa, do assoalho do vidro, da janela, so superfcies planas. A superfcie de uma bola de futebol esfrica. No momento, vamos nos preocupar com as superfcies planas. Metro Quadrado D C
1m

Chama-se metro quadrado ao quadrado que tem um metro de lado

1m

Mltiplos e Submltiplos do Metro Quadrado


Mltiplos Unidade Submltiplo

quilmetro hectmetro decmetro metro decmetro centmetro milmetro quadrado quadrado quadrado quadrado quadrado quadrado quadrado
1 000 000 10 000 100 1 0,01 0,0001 0,000 001

m km2

m2 mm2

hm2

dam2

m2

dm2

cm2

113

PROF. WELLINGTON BRITO

Transformaes de Unidades Passo a Passo I) Para se converter um nmero, medido numa unidade, para a unidade imediatamente inferior, deve-se multiplic-lo por 100. II) Para se converter um nmero, medido numa unidade, para a unidade imediatamente superior, deve-se dividi-lo por 100.

Na prtica afasta-se a vrgula para a direita (I) e para a esquerda (II) de duas em duas ordens decimais. Exemplos:
Transformar: 1) 5,24 dam2 em dm2 5,24 dam2 = 52 400 dm2 2) 241,2 cm2 em dam2 241,2cm2 = 0,000 2412 dam2

Unidades Agrrias

A medida de terras se faz segundo unidades especiais ditas agrrias. A unidade fundamental are. Chama-se are ao quadrado que tem 10 metros de lado . As unidades agrrias se resumem no seguinte quadro: UNIDADES AGRRIAS Hectare Are Centiare ha a ca 100 a 1a 0,01a 10.000 m2 100 m2 1 m2

EXERCCIOS SISTEMA MTRICO DECIMAL (II) I) Calcular, em metros quadrados, as seguintes grandezas: 1) 5/4 dam2 2) 3)
4) 3/5 cm2
2

7) 1
2

dam2

( (

1 1 3 0,2 + 1

+ 0,2 mm 5) 1
2

) )

dm

( (

1 +1 mm 8) 0,125+ 8 2 4 1 3
2

6) 2

+ 0,02 km

( ) (
9)

0,25+1

) )

hm
2

km

II) Efetuar as seguintes operaes em dm2

114

MATEMTICA PASSO A PASSO

1) 2) 3) 4) 5)

2,25 km2 ( 80 hm2 120 dm2 ) 0,4 hm2 [ ( 5,2 dam2 ( 8,6 m2 120 dm2)] 54,5 dm2 [ 0,04 m2 ( 12 dm2 1100 cm2)] 100 m2 + [ 120 dm2 (12400 cm2 84 dm2)] (2,4 dam2 + 120 dm2) ( 540 cm2 + 2,8 m2) 4) 15,45a 5) 120 ca + 15,5a 6) 10 ha 10a

III)Transformar, em metros quadrados, as seguintes medidas agrrias:

1) 120 ha 2) 10 ha + 105 a 3) 1 ha + 1a + 1ca Respostas

I) 1) 125m2 7) 125 m2 II) 1) 145000120dm2 III) 1) 1200000 m2 2 2) 0,000 00153 m 8) 3 750m2 2) 348740 dm2 2) 110500 m2 3) 0,0145 m2 9) 1500000m2 3) 51,5 dm2 3) 10101 m2 4) 0,00006 m2 4) 10080 dm2 4) 1545 m2 5) 0,00001875 m2 5) 23834,6 dm2 5) 1670m2 6) 2 353333,333 m2 6) 99000 m2

REAS PLANAS rea do Retngulo

A rea do retngulo igual ao produto da base pela altura Representando-se a rea por S, a medida da base por b e a medida da altura por h, vem: S=bxh Exemplo: O permetro de um retngulo 48 cm e a base o triplo da altura. Determinar a rea desse retngulo. Teremos a soluo: h b=3xh I) Clculo da base e da altura Como o permetro vale 48 cm ento: base + altura = 24 cm Ou: b + h = 24 cm Mas a base vale 3 alturas, ou seja: b=3xh (que l-se: "base igual a 3 alturas) Donde: 1h = 6 cm Logo: b = 18 cm

Logo: 3h + h = 24 cm 4h = 24 cm. Mas como: b = 3 x h, ento: b = 3 x 6 cm II) Clculo da rea: 115

PROF. WELLINGTON BRITO

S=bxh Ou: S = 18 cm x 6 cm = 108 cm2 S = 108 cm 2 L L Donde: A rea do quadrado igual ao quadrado dos lados rea do Quadrado O quadrado um retngulo que tem lados de medidas iguais Logo: S = L x L ou S = L2

Exemplo: O permetro de um quadrado mede 96 dm. Calcular a rea. O lado a quarta parte do permetro, ou: L = 96 dm 4 = 24 dm S = L2 Donde: S = 24 dm x 24 dm ou: S = 576 dm2 D h A E
b

rea do Paralelogramo C B A rea do paralelogramo igual ao produto da base pela altura. Ou: S=bxh

A C
y

rea Do Tringulo D C A rea do tringulo igual ao semiproduto da base pela altura. h B Ou: S= bxh b 2
x

Caso Particular: Se o tringulo retngulo, seus lados tm nomes prprios, conforme est assinalado na figura abaixo. Os lados que formam o ngulo reto so catetos e o terceiro lado chama-se hipotenusa. 116 Os catetos soindiferentemente base e altura do tringulo.

MATEMTICA PASSO A PASSO

z Assim:

B A rea do tringulo retngulo igual ao semi-produto dos catetos.

Ou: S = y . z 2 rea do Trapzio W X b b Z Y A rea do trapzio igual ao produto da semi-soma das bases, pela altura. S = ( b + b ) x h 2

rea Do Losango
D Q

A rea do losango igual ao semi-produto de suas diagonais. d A


C

Ou: S = d x d 2

d rea Do Circulo Seja um crculo de raio r, que vamos dividir no maior nmero possvel de partes de medidas iguais. r A rea do crculo igual ao produto do nmero (3,14) pelo quadrado do raio. S=r2

Questes Comentadas 1) Calcular a rea do losango onde a soma das diagonais vale 30 cm sendo uma o dobro da outra.
117

PROF. WELLINGTON BRITO

Raciocnio: Chamando as diagonais de d e d `, tem-se: d + d = 30 cm d Como d o dobro de d, vem: d = 2d Ou: 2d+ d = 30 cm 3d = 30cm d = 10 cm e d = 20 cm Donde: S = d x d = 20 x 10 = 100 cm 2 ou 2 2 S = 100 cm2 d

2) Calcular a rea de um trapzio onde as bases medem 8m e 10m e a altura 2/5 da base maior. Pelos dados do problema temos: vem: b = 10 m Como: h = 2 x 10m = 4m b= 8 m 5 h = 2/5 b S = (b + b) x h = (10 + 8) x 4 Ou: S = 36 m2 2 2

3) Calcular a rea do crculo, sabendo-se que o comprimento 18,84 dm. Raciocnio: Como temos: C = 18,84 dm 2 r = 18,84 dm C=2r Ou: 6,28 r = 18,84dm dm r = 18,84 dm 6,28 r = 3dm Calculando-se a rea vem: S = r = 3,14 x 32 = 3,14 x 9 ou S=28,26dm2 EXERCCIO REAS PLANAS 1) Calcular a rea do retngulo cuja base vale 1,25m e cuja altura 1/5 da base. h b 2) A soma entre a base e a altura de um retngulo 7,2 m e a base o triplo da altura. Calcular a rea. h h b 118

MATEMTICA PASSO A PASSO

3) O permetro de um quadrado 60 cm.Calcular a rea do retngulo cuja base o lado desse quadrado e cuja altura a metade desse lado. h L

b 4) O permetro de uma circunferncia 314m. Calcular a rea do crculo cujo contorno essa circunferncia. 5) Num tringulo, a base vale 0,54m e a altura 2/3 da base. Calcular a rea h b

6) Calcular a rea do retngulo cuja base vale 150cm e cuja altura mede 12 dm h b 7) A soma entre a base e a altura de um retngulo 36cm e a base o dobro da altura. Calcular a rea. h b 8) O permetro de um retngulo 160mm. A base os 9/7 da altura. Calculara rea do retngulo. h b 9) O permetro de um quadrado 30cm. Qual sua rea 119

PROF. WELLINGTON BRITO

10) Para cercar um terreno retangular com 3 voltas de arame, foram gastos 216m. Calcular a rea do terreno se o comprimento o triplo da distncia que vai da frente ao fundo. a b 11) Num tringulo a soma dos catetos 20cm e um deles os 7/3 do outro. Qual a rea. C b

12) A soma entre a base e a altura de um tringulo 72cm sendo a base o dobro da altura. Calcular a rea do tringulo. h b 13) Sobre os catetos de um tringulo constroem-se quadrados, Calcular a rea de cada um desse quadrados, sabendo-se que a soma dos catetos de 32cm e um deles o triplo do outro. S1 A S2 14) Calcular a rea do crculo cujos 3/5 do raio medem 15metros. 15) Calcular a rea do losango onde as diagonais medem 6 dm e 54 cm, respectivamente. B

120

MATEMTICA PASSO A PASSO

16) Num trapzio a altura vale 6cm e as bases medem respectivamente, 8 e 10cm. Calcular a rea. h b 17) Um trapzio retngulo (tem um ngulo reto). A altura vale 10 dm e as bases 8 e 14dm, respectivamente. a) Calcular a rea do mesmo. b) Calcular a rea que pertence ao trapzio e no per tence a um crculo que tem centro no ponto mdio do lado AD do trapzio e raio igual metade desse lado. D C h A B

18) Calcular a rea do crculo cujos 7/3 do raio medem 42 cm 19) Calcular a rea dos seguintes tringulos Assinalados que fazem parte do trapzio issceles abaixo. b= 12 cm h = 6 cm b=18 cm 20) Comprei uma chcara de forma retangular que mede 120m de frente por 200 m da frente at o fundo.Paguei R$2400,00 quanto me custou a chcara?

21) Para ladrilhar a rea de minha casa, que tem forma retangular e mede 6,4m por 9,60m, comprei ladrilhos quadrados de 0,20 m de lado. a) Quantos ladrilhos foram ocupados? b) Quanto gastei se paguei R$ 8,00 por cento?

121

PROF. WELLINGTON BRITO

22) A rea de um tringulo 54 dm2 e sua altura 12 dm. Calcular a base do tringulo.

Respostas 1) 3125 cm2 2) 9,72 m2 3) 112,5 cm 4) 7850 m2 5) 972 cm2 6) 180dm2 7) 288 cm2 8) 1575 mm2 9) 56,25 cm2 10) 243 m2 11) 42 cm2
2

12) 576 m2 13) S1 = 64 cm2 S2 = 576 cm2 14) 1962,5 m2 15) 1620 cm2 16) 54 cm2 17) a) 110 dm2 b) 70,75 dm2 18) 1017,36 cm2 19) 9 cm2 20) R$ 5.760,00 21) a) 1536 ladrilhos b) R$ 122,88 22) 9 dm

UNIDADES DE VOLUME E DE CAPACIDADE Metro Cbico

A unidade fundamental para medida de volume o metro cbico. Chama-se metro cbico ao volume de um cubo cuja aresta mede 1 metro. 1m Abrevia-se metro cbico pelo smbolo m3. 1m 1m
Mltiplos Unidade Submltiplo decmetro centmetro milmetro cbico cbico cbico dm3 cm3 mm3

quilmetro hectmetro decmetro metro cbico cbico cbico cbico km3 hm3 dam3 m3

Os mltiplos e submltiplos do metro cbico relacionam-se entre si segundo a tabela acima. 122

MATEMTICA PASSO A PASSO

Onde cada unidade 1000 vezes maior que a unidade imediatamente inferior. Transformaes De Unidades Passo A Passo. 0,0025 km 3 = 2 500 dam3 2) 3421,4 cm3 em m3 3421,4 cm3 = 0,0034214 m3 3) 0,000 0001 hm3 em mm3 0,000 0001 hm 3 = 100 000 000 mm3 Unidades de Capacidade A unidade fundamental para medir capacidade o litro que se abrevia . Define-se: O litro o volume equivalente a um decmetro cbico.

Converter: 1) 0,0025 km3 em dam3

Do mesmo modo que as unidades de medida, anteriores, estabelecem-se os mtiplos e submltiplos do litro que resumiremos no seguinte quadro:
Mltiplos quilolitro hectolitro decalitro 1 000 k 100 h 10 da Unidade litro 1 Submltiplos decilitro centilitro mililitro 0,1 d 0,01 c 0,001 m

Relaes entre as unidades de Volume e de Capacidade

1 litro = 1 dm3 relaciona as unidades de capacidade e volume. Esta relao pode ser resumida no quadro seguinte: 1 hectolitro = 100 = 100 dm3 1 litro 1 mililitro = = 1 0,001 123 = = 1 dm3 1 cm3

A definio de litro, isto :

Transformaes de Unidades Passo a Passo

PROF. WELLINGTON BRITO

Converter:
1) 1,4 hl em L 4) 58 450 dl em dam3 1,4 hl = 140L 58 450 dl = 5 845L. E como:1L = 1dm3 2) 53 825 ml em dal 58 450 dl = 5 845 dm3 53 825 ml = 5,3825 dal E: 5845 dm3 = 0,005845 dam3 3) 22,5 m3 em l Ento: 58 450 dl = 0,005845 dam3 3 3 22,5 m = 22 500 dm .E como: 1 dm3 = 1 l 22,5 m3 = 22 500 l

VOLUMES DOS PRINCIPAIS SLIDOS GEOMTRICOS Volume do Paraleleppedo Seja o paraleleppedo da figura abaixo e digamos que suas medidas sejam: a = 5 cm b = 4 cm c = 6 cm

b a c

O volume de um paraleleppedo retngulo dado pelo produto de suas trs dimenses a, b, c. Ou: V = a. b. c

Onde: V o volume ; a a largura; b a altura; e c a profundidade. Se observarmos a frmula: V = a. b. c., poderemos escrev-la: V = (a.c) . b E como a. c. indica rea da base, diramos: O volume do paraleleppedo retngulo igual ao produto da rea da base pela altura. Assim: V = 5.4. 6 V = 120 cm3 Volume do Cubo 124

MATEMTICA PASSO A PASSO

Como o cubo um paraleleppedo retngulo de arestas iguais, chamando-se essas arestas de a, vem: V=axaxa Ou: O volume do cubo dado pelo ou V = a3 cubo de sua aresta. a a Volume do Cilindro chamando-se de B a rea da base e h altura do cilindro, teremos: V=Bxh E como: B = r2, ento: V = r2 h

h B

Volume do Cone Consideremos um cone e um cilindro cujas bases tm o mesmo raio e cujas alturas sejam iguais. Experimentalmente comprova-se que so necessrios 3 cones de gua, ou areia, ou outra qualquer substncia, para encher o cilindro. Conclui-se que: O volume do cone a tera parte do h volume do cilindro que tem a mesma base e a mesma altura do cone. r r Assim: V = 1 B. h 3
1 V= 3

r2 h

Questes Comentadas
1) Calcular o volume do cone onde a soma entre o raio da base e a altura 36 cm, sendo a altura o triplo do raio. Raciocnio: Como r + h = 36 cm e h = 3r, Vem: r + 3r = 36 cm h Ou: 4r = 36 cm Donde: r = 9 cm e h = 27 cm. r O volume ficar: 125

V = 1 r2 h Ou: V= 1 x 3,14 x 92 x 27 3 3 Donde: V = 2 289,06 cm3 2) Calcular em hl a capacidade de um silo de forma cilndrica, onde o raio da base vale 3 m e a altura 10m. Raciocnio: V = r2 . h V = 3,14 x 3 2 x 10 10 m V = 282,60 m3 3 Como:1 dm = 1 I, vem: V = 282600 dm3 em litros. 3m em hectolitros ficar: V = 2826 hl EXERCCIO UNIDADE DE VOLUME E CAPACIDADE I) Escrever, em metros cbicos, as seguintes grandezas: 1) 0,005 km3 6) 5421,5 dm3 3 2) 4,2 hm 7) 123 125 cm3 3) 3/2 de hm3 8) 1 1 de dam3
4

PROF. WELLINGTON BRITO

( ( 5)

4) 1 + 1 + 0,5
8

41
4

) ) 0,5

dm3 cm3

( 10) (
9)

2
3

31
2

) + 0,2)
+ 0,2

hm3 km3

II) Resolver os seguintes problemas: 1) Determinar qual a capacidade em litros de uma caixa dgua que tem forma de paraleleppedo, cujas arestas medem 0,60m por 0,40 m x 1m. 2) Um cubo de 0,80m de aresta est cheio dgua at seus 3/4. Quantos litros contm? 3) Um tanque de gasolina tem forma cilndrica. O raio da circunferncia da base 1,20m e o comprimento do tanque 6 metros. sabendo-se que se pode colocar o lquido at os 8/9 de sua capacidade pergunta-se: quantos litros cabem no tanque? 4) A altura de um cone 0,60 dm r o raio da base 3/5 dessa altura. Calcular seu volume. 5) Um reservatrio de gua de forma cilndrica est cheio at os seus 5/8.O raio da base 2,5m e a altura 5m. o consumo 126

MATEMTICA PASSO A PASSO

de gua exige 61328,125 decilitros por hora. Com essa gua apenas, pergunta-se durante quantas horas a cidade atendida pelo reservatrio ter gua? 6) A quantos hectolitros corresponde o volume de 152,4 cm 3. 7) Dizer quantos cm3 esto contidos em 1253 dl. 8) Uma indstria farmacutica importa 10 frascos de 5 l cada, de vacinas antiplio pretende revender a vacina em frascos de 20 cm 3. Perguntase: quantos frascos ter para vender? 9) Dois tambores de 200 litros cada, esto cheio de cido que vai ser diludo em gua em partes iguais. Quantas latas de 0,8cm3 sero necessrios para conter o cido diludo? 10) Uma indstria importa vinho estrangeiro em 20 barris de 160 litros cada. Vai engarraf-lo em frascos que contm 0,75 litros cada. Quantas sero as garrafas de vinho? 11) A soma dos volumes de dois cubos 2200 hl e o volume do maior 7/4 do volume do menor. Qual a capacidade de cada um em litros? 12) A diferena dos volumes de dois frascos 280 cl e o menor os 3/11 do maior. Quantos cm 3 contm cada um? Respostas I) II) 1) 5 000 000 m3 1) 240 L 11) 140000 L - 80 000 L 3 2) 4 200 000 m 2) 384 L 12) 3850 cm3 1050 cm3 3) 1 500 000 m3 3) 24 115,2 L 4) 0,001 625 m3 4) 0,081 388 8 dm3 5) 0,000 00375 m3 5) 10 horas 3 6) 5,4215 m 6) 0,001524 hl 7) 0,123 125 m3 7) 125300 cm3 8) 1250 m3 8) 2500 frascos 9) 2 533 333,3 m3 9) 1000 000 latas 10) 3 700 000 000 m3 10) 4266,6 garrafas UNIDADES DE MASSA Introduo O que comumente se chama como peso de um corpo , na realidade matemtica e fsica, a massa do corpo. As balanas usuais medem a massa dos corpos e no do seu peso. O peso de um corpo a resultante da ao da gravidade sobre a 127

PROF. WELLINGTON BRITO

massa desse corpo. Desse modo, enquanto a massa constante, o peso varia conforme o local em que se acha o corpo, porque a ao da gravidade varia de local para local da terra. Estudaremos as unidades de massa, a partir da unidade fundamental que o quilograma. O Quilograma O quilograma a massa de 1dm3 de gua destilada temperatura de 40 C. Indica-se por kg. Para identificar mltiplos e submltiplos das unidades de massa, toma-se como referncia o grama , "massa equivalente a 0,001 do quilograma". Assim teremos:
Mltiplos Uni dade 100 g hg 10 g dag 1g g Submltiplo

quilograma hectograma decagrama grama decigrama centigrama miligrama

1 000 g kg

0,1 g dg

0,01 g cg

0,001 g mg

Existem outros mltiplos especiais do quilograma e um submltiplo, que so: Tonelada (t) = 1000 kg Megatonelada (megaton) = 1000 t ou 1000 000 kg Quilate = 0,2g O quilate serve para medir a massa de pedras e metais preciosos. Mudanas de Unidades - Passo a Passo As converses se fazem facilmente, pois cada unidade de massa do quadro anterior 10 vezes maior que a imediatamente inferior (exceo aos mltiplos especiais). Converter: 1) 57,2 kg em dg 2) 58342,50 cg em hg 57,2 kg = 572000 dg 58342,50 cg = 5,83425hg 2) Quantas toneladas esto contidas em 8 432 738,4 hg? Como: 8 432 738,4 hg = 843 273,84 kg e valendo a tonelada 1000kg. Vem: 8 432 738,4 hg = 843,27384 t. 128

MATEMTICA PASSO A PASSO

EXERCCIO UNIDADE DE MASSA I) Expressar em kg as seguintes grandezas: 1) 4213 g 3) 53,12 cg 2) 3 dg 4) 1 1 hg 4 6 II) Expressar em gramas as seguintes grandezas: 1) 4 kg 3) 7 1 dg 5) 34,5 hg 7) 3 1 cg 5 2 4 2) 3 hg 4) 0,001 kg 6) 831,42 dag 8 8) 4 1 + 0,2 hg 9) 2 1 + 0,125 hg 10) 0,2 + 1

dag

III) Efetuar em gramas as operaes seguintes: 1) 3,5 g + ( 5,6 hg 49 dag) 2) 5,20 dg + [ 85 dag (4,3 kg 40 hg) ] 3) 4,2 kg [ (60 dg + 8,4 g) + (25 dag 240 g)] 4) 125 hg [ 10 dag ( 130g 1120 dg)] 5) 0,09 hg + [0,05 kg + (120 hg 11,2 kg)] Respostas I) 1) 4,213 kg 2) 0,000 075 kg 3) 0,000 5312 kg 4) 0,116 kg

II) 1) 800 g 6) 8 314,2 g III) 1) 73,5 g 2) 37,5 g 7) 0,032 5 g 2) 550, 52 g 3) 0,75 g 8) 453,3 g 3) 4175,6 g 4) 1 g 9) 225 g 4) 12 418 g 5) 3 450 g 10) 13,66 g 5) 859 g

UNIDADES DE TEMPO E NGULO Introduo: Digamos que sejam decorridos, at agora, 9 horas, 10 minutos e 30 segundos do dia. Esse tempo, assim expresso, rene as seguintes unidades de medida: 1) horas; 2) minutos e 3) segundos Essas unidades tm, entre si, as seguintes relaes: Uma hora = 60 minutos; e Um minuto = 60 segundos. uma variao sexagesimal. Isto , no se trata de uma variao decimal. Diz-se ento que o nmero 9 h 10 min 30 seg um nmero complexo, definindo-se de um modo geral: 129

PROF. WELLINGTON BRITO

Chama-se nmero complexo aquele que expressa uma determinada grandeza, em diferentes unidades que no tm entre si relaes decimais. Exemplos: 1) Tempo: 2 dias, 3 horas, 20 minutos e 10 segundos. 2) ngulos: 15 graus e 18 minutos. Daremos a seguir as relaes entre essas unidades. Unidades de Medida de Tempo NOME
Ano comercial Ms comercial Um dia Uma hora Um minuto Um segundo

SMBOLO
a me d ou da h min s ou seg

VALORES
360 dias 30 dias 24 horas 60 minutos 60 segundos do dia mdio solar

1 86400

Unidades de Medida de ngulo. Um ngulo se mede com um transferidor. Um transferidor de meia circunferncia tem 180 graus que se indica 180o. Temos, pois as seguintes unidades de medida de ngulo.

NOME Um ngulo raso Um ngulo reto Um grau Um minuto de grau

= = = =

VALORES E SIMBOLOS 180 graus ou 180 0 90 graus ou 900 60 minutos de grau ou 60 60 segundos de grau ou 60"

Experimentamos um ngulo segundo os exemplos seguintes: = 300 20 42" b = 1350 10 40" ngulo reto = 900 = 890 60 = 890 59 60" ngulo raso = 1800 = 1790 60 = 1790 59 60"

130

MATEMTICA PASSO A PASSO

Questes Comentadas 1) 2 h, 10 min e 20 seg. Deveremos transformar todo o tempo em


segundos, ou: 2 h = 2 x 60 min = 120 minutos + 10 " 130 Mas, 130 min = 130 x 60 seg = 7 800 seg que, somados aos 20 seg, j existentes daro: 2 h, 10 min e 20 seg = 7 820 seg 2)Transformar, em horas, minutos e segundos, o tempo de 24 370 segundos. Como cada 60 seg perfazem 1 minuto, vem: 24 370 60 0 370 406 min 10 seg E como cada 60 minutos perfazem uma hora, teremos: 406 min 60 46 min 6 h ou 24 370 seg = 6 h, 46 min e 10 seg

Adio de Complexos - Passo a Passo Seja efetuar: 3 h 42 min 30 seg 15 h 10 min 40 seg S = 18 h 52 min 70 seg Ora, os 70 seg se reduzem a 1 min e 10 seg, donde: S = 18 h, 53 min, 10 seg

Subtrao de Complexos - Passo a Passo Seja efetuar: 200 15 30" 100 10 40" Como a subtrao impossvel na ordem dos segundos de grau, faz-se o emprstimo de um minuto de grau para aquela ordem: 20 0 14 90" 100 10 40" 10 0 4 50" Para a adio e subtrao de nmeros complexos, conclui-se: 131

PROF. WELLINGTON BRITO

Somam-se ou subtraem-se cada unidade da mesma espcie, reduzindo-se nos resultados os valores incomplexos aos equivalentes complexos. Multiplicao e Diviso de Nmeros Complexos Passo a Passo O produto de nmeros complexos envolve: Multiplicao de inteiro por complexo: Seja efetuar: (15 h 10 min 20 seg ) x 5 = 75 h 50 min 100 seg. Ou: 75 h 51 min 40 seg Multiplica-se o fator inteiro pelas unidades do nmero complexo e reduzem-se os resultados incomplexos aos equivalentes complexos.

Isto :

A diviso de nmeros complexos envolve dois casos: 1) Diviso de complexos por incomplexos: Seja dividir o ngulo de 245 0 18 min 24 seg em 6 partes iguais. ( 2450 18 24" ) : 6 Ou: 2450 18 24" 6 050 300 + 00 400 53 4" x 60 318 24" 300 18 00 Isto : o 1o, resto de 5, transforma-se em minutos () obtendo-se 300 que, somados aos 18` existentes, produzem 318`. Estes 318`so divididos em 6 partes, dando o quociente 53. 2) Diviso de incomplexos por complexos: Para o revestimento e pintura de 50 de um reservatrio de gua de forma circular, foram gastos 15 h 30 min de trabalho. Quantos graus em mdia foram realizados por hora de trabalho? Deve-se efetuar: 132

MATEMTICA PASSO A PASSO

50 : (15 h e 30 min). Como o problema pede "quantos graus por hora", devemos transformar 15 h e 30 minutos em nmero fracionrio: 15 h e 30 min = 15 h + 30 h = 15 h + 1 h = 31 de hora. 60 2 2 Logo: 50 : 31 de h = 50 x 2 = 100 graus por hora. 2 31 31 Ou: 100 31 07 3 13 32" e 28 x 60 31 420 110 17 x 60 1020" 90 28 Em cada hora faz-se o revestimento e a pintura de: 3 13 32" e 28 " do reservatrio.

( )
31

EXERCCIO UNIDADE DE TEMPO E NGULO I) Escrever os seguintes nmeros complexos na unidade pedida: 1) 10 h 20 min 15 seg em segundos 2) 5 10 18" em segundos (") 3) 3 d 20 h 15 min em minutos 4) 118 25 30" em segundos (") 5) 37 38 5" em segundos (") 6) Quantos minutos h num dia? 7) Quantos segundos de ngulo existem em 180 ? 8) Decorreram 7 do dia. Que horas so? 10 9) So decorridos 3 do ano. Quantos meses e dias se passaram? 8 II) Transformar em nmeros complexos as seguintes grandezas: 1) 42 351 seg 2) 35 400" 3) 75 358 seg 4) 25 001 min 5) 2 535 " 6) 2 437 min 133

PROF. WELLINGTON BRITO

III) Efetuar as seguintes operaes: 1) 2) 3) 4) 5) 6) 7) 8) 9) 10) 11) 12) 13) 14) 20 h 40 min 16 seg + 10 h 12 min 52 seg 30 15 42" + 20 50 20" 5 d 22 h 40 min + 2 d 20 h 30 min 12 h 15 min 30 seg 10 h 20 min 40 seg 25 8 10" 10 15 30" 3 d 20 h 10 min 20 seg 22 h 20 min 20 seg ( 4 h 10 min 20 seg ) x 5 ( 3 40 30" ) x 6 (20 h 40 min 12 seg) x 3 5 (30 15 40" ) x 3 4 (15 h 20 min 48 seg) : 6 ( 42 10 15" ) : 12 Dividir o tempo de 17 horas em 5 partes iguais. Dividir o ngulo de 32 em 6 partes iguais.

Respostas
I) II) 1) 11h 45min e 51 seg 2) 9 50 3) 20h 55min e 58 seg 4) 17 d 8h e 41 min 5) 42 15" 6) 40 h e 37 min

1) 37 215 seg 2) 18 618" 3) 5 535 min 4) 426 330 seg 5) 135 485 " 6) 1 440 min 7) 648 000 "
8) 9) III) 1) 1d 6h 53 min e 8 seg 2) 51 6 2" 3) 8d 19 h e 10 min 4) 1 h 54 min e 50 seg 16 h e 48 min 4 m e 15 d

134

MATEMTICA PASSO A PASSO

5) 14 52 40" 6) 2 d 21 h 50 min 7) 20 h 51min 40seg 8) 22 3 9) 12 h 24 min e 7 1 seg 5 10) 22 41 45" 11) 2 h 33 min e 28 seg 12) 3 30 5 1 1 " 4 13) 3 h e 24 min 14) 5 20

QUESTES DE CONCURSOS E VESTIBULARES 1) (INFRAERO) A dose diria recomendada de um remdio lquido de 40 gotas. Uma gota desse medicamento pesa em mdia, 5 x 10-2 gramas. Ento, num frasco contendo 80 gramas desse remdio, temos medicamento suficiente para um tratamento de no mximo: a) 40 dias b) 35 dias 2) c) 30 dias d) 20 dias e) 25 dias

(TJ) Quantos cm3 existem em 10 litros? a) 10 b)100 c) 1000 d)10.000 e)100.000

3)

(TJ) Se uma soluo contm 2mg/ml de uma substncia dissolvida, quanto da substncia existe em um litro de soluo? a) 200 mg b) 2g c) 20g d) 200g e) 2kg

4)

(ECT) Contornou-se com 319 palmeiras, plantadas a mesma distncia, uma da outra, um terreno retangular com 380 dm de 135

PROF. WELLINGTON BRITO

frente por 60 dam de fundo. De quantos em quantos metros foram plantadas as palmeiras? a) de 2m em 2m b) de 7m em 7m 5) c) de 5m em 5m d) de 4m em 4m e) de 6m em 6m

(UECE) Duas caixas dgua, a primeira em forma de um paraleleppedo e a segunda em forma cbica, possuem as seguintes dimenses: Base 6m por 40dm e altura 0,2 dam, a primeira; Aresta de 200cm, a segunda. O volume da segunda caixa dgua, comparado com o volume da primeira : a) a metade b) um tero c) um sexto d) um oitavo (UFC) A capacidade, em litros, de uma caixa de formato cbico que tem 50 centmetros de aresta de: a) 625 b) 500 c) 375 d) 250 e) 125

6)

7)

(UFC) Uma piscina na forma de um paraleleppedo retngulo de 9m de comprimento, 4m de largura e 2m de altura, est sendo abastecida de gua razo constante de 50 litros por minuto. O tempo necessrio, em horas, para encher esta piscina, sem desperdcio de gua, : a) 26 b) 24 c)22 d) 20 e) 18

8)

(BB) Quantos labirintos de 0,2m x 0,2m so precisos para revestimento de uma sala de 5m de comprimento por 6m de largura? a) 600 b) 650 c) 700 d) 750 e) 800

9)

(TRT) A velocidade de 180 km/h eqivale a quantos metros por segundo? a) 5 b) 30 c) 50 d) 300 e) 500

136

MATEMTICA PASSO A PASSO

10) (UECE) Quando um relgio de ponteiros, marca exatamente 3h 30min, a medida do ngulo menor entre os ponteiros (das horas e dos minutos) : a) 850 Respostas 1) A 03)B 05)C 06)E 07)B 08)D 09)C 10)C b) 800 c) 750 d) 700

02) D 04)D

RAZO E PROPORO Razes - Passo a Passo 1. Razo de dois nmeros Razo do nmero a para o nmero b (diferente de zero) o quociente de a por b. a ou a : b (lemos: a para b ) b Os nmeros a e b so os termos da razo; a chamado antecedente e b, conseqente da razo. Exemplos: 1. A razo de 3 para 12 : 3 = 1 12 4 2. A razo de 20 para 5 : 20 = 4 5 3. A razo entre 5 e 1 : 137 5 = 5x 2 = 10 Indicamos:

PROF. WELLINGTON BRITO

2 2. Razo de duas grandezas

1 2

Razo de duas grandezas, dadas em uma certa ordem, a razo entre a medida da primeira grandeza e a medida da segunda. Se as grandezas so da mesma espcie, suas medidas devem ser expressas na mesma unidade. Neste caso, a razo um nmero puro. Exemplos: 1. A razo de 2m para 3m : 2m = 2 3m 3

1 2. A razo de 30 dm para 6m : 30 dm = 3 m = 1 6m 6m 2 2 Observao: Se as grandezas no so da mesma espcie, a razo um nmero cuja unidade depende das unidades das grandezas a partir das quais se determina a razo.

Exemplo: Um automvel percorre 160 km em 2 horas. A razo entre a distncia percorrida e o tempo gasto em percorr-la : 160 km = 160 km/h = 80 km/h 2h 2 Podemos dizer, ento, que esse automvel faz em mdia 80 km em 1 hora ou 80 km/h. Exerccio Razo 1. Calcule a razo entre os nmeros: a) 256 e 960 c) 5 e 1 e) 2 - 1 e 3 3 5 b) 1,25 e 3,75 d) 1 e 0,2 2 2. Calcule a razo entre as seguintes grandezas: a) 27 km e 3L de lcool d) 20 cm e 4dm 3 b) 40 g e 5 cm e) 20 d e 2 me 15 d

( )

138

MATEMTICA PASSO A PASSO

c) 24 kg e 80 kg Propores - Passo a Passo Definio Dados quatro nmeros (15,3,20 e 4), como a razo entre os dois primeiros nmeros(15 e 3) igual razo entre os dois ltimos (20 e 4), isto : 15 = 5 3 e 20 = 5, 4

dizemos que os nmeros 15, 3, 20 e 4, nesta ordem, forma uma proporo, que expressamos mediante a igualdade das duas razes: 15 3 = 20 4

Assim: Dados, em uma certa ordem, quatro nmeros (a, b, c e d) diferentes de zero, dizemos que eles formam uma proporo quando a razo entre os dois primeiros (a e b) igual razo entre os dois ltimos (c e d).

Simbolicamente, representamos uma proporo por: a = c b d e lemos: "a est para b, assim com c est para d". Essas anotaes pem em evidncia o fato de que uma proporo uma igualdade entre duas razes. Exemplos: 1. 18 27, 18 27 = pois =3 e =3 6 9 6 9 9 9 3 2 2 2 2 1 2 3 2 9 4 2. = ,pois = 2: = x =6e = x = 6 1 3 1 3 1 3 2 3 1 1 3 4 3 4 139

PROF. WELLINGTON BRITO

Elementos Na proporo: a c b d temos: a, b, c e d so termos ( 1, 2, 3 e 4 termos, respectivamente) a e c so os antecedentes b e d so os conseqentes a e d so os extremos b e c so os meios Propriedade fundamental Sejam a, b, c e d nmeros reais diferentes de zero, tais que: a c b d Em toda proporo, o produto dos extremos igual ao produto dos meios.

Questes Comentadas
1) Verifique se so ou no verdadeiras as seguintes propores: a) 6 7 24 28 b) 2 3 12 15 6 x 28 = 7 x 24 2 x 15 3 x 12

Resoluo: a) Temos: 6 x 28 = 168 e 7 x 24 = 168 logo, verdadeira. b) Temos: 2 x 15 = 30 e 3 x 12 = 36 logo, falsa

2) Comprove se os nmeros 3, 7, 15 e 35, no obrigatoriamente nesta ordem, formam uma proporo e, em caso afirmativo, escreva-a. Resoluo: Temos: 3 x 35 = 105 e 7 x15 = 105 140 3 x 35 = 7 x 15

MATEMTICA PASSO A PASSO

Logo:

3 7

15 35

3) Calcule x nas propores:


a) 15 20 60 x b) 7 6 x 5 3 2

Resoluo: a) Temos, aplicando a propriedade fundamental: 4 15 X x = 20 x 60 x = 20 x 60 x = 80 15 Logo: x = 80 1

b) Temos:

1 5x = 7 4

7 x=4 5 x=7x1 4 5 x= 7 20

5Xx=7x3 622 Logo: x = 7 20

Razes Iguais Passo a Passo Considerando as razes: 6 , 10 , 12 , 8 3 5 6 4 vemos que todas so iguais a 2. Logo, podemos escrever: 6 10 12 8 3 5 6 4 Essa expresso denominada srie de razes iguais ou proporo mltipla. Em smbolos: a c ... m b d n Propriedade

Em uma srie de razes iguais, a soma dos antecedentes est para a soma dos conseqentes assim como qualquer antecedente est para o seu respectivo conseqente. 141

PROF. WELLINGTON BRITO

Exemplo: 6 10 3 5

12 6

8 4

6+10+12+8 3+5+6+4

6 ou 10 ou 12 ou 8 3 5 6 4

Questes Comentadas
1. Calcule x, y e z, sabendo que x = y = z e x + y + z = 420. 9 11 15 Resoluo: Temos, pela propriedade fundamental da srie de razes iguais: x+y+z = x ou y ou z 9 + 11 + 15 9 11 15 Como: x + y + z = 420, podemos escrever: 12 420 = x ou y ou z Da: 420 = x x = 420 x 9 = 108 35 9 11 15 35 9 35 1 12 420 = y y = 420 x 11 = 132 35 11 35 1
12

420 = z 35 15

z = 420 x 15 = 180 35 1

Logo: x = 108 , y = 132 e z = 180 2. Determine os antecedentes de uma proporo, sabendo que sua soma 47 e que os conseqentes so 2 e 8. Resoluo: Temos, chamando de x e y os antecedentes: x y = e x + y = 47 2 8 Pela propriedade fundamental da srie de razes iguais, podemos escrever: x + y = x ou y 2 + 8 2 8 como : x + y = 47 , vem : 47 = x ou y 10 2 8 Da : 47 = x x = 47 x 2 = 94 = 9,4 10 2 10 10 47 = y 10 8 y = 47 x 8 = 376 = 37,6 10 10 142

MATEMTICA PASSO A PASSO

9,4 + 37,6 = 47,0 Logo, os antecedentes so 9,4 e 37,6, respectivamente.

E XERCCIO RAZO E PROPORO 1) Determine a razo entre os nmeros: a) 226 e 1.017 b) 1,25 e 0,75
c) 12 e 9 30 12 d)

2 + 3 5 4

) (
e

15 4

2 5 8

2) Calcule a razo entre as seguintes grandezas: a) 80 m e 48 dam b) 150 m2 e 45 ares 3 c) 0,725 m e 5.000 L d) 9d 17h 20min e 8d 12h 10min

3) Verifique se a razo de 6 me 20 d para 3a 5me 20 d igual


razo de 640 L para 2 m3. 4) Verifique se as seguintes expresses formam proporo: 3 a) 4 5 8 5 2 25 12 1 3 1 2 1 3 1 5

- 2

b) 0,01 = 20 0,1 200

c)

= 1 25

5) Calcule o valor de x na proporo:


a) 2 3 x 7 5 4 5 7 2 1 2 2 3 1 2 1 1 143 e) 1 + 1 4 3 2 2 3 5 x 0,3 - 1 2 + x x 3 1 7 3 x 1 +1 4 2

b) x 5 1 7 c) x - 5 4

f)

1 - 0,5 6 1- 2 3

8 g) 3

PROF. WELLINGTON BRITO

d) 0,1 ( 1 0,1 ) 0,1 1 x 0,4

4 x

h) 2 x

2 2 +1 4

)
6

6) Escreva uma razo igual a 15, cujo antecedente seja 5 . 4 3

7) Escreva uma razo igual a 1, cujo conseqente seja 4 1


5 conseqentes sejam 28 e 36. 4 c) 1 e x+y= 6 x = y 8 3 e x y = 85

8) Escreva uma proporo cujas razes sejam iguais a 1 e cujos 9) Calcule x e y , sabendo que:
a) x 5 b) 1 2 x y e x + y = 187 12 1 3 y

10) Calcule dois nmeros, sabendo que sua soma 169 e que a razo 4 9

11) Dois nmeros, cuja soma 28, guardam entre si a relao 3.


Quais so esses nmeros? so esses nmeros? 5 4

12) Dois nmeros, cuja diferena 12, esto na relao 8. Quais 13) A idade de um pai est para a de seu filho como 7 est para 5.
Se a soma das idades 52, qual a idade de cada um? 3

14) Decomponha o nmero 35 em duas partes, tais que a razo


entre elas seja 3. 6 2 15) Qual o nmero que,aumentado de 2 unidades,est para 5 assim como 28 est para 20? 16) Qual o nmero que, diminudo de 3 unidades, est para o seu consecutivo assim como 5 est para 6? 17) A soma de trs nmeros igual a 555. O primeiro est para o segundo como 8 est para 5. A diferena entre esses dois nmeros igual a 69. Quais so os trs nmeros?

144

MATEMTICA PASSO A PASSO

18) A importncia de R$ 588 foi dividida entre trs pessoas. Sabendo que a parte da primeira est para a da segunda como 5 para 7, e que a parte da segunda est para a da terceira como 7 para 9, determine as trs partes. Respostas 1. a) 2 b) 5 c) 8 d) 46 9 3 15 45 2. a) 1 6 3. No 4. a) sim b) sim c) no 5. a) 8 c) 5 21 3 b) 4 d) 2,5 6. 5 3 4 9 7. 5 6 4 1 6 8. 7 9 e) 105 64 f) 3 5 g) 2 h) + 3 2 10. 52 e 117 11. 12 e 16 12. 32 e 20 13. 42 anos e 10 anos 14. 15. 5 16. 23 17. 184, 115 e 256 18.R$ 140, R$ 196 e R$ 252 7 e 7 2 3 b) 1 c) 29 d) 8 30 200 7 09. a) x = 55 e y = 132 b) x = 1 e y = 1 10 15 c) x = 136 e y = 51

28

36
GRANDEZAS PROPORCIONAIS

Introduo

A maioria dos problemas que se apresentam em nosso dia-a-dia liga duas grandezas relacionadas de tal forma que, quando uma delas varia, como conseqncia varia tambm a outra. Assim, a quantidade de combustvel gasto por um automvel depende do nmero de quilmetros percorridos. O tempo gasto numa construo depende do nmero de operrios empregados. A relao entre duas grandezas variveis estabelece a lei de variao dos valores de uma delas em relao outra. Segundo tal lei, as grandezas relacionadas podem ser direta ou indiretamente proporcionais. 145

PROF. WELLINGTON BRITO

Grandezas Diretamente Proporcionais Definio

Uma barra de alumnio de 100 cm3 de volume pesa 270 g; nas mesmas condies, uma barra de 200 cm 3 pesar 540 g e uma de 30 cm3, 810 g. podemos, ento, escrever a seguinte tabela: Volume (cm3) 100 200 300 500 Massa (g) 270 540 810 1.350

Examinando a tabela, vemos que a grandeza massa depende da grandeza volume, j que aumentando uma (volume), a outra (massa) tambm aumenta. Propriedade Fundamental Sendo (x1, y1) e (x2, y2) pares de valores correspondentes de duas grandezas proporcionais, podemos escrever: y2 y1 x2 x1 Alternando os extremos, obtemos: x1 y1 x2 y2 que nos d a propriedade caracterstica das grandezas diretamente proporcionais: Dadas duas grandezas diretamente proporcionais, a razo entre dois valores de uma delas igual razo entre os dois valores correspondentes da outra. Questes Comentadas 1) Verifique se so diretamente proporcionais as seqncias de nmeros (6,9,12, 15) e (2, 3, 4, 5). Resoluo: Temos: 6 = 9 = 12 = 15 = 3 2 3 4 5 Logo, essas seqncias de nmeros so diretamente proporcionais e a razo de proporcionalidade 3. 2) Os nmeros das seqncias (6,9,20) e (2,3,6) so proporcionais? Resoluo: Temos: 6 = 9 20 Logo, esses nmeros no so 2 3 6 proporcionais 146

MATEMTICA PASSO A PASSO

3) Sendo x e y grandezas diretamente proporcionais, calcule os valores de a e b: x y 7 9 b 21 a 39

Resoluo: Sendo k a razo de proporcionalidade, temos: 21 k = =3 7 Logo: a = 3 a=9x3 a = 27 9 39 = 3 b x 3 = 39 3b = 39 b = 39 b 3 Assim: a = 27 e b = 13

b = 13

4) Quais os menores nmeros inteiros proporcionais aos nmeros 2 , 3 e 1 ? 3 4 6 Resoluo Vamos multiplicar cada um dos nmeros dados pelo menor mltiplo comum dos denominadores. Como o m.m.c. (3,4,6) =12, temos:
4 3 2

2 x 12 = 8 , 3 x 12 = 9 , 1 x 12 = 2 31 41 61 Logo, os nmeros pedidos so 8,9 e 2 EXERCCIO GRANDEZAS DIRETAMENTE PROPORCIONAIS 1) O nmero de dias gastos na construo de um muro diretamente proporcional ao nmero de operrios empregados nesse servio? Por qu? 2) Verifique se so ou no proporcionais os nmeros das seqncias: a) (40, 38, 35) e (8, 7, 5) b) (5, 6, 7) e (75, 90, 105) 3) Qual a razo de proporcionalidade entre as seqncias de nmeros diretamente proporcionais ( 5, 8, 11) e (40, 64, 88)? 4) Determine os valores de a e b nas seqncias de nmeros proporcionais (6,a,21) e (2, 5, b). 5) Dados os nmeros 1 , 3 e 7 , determine os trs menores 147

PROF. WELLINGTON BRITO

5 6 10 nmeros inteiros proporcionais a esses nmeros. Respostas 1) No. Aumentando o nmero de operrios, o nmero de dias gastos na construo, diminuir. 2) a) No b) sim 3) k = 8 4) a = 15 e b = 7 5) 6,15 e 21 GRANDEZAS INVERSAMENTE PROPORCIONAIS

Definio Uma distncia de 1.200 km pode ser percorrida por um avio, a uma velocidade de 100 km/h, em 12 horas; a uma velocidade de 200 km/h, em 6 horas; e a uma velocidade de 300 km/h, em 4 horas. Podemos, ento, escrever a tabela: Velocidade (km/h) Tempo (h) 100 12 200 6 300 4 400 3

Vemos que, tambm aqui, a grandeza tempo depende da grandeza velocidade, j que aumentando a velocidade o tempo diminui.

Propriedade Fundamental

Sendo (x1, y1) e (x2, y2) partes de valores correspondentes de duas grandezas inversamente proporcionais, podemos escrever: x1, y1 = x2, y2 ou: x1 y2 x2 y1 caracterstica das

que nos d a propriedade inversamente proporcionais:

grandezas

Dadas duas grandezas inversamente proporcionais, a razo entre dois valores de uma delas igual ao inverso da razo entre os dois valores correspondentes da outra.

Questes Comentadas
148

MATEMTICA PASSO A PASSO

1) Verifique se so ou no inversamente proporcionais as seqncias de nmeros: a) (2,3,6,10) e (45,30,15,9) b) (2,5,8) e (40,30,20) Resoluo: a) Temos: 2 x 45 = 3 x 30 = 6 x 15 = 10 x 9 = 90 Logo, so inversamente proporcionais e o fator de proporcionalidade 90. b) Temos: 2 x 40 5 x 30 Logo, no so inversamente proporcionais. 2) Determine os valores de a e b nas seqncias de nmeros inversamente proporcionais ( 2, 3, b) e ( 15, a, 5). Resoluo: Temos: Da: k = 2 x 15 3a = 30 5b = 30 k = 30 a = 30 3 a = 10 b= 6

b = 30 5 Logo: a = 10 e b = 6

EXERCCIO GRANDEZAS PROPORCIONAIS


1) 2) D um exemplo de grandeza inversamente proporcionais. Verifique se so ou no inversamente proporcionais as seqncias de nmeros: a) (20,12,10) e (6,10,12) 3) 4) 5) b) (1,2,5 ) e (4,8,20) Qual o fator de proporcionalidade entre as seqncias de nmeros inversamente proporcionais ( 1,3,5) e (60,20,12)? Sabendo que os nmeros das seqncias (1, a, -4) e (4,2,b) so inversamente proporcionais, determine a e b. O produto de dois nmeros direta ou inversamente proporcional a cada um de seus fatores? Por qu?

149

PROF. WELLINGTON BRITO

6) 7)

O quociente direta ou inversamente proporcional ao divisor? Por qu? Diga se so direta ou inversamente proporcionais as seguintes grandezas: a) quantidade de metros de arame e preo. b) Velocidade e tempo c) Tempo e nmero de operrios empregados para um determinado servio d) Salrio e nmero de horas de trabalho: e) Quantidade de alimento e nmero de pessoas a serem alimentadas D exemplo de: a) grandezas diretamente proporcionais; proporcionais. b) grandezas inversamente

8)

9)

Verifique se so ou no proporcionais as seguintes sucesses de nmeros; em caso afirmativo, determine o coeficiente de proporcionalidade: a) 120, 180 e 375 48, 72 e 150 b) 0,24; 0,21 e 0,15 0,8; 0,7 e 0,05

10) Verifique se so ou no inversamente proporcionais as sucesses de nmeros a seguir; em caso afirmativo, determine o coeficiente de proporcionalidade: a) 90,60 e 45 28,42 e 56 b) 0,45; 0,12 e 0,035 10,5; 2,8 e 36

11) Determine o fator de proporcionalidade entre as seguintes sucesses de nmeros proporcionais:

a)

4, 16 e 20 12, 48 e 60

b) 1 e 22 ; 2

5 6

e 36

12) Determine o coeficiente de proporcionalidade entre as seguintes sucesses de nmeros inversamente proporcionais: a) 6,10 e 5 20, 12 e 24 b) 2 , 4 e 7 3 5 8 ; 42, 35 e 32

13) Determine os valores de x e y e z nos seguintes grupos de


nmeros diretamente proporcionais: a) x y 0,7 2 5 2 b) x, 0,3, 2 , 5 3 7 ; 9, 3 , y , z 5

14) Determine os valores de m e n e p nos seguintes grupos de


nmeros inversamente proporcionais: a) 5 , n , p , 7 b) 8 , 3
4

, p,

150

MATEMTICA PASSO A PASSO

m , 4, 14 , 8 15)

5 m , n , 9,1

Determine os quatro menores nmeros inteiros proporcionais aos nmeros: 5 e 8 b) 0,5; 2,37; 0,8 e 3,4 4 5 menores nmeros inteiros inversamente proporcionais aos 3,4,5 e 8? b) No 10) a) sim, k = 2520 11) a) k = 1/3 b) No

a) 3 , 2 , 8 3 16) Quais os nmeros Respostas: 2) 2) a) Sim

3) k = 60 4) a = 2 e b = - 1 5) Diretamente
6) Inversamente d) Direta e) Direta b) Inversa

b) k = 3/5

12) a) k = 120 b) k = 28 13) a) x = 0,7 e y 1,75 b) x = 9/2, y = 4/3 e z = 10/7 14) a) m = 11,2 ; n = 14 e p = 4 b) m = 9/8, n = 45/19 e p = 1 15) a) 45, 80, 150, 192 b) 50, 237, 80, 340 16) 40,30,24,15

7) a) Direta b) Inversa

9) a) sim, k = 5/2 b) no

DIVISO PROPORCIONAL REGRA DE SOCIEDADE Diviso Proporcional Diviso em partes diretamente proporcionais Suponhamos que voc queira dividir o nmero 180 em partes diretamente proporcionais a 2, 5 e 11. Isso significa dividir o nmero 180 em trs parcelas, tais que a razo da primeira parcela para o nmero 2 seja igual razo da segunda parcela para o nmero 5 e igual razo da terceira parcela para o nmero 11. Assim, chamando de x, y e z, respectivamente, cada uma dessas parcelas, devemos verificar que: 151

PROF. WELLINGTON BRITO

1 2 5 11 Alm disso, com x, y e z so as parcelas em que dividimos o nmero 180, devemos ter: x + y + z = 180 Como 1 uma srie de razes iguais, podemos escrever, pela propriedade:
x + y + z = x = y = z 2 + 5 + 11 2 5 11 180 = x = y = z 18 2 5 11 180 = 10 temos: 18 x = y = z = 10 2 5 11 x = 10 x = 2 x 10 = 20 2 y = 10 y = 5 x 10 = 50 5 z = 10 z = 11 x 10 = 110 11

ou: Como:

Da

Sendo 20 + 50 + 110 = 180, conclumos que as partes procuradas so: 20, 50 e 110. Observao: Por conveno, chamamos, simplesmente, de diviso proporcional a diviso diretamente proporcional.

Questes comentadas
1. Divida o nmero 70 em partes proporcionais aos nmeros 2, 3 e 5. Resoluo: Indicando as partes por x, y e z, devemos ter: x 2 x = y = z 70 y 3 sendo 2 3 5 z 5 x + y + z = 70 10 Como: = 70 Vem: x = 2 x 7 = 14 k=7 k 152

MATEMTICA PASSO A PASSO

y = 3 x 7 = 21 z = 5 x 7 = 35 70 Logo, as partes procuradas so: 14, 21 e 35 2. Divida 184 em partes diretamente proporcionais a 1, 2 e 3 2 3 4 Resoluo De acordo com a propriedade dos nmeros proporcionais, multiplicando todos os nmeros da seqncia 1 , 2 e 3 pelo 2 3 4 m.m.c. dos conseqentes (12), obtemos uma seqncia de nmeros inteiros que mantm a proporcionalidade e facilita os clculos: 6 4 3 1 x 12 = 6, 2 x 12 = 8, 3 x 12 = 9 2 3 4 1 1 1 Resulta, ento: x 6 x = y = z 184 y 8 sendo 6 8 9 z 9 x + y +z = 184 23 Como : k = 184k = 8 23 vem: x = 6 x 8 = 48 y = 8 x 8 = 64 z = 9 x 8 = 72 184

10

Logo, podemos afirmar que as partes so: 48, 64 e 72

Diviso em partes inversamente proporcionais

Suponhamos, agora, que voc queira dividir o nmero 210 em partes inversamente proporcionais a 3, 5 e 6. Isso significa dividir o nmero 210 proporcionalmente aos inversos dos nmeros 3, 5 e 6, isto , determinar parcelas x, y e z, tais que: x = y = z 1 1 1 3 5 6 Como o m.m.c. ( 3, 5, 6 ) = 30, temos: 10 6 5 153

PROF. WELLINGTON BRITO

1 x 30 = 10 3 1 Logo: x 210 y z Como: Vem:

1 x 30 = 6, 1 x 30 = 5 5 6 1 1 10 x = y = z 6 sendo 10 6 5 5 x + y + z = 210 21

210 k = 10 21 x = 10 x 10 = 100 y = 6 x 10 = 60 z = 5 x 10 = 50 210

k=

Logo, as partes procuradas so: 100, 60 e 50

Diviso Proporcional Composta Neste caso, o problema consiste em dividir um nmero em partes direta ou inversamente proporcionais a certos nmeros a, b, c e, simultaneamente, em partes direta ou inversamente proporcionais a outros tantos nmeros a, b, c. Tomando por base o que vimos sobre grandezas proporcionais a vrias outras, podemos achar o processo de resoluo do problema. Consideremos, para efeito de raciocnio, o caso da diviso da grandeza de valor n em partes proporcionais aos nmeros a, b, c e tambm aos nmeros a, b, c, respectivamente. Sejam x, y, x, os valores das partes pedidas. Como x, y, z so proporcionais a a, b, c, e tambm a a, b, c, so grandezas compostas; portanto, so proporcionais, respectivamente, aos produtos aa, bb, cc.

Questes Comentadas
1. Divida 392 em partes ao mesmo tempo diretamente proporcionais a 2, 3, 4 e a 3,5,7. Resoluo Temos: 2 x 3 = 6, 3 x 5 = 15, 4 x 7 = 28 x 6 x = 6 x 8 = 48 392 y 15 k = 392 = 8 y = 15 x 8 = 120 z 28 49 z = 28 x 8 = 224 49 392 154

MATEMTICA PASSO A PASSO

Logo, as partes so: 48, 120 e 224 2. Divida 175 em partes diretamente proporcionais a 5 , 3, 4 e, 4 ao mesmo tempo, inversamente proporcionais a 3 , 6 , 2. 4 Resoluo:
1 1 2

Temos: 5 x 4 = 5 , 3 x 1 = 1 , 4 3 3 6 2
1 2

4 x 1 =2 2
1

Da: 175

x y z

5 1 2 2

10 3 3

x = 10 x 7 = 70 k = 175 = 7 25 y = 3 x 7 = 21 x = 12 x 7 = 84 175

12 25 Logo as partes so: 70, 21 e 84.

3. Divida 363 em trs partes, de modo que a Segunda seja o dobro da primeira e a terceira o qudruplo da segunda. Resoluo: Considerando a primeira parte proporcional a 1, temos: 1 1 2 2x1=2 3 4x2=8 Como o problema resulta em dividir 363 em partes diretamente proporcionais a 1, 2 a 8, temos: x 1 x = 1 x 33 = 33
2 k = 363 = 33 8 11 11 Logo as partes so: 33, 66 e 264 363 y z y = 2 x 33 = 66 z = 8 x 33 = 264 363

EXERCCIO- DIVISO PROPORCIONAL 1) Divida o nmero 2.990 em partes proporcionais a 5, 7 e 11. 2) Divida 183 em partes proporcionais a 1 , 1 e 3 4 1 7

3) Dois operrios contratam um servio por R$ 180. Como devem repartir essa quantia, se um trabalhou 7 horas e o outro 8 horas, sendo a diviso diretamente proporcional ao tempo de servio? 4) Divida o nmero 260 em partes inversamente proporcionais aos nmeros 2, 3 e 4. 155

PROF. WELLINGTON BRITO

5) Um pai deixou R$ 2.870,00 para serem divididos entre seus trs filhos na razo inversa das suas idades: 8, 12 e 28 anos. Quanto recebeu cada um? 6) Divida o nmero 2.190 em trs partes que sejam, ao mesmo tempo, diretamente proporcionais a 2, 3, 5 e a 6, 7, 8. 7) Divida 6.050 em trs partes que sejam, a um tempo, inversamente proporcionais a 3, 5 e 6 e diretamente proporcionais a 4, 6 e 9. 8) Divida 292 em trs partes ao mesmo tempo inversamente proporcionais a 3,5 e 6 e a 4,6,9. Respostas 1) 650, 910 e 1.430 2) 84, 63 e 36 3) R$ 84 e R$ 96 4) 120, 80 e 60 5) R$ 1.470, R$ 980 e R$ 420 6) 360, 630 e 1200 7) 2000, 1.800 e 2.250 8) 180, 72 e 40

Introduo

REGRA DE SOCIEDADE

A regra de sociedade uma das aplicaes da diviso proporcional. Tem por objeto a diviso dos lucros ou dos prejuzos entre as pessoas (scios) que formam uma sociedade, por ocasio do Balano geral exigido anualmente por lei ou quando da sada de um dos scios ou da admisso de um novo scio. Por conveno, o lucro ou o prejuzo dividido pelos scios proporcionalmente aos capitais que empregaram, levando-se em conta as condies estipuladas no contrato social. 156

MATEMTICA PASSO A PASSO

Regra de Sociedade - Passo a Passo Classicamente, h quatro casos a considerar:

1) Os capitais so iguais e empregados durante o mesmo tempo. A fim de obtermos a parte de cada scio, dividimos o lucro ou o prejuzo pelo nmero deles. Exemplo: Trs scios obtiveram um lucro de R$ 222.600. Sabendo que seus capitais eram iguais, vamos determinar a parte de cada um nos lucros: 222.600 = 74.200 3 Logo, a parte de cada um no lucro de: R$ 74.200 2) Os capitais so desiguais e empregados durante o mesmo tempo.
Neste caso, dividimos o lucro ou prejuzo em partes diretamente proporcionais aos capitais dos scios. Exemplo: Por ocasio do Balano anual de uma firma comercial formada por trs scios, verificou-se um prejuzo de R$ 27.000. Vamos determinar a parte correspondente a cada scio, sabendo que seus capitais so de R$ 540.000, R$ 450.000 e R$ 360.000: x 540 x = 540 x 0,02 = 10,8 27 y 450 k = 27 = 0,02 y = 450 x 0,02 = 9,0 z 360 1.350 z = 360 x 0,02 = 7,2 1350 27,0 Logo, o prejuzo corresponde a cada scio , respectivamente, de: R$ 10.800, R$ 9.000 e R$ 7.200

3) Os capitais so iguais e empregados durante tempos desiguais. Teoricamente, o lucro ou o prejuzo correspondente a cada scio seria determinado dividindo-se o lucro ou o prejuzo da sociedade em partes diretamente proporcionais aos tempos. Porm, na prtica este caso no ocorre, porque, em uma sociedade, os scios no podem permanecer por tempos desiguais. No momento em que um antigo scio se retira ou um novo scio admitido, procede-se a uma reforma do contrato social, aps o Balano, calculando-se o Ativo e o Passivo. 157

PROF. WELLINGTON BRITO

4) Os capitais so desiguais e empregados durante tempos tambm desiguais. Teoricamente, as partes do lucro ou do prejuzo seriam diretamente proporcionais aos produtos dos capitais pelos respectivos tempos. Tambm neste caso vale a observao feita para o caso anterior. Observao: No devemos confundir este caso com aquele em que os scios integralizam suas quotas de capital em pocas diferentes. Questo Comentada
Antonio e Jos organizaram uma firma comercial com um capital social de R$ 2.000.000, devendo cada um deles entrar com R$ 1.000.000. No ato da organizao, 1 de maro, Antonio integralizou sua quota e Jos contribuiu com apenas R$ 700.000, responsabilizando-se por integralizar sua quota aps 5 meses. Em 31 de dezembro foi procedido o Balano, tendo sido apurado um lucro de R$ 740.000. Qual a parte a ser creditada a cada scio? Resoluo: Antonio, tendo integralizado seu capital de R$ 1.000.000 em 1 de maro, ter um lucro diretamente proporcional a esse capital durante os 10 meses (1 de maro a 31 de dezembro), isto , diretamente proporcional a 1.000.000 x 10 ou 10.000.000. Jos, tendo completado seu capital em 1 de agosto, ter uma parte do seu lucro correspondente a R$ 700.000 durante 10 meses (1 de maro a 31 de dezembro) e outra relativa aos restantes R$ 300.000 durante 5 meses (1 de agosto a 31 de dezembro); a primeira diretamente proporcional a 700.000 x 10 ou 7.000.000 e a segunda, a 300.000 x 5 ou 1.500.000. Assim, seu lucro diretamente proporcional a 7.000.000 + 1.500.000 = 8.500.000. Temos,ento: x 100 x = 100 x 0,04 = 4,0 7,4 y 85 k = 7,4 k = 0,04 y = 85 x 0,04 = 3,4 185 185 7,4 Logo, a Antonio devem ser creditados R$ 400.000 e a Jos, R$ 340.000.

EXERCCIODIVISO PROPORCIONAL/REGRA DE SOCIEDADE 1) Divida o nmero 870 em partes diretamente proporcionais aos nmeros 7, 10 e 12. 2) Divida 3.751 em partes diretamente proporcionais a 7 , 5 e 3 4 8 2 3) Divida o nmero 325 em partes diretamente proporcionais aos nmeros 0,4; 1,2 e 3,4. 4) Divida o nmero 870 em partes inversamente proporcionais aos nmeros 3, 5 e 9. 158

MATEMTICA PASSO A PASSO

5) Divida o nmero 3.161 em partes inversamente proporcionais a 2 , 4 e 7 . 3 5 8 6) Decomponha 760 em partes inversamente proporcionais a 0,4; 3,2 e 6,4. 7) Divida o nmero 414 em partes diretamente proporcionais a 4, 8 e 10 e a 5, 6 e 7, ao mesmo tempo. 8) Divida o nmero 1.842 em partes diretamente proporcionais, simultaneamente, aos nmeros 3, 5 e 9 e 1 , 1 e 1 . 5 6 8 9) Divida o nmero 330 em partes inversamente proporcionais, simultaneamente, aos nmeros 3, 2 e 8 e 2, 4 e 6. 10) Divida o nmero 1.080 em partes diretamente proporcionais a 1 e 3 e inversamente proporcionais a 5 e 6, ao mesmo tempo. 2 4 11) Trs tcnicos receberam ao todo R$ 2.550. O primeiro trabalhou 15 dias razo de 6 horas por dia; o segundo, 25 dias razo de 4 horas por dia; e o terceiro, 30 dias razo de 5 horas por dia. Quanto recebeu cada um deles? 12) Uma pessoa, ao morrer, deixou a herana de R$ 21.720,00 para ser repartida entre trs herdeiros, ao mesmo tempo, em partes diretamente proporcionais a 3, 5 e 3 e inversamente 4 a 2 , 3 e 1. Quanto recebeu cada um? 3 5 3

13) Para a execuo de um servio, foram empregados 12 homens, 20 mulheres e 30 menores. Sabendo que o pagamento total foi de R$ 16.200, que cada mulher recebeu 3/4 da quantia de um homem e que cada menor recebeu 4/5 da quantia de cada mulher, quanto recebeu cada homem, cada mulher e cada menor? 14) Trs scios empregaram, respectivamente, os capitais de R$ 18.000, R$ 22.500 e R$ 27.000 e obtiveram um lucro liquido de R$ 27.000. Qual ser a parte de cada um? 159

PROF. WELLINGTON BRITO

Respostas: 1) 210, 300 e 360 2) 1.694, 605 e 1452 3) 26,78 e 221 4) 450,270 e 150 5) 1.218, 1.015 e 928 6) 640, 80 e 40 7) 60, 144, 210 8) 432, 600 e 810 9) 176, 132 e 22 10) 480 e 600 11) R$ 675 R$ 750 e 1.125 12) R$ 6.480 R$ 12.000 e R$ 3.240 13) R$ 360, R$ 270 e R$ 216 14) R$ 7.200, R$ 9.000 e R$ 10.800

REGRA DE TRS Introduo

Chamamos de regra de trs os problemas nos quais figura uma grandeza que direta ou inversamente proporcional a uma ou mais grandezas.

160

MATEMTICA PASSO A PASSO

Temos dois tipos de regra de trs: a simples, que trabalha com apenas duas grandezas, e a composta, que envolve mais de duas grandezas. Regra de Trs Simples

Neste caso, so dados dois valores de uma grandeza e um valor de outra, o qual corresponde a um dos valores da primeira grandeza. Devemos, ento, obter o valor da segunda grandeza que corresponde ao segundo valor da primeira.

Questes Comentadas
1) Comprei 6m de tecido por R$ 15,00. Quanto gastaria se tivesse comprado 8 m? Resoluo: Neste problema figuram duas grandezas: comprimento e preo do tecido. Se o comprimento for multiplicado por 2, 3, ..., o preo ficar multiplicado por 2, 3,... Podemos, ento, concluir que estamos trabalhando com grandezas diretamente proporcionais. Chamamos de x o valor que desejamos conhecer (preo de 8m de tecido), dispomos, em uma primeira linha horizontal, os valores conhecidos das duas grandezas que se correspondem e, em uma segunda linha, o outro valor conhecido da primeira e o x, que representa o valor correspondente da segunda e que se quer conhecer: Comprimento Preo (m) (R$) 6 15 8 x

Em seguida, colocamos uma seta vertical na coluna onde se encontra o x, com a ponta voltada para ele. Se as grandezas forem diretamente proporcionais, como no nosso exemplo, colocaremos uma segunda seta vertical de mesmo sentido na coluna dos outros dados. Assim: 6 15 161

PROF. WELLINGTON BRITO

x pelas razes que

Armamos a proporo formada construmos, seguindo as setas: 6 8


=

15 x
x= x = 20

e determinamos o valor de x:

8 x 15 6 Logo, o preo procurado : R$ 20,00 Observaes:

I) importante observar que as quantidades correspondentes a uma mesma grandeza devem ser expressas na mesma unidade de medida. II) Quando as grandezas que figuram no problema so diretamente proporcionais, dizemos que a regra de trs direta. 2) Se 6 operrios fazem certa obra em 10 dias, em quantos dias 20 operrios fariam a mesma obra? Resoluo: Temos: Operrios Dias 6 10 20 x Se o nmero de operrios for multiplicado por 2,3 ..., o nmero de dias ficar dividido por 2, 3, ..., respectivamente. Logo, as grandezas relacionadas so inversamente proporcionais.

Assim, a coluna que contm x assinalada como no problema anterior e a outra coluna assinalada com uma segunda seta vertical, de sentido contrrio ao da primeira: 6 10 20 x 162

MATEMTICA PASSO A PASSO

Em seguida, invertemos os valores da coluna do nmero de operrios (por ser uma grandeza inversamente proporcional de nmero de dias): 20 10 6 x 3 1 Da: 20 = 10 x = 6 x 10 x =3 6 x 20 10 1 Logo, sero necessrios: 3 dias Observaes: 1) Quando as grandezas que figuram no problema so inversamente proporcionais, dizemos que a regra de trs inversa. 2) Convm observar que, nos problemas de Matemtica, geralmente so consideradas condies iguais. No problema 2, por exemplo, supe-se que os operrios produzam igualmente e que as condies de trabalho tambm sejam iguais. EXERCCIO REGRA DE TRS 1) Um operrio recebe R$ 836,00 por 20 dias de trabalho. Quanto receber por 35 dias? 2) Uma viagem foi feita em 12 dias, percorrendo-se 150 km por dia. Quantos dias seriam empregados para fazer a mesma viagem, percorrendo-se 200 km por dia? 3) Se 1 c de lcool pesa 8 g, a quantos litros equivalem 32,4 kg de lcool? 4) Em um navio com uma tripulao de 800 marinheiros h vveres para 45 dias. Quanto tempo duraro os vveres se o navio receber mais 100 marinheiros? Respostas: 1) R$ 1.463,00 2) 9 dias 3) 40,5 litros 4) 40 dias

Regra de Trs Composta

Como dissemos antes, a regra de trs composta ocorrem trs ou mais grandezas relacionadas entre si. Neste caso de cada grandeza so dados dois valores, com exceo de uma delas, da 163

PROF. WELLINGTON BRITO

qual dado apenas um valor, relacionado com um dos valores de cada uma das outras grandezas.

Questes Comentadas
1) Se para imprimir 87.500 exemplares 5 rotativas gastam 56 min, em que tempo 7 rotativas, iguais s primeiras, imprimiro 350.000 desses exemplares? Resoluo: Temos a seguinte disposio prtica dos dados: Exemplares Rotativos Tempo (min) 87.500 5 56 350.000 7 x Fixando a segunda grandeza (nmero de rotativas), vemos que a primeira grandeza (nmero de exemplares) e a terceira (tempo) so diretamente proporcionais, pois duplicando o nmero de exemplares, o tempo empregado duplicar. Fixando, agora, a primeira grandeza,vemos que a segunda e a terceira so inversamente proporcionais, pois duplicando o nmero de rotativas, o tempo necessrio se reduzir metade. Assim temos: 87.500 350.000 87.500 350.000 5 7 7 5 56 x 56 x

Invertendo os valores da segunda grandeza, vem:

O que nos permite escrever, pela propriedade da grandeza proporcional a vrias outras: 56 = 87.500 x 7 x 350.000 x 5 Da: 8 4 x = 56 x 350.000 x 5 x = 160 87.500 x 7 1 1 Isto : x = 160 min ou x = 2 h 40 min 2) Quinze operrios, trabalhando 9 h por dia, construram 36 m de muro em 16 dias.Em quanto tempo 18 operrios faro 60 m do mesmo muro, trabalhando 8 h por dia? Resoluo : Temos: operrios jornadas comprimentos dias 164

MATEMTICA PASSO A PASSO

(h) (m) 15 9 36 16 18 8 60 x Verificaremos, com facilidade, que a quarta grandeza (nmero de dias) diretamente proporcional terceira (comprimento) e inversamente proporcional primeira (nmero de operrios) e segunda (jornada de trabalho dirio). Assim: 15 9 36 16 18 8 60 x Invertendo os valores da primeira e da segunda grandezas, temos: 18 8 36 16 15 9 60 x 1 5 2 10 1 5 Calculando o valor de x: x = 16 x 60 x 9 x 15 = 5 x 5 x = 25 18 x 8 x 36 2 1 6 1 2 1 Logo os operrios faro o muro em 25 dias EXERCCIO REGRA DE TRS SIMPLES/COMPOSTA 1) Se 35 m de um tecido custam R$ 140, quanto se pagar por 12 m? 2) Se 20 tratores levaram 6 dias para realizar um trabalho, quantos tratores o fariam em 4 dias? 3) Um trem percorreu 24,5 km em 28 min. Que distncia percorreria, com a mesma velocidade, em 54 min? 4) Um empreiteiro calculou terminar uma obra em 32 dias, empregando 15 operrios. Tendo conseguido apenas 12 operrios, em quantos dias terminar o mesmo trabalho? 5) Um operrio faz, em 12 dias, um trabalho cuja dificuldade representada por 0,2. Em quantos dias poderia fazer outro trabalho cujo coeficiente de dificuldade fosse 0,25? 6) Trabalhando 6 h por dia um operrio pode fazer um trabalho em 24 dias. Em quantos dias, nas mesmas condies, poderia faz-lo, trabalhando 8 h por dia? 7) Em um acampamento militar com 300 soldados h vveres para 20 dias. Tendo chegado mais 140 soldados, a quanto 165

PROF. WELLINGTON BRITO

se deve reduzir a rao diria para que o alimento dure ainda o mesmo tempo? 8) Uma lebre est 80 m frente de um co que a persegue. Enquanto a lebre percorre 19 m, o co percorre 21 m. Quantos metros dever percorrer o co para alcanar a lebre? 9) Um automvel, correndo com uma velocidade e 84 km/h, deve percorrer uma certa distncia em 9 h. Depois de 3 h de viagem houve um desarranjo no motor e o automvel teve de parar durante 45 min. Com que velocidade deve continuar a viagem para chegar ao ponto final na hora fixada? 10) Se 4 de uma obra foram avaliados em R$ 268.400, qual o 5 valor de 5 da mesma obra? 11 11) As dificuldades de dois trabalhos esto na razo de 3 para 4. Um operrio, que faz 20 m do primeiro trabalho, quantos metros far do segundo, no mesmo tempo? 12) Duas polias, de 16,8 cm e 11,2 cm de dimetro, respectivamente esto ligadas por uma correia de transmisso. Enquanto a maior d 540 voltas, quantas voltas d a menor? 13) Para fazer um muro de 52 m de comprimento, 30 operrios gastam 15 dias de 8 h. Quantos dias de 9 h gastaro 25 operrios para fazer 39 m de um muro igual? 14) Comparando-se os preos pelos quais so vendidas diversas frutas, verificamos que 15 peras valem 9 mas; 25 abacates valem 15 mas; e 16 laranjas valem 12 abacates. Quantas laranjas podero ser trocadas por 9 peras? 15) Um motoqueiro, numa velocidade de 80 km/h, percorreu certa distncia em 6 dias, viajando 4 1 h por dia. 2 afrouxando em 1 a sua velocidade e viajando 6 h por dia, 10 quantos dias levar para percorrer a mesma distncia? 16) Certo trabalho executado por 8 mquinas iguais, que trabalham 6 h dirias em 15 dias. Quantos dias levariam 10 mquinas do mesmo tipo para executar o triplo do trabalho anterior,trabalhando 5 h dirias,com a velocidade que torna o rendimento 1 maior? 166

MATEMTICA PASSO A PASSO

8 17) Dois cavalos foram pagos em razo direta de suas velocidades e inversa de suas idades. Sabendo que a velocidade do primeiro est para a do segundo como 3 est 4, que as idades do primeiro e do segundo so respectivamente, 3 anos e 9 meses e 5 anos e 4 meses, e que pelo primeiro foram pagos R$ 480, 00, qual foi o preo do segundo? 18) Na construo de uma estrada trabalharam 20 homens durante 18 dias em seguida trabalharam 24 homens durante 10 dias. Em quanto tempo teria ficado pronta se os 24 homens houvessem trabalhado desde o comeo? Respostas: 1) R$ 48,00 2) 30 tratores 3) 47,25 km 4) 40 dias 5) 15 dias 6) 18 dias 7) Reduzida em 7/22 8) 840 metros 9) 96 km/h 10) R$ 152.500,00 11) 15 metros 12) 810 voltas 13) 12 dias 14) 12 laranjas 15) 5 dias 16) 38 dias e 2horas 17) 17)R$ 450,00 18) 25 dias QUESTES DE CONCURSOS E VESTIBULARES 1) (T.J) A razo entre dez minutos e um dia de: a) 1:120 b) 1:144 c) 1:180 d)1:196 e) 1:240 167

PROF. WELLINGTON BRITO

2)

(C.E.F) O faxineiro A limpa certo salo em 4 horas. O faxineiro B faz o mesmo servio em 3 horas. Se A e B trabalharem juntos, em quanto tempo, aproximadamente, espera-se que o servio seja feito? a) 2 horas e 7 min. c) 1 hora e 57 min. e) 1 hora e 36 min. b) 2 horas e 5 min. d) 1 hora e 43 min. (T.J) Em 4 horas duas torneira enchem um tanque. Sozinha, uma delas encheria o tanque em 7 horas. Quanto tempo seria necessrio para a segunda torneira encher o tanque? a) 9 h b) 9h 20min c)9h 30min d) 9h 40min e) 9h 50min (T.R.E) A idade de um pai est para a idade de seu filho assim como cinco est para dois. Calcule essas idades, sabendo que a diferena entre elas de 21 anos. a) 37 e 16anos b) 36 e 15anos c) 49 e 18anos d) 35 e 14 anos e) 33 e 12anos (T.J) A planta de uma casa foi elaborada na escala 1:50. Ento, a rea real, em metros quadrados, de uma sala cujas medidas na planta so de 12cm e 14cm : a) 24 b) 28 c) 42 d) 48 e) 54 (SEFAZ) A miniatura de um foguete balstico foi feita na escala de 1/400. O comprimento real do foguete 116m. O comprimento correspondente na miniatura de: a) 0,029cm b) 4,6m c) 2,9dm d) 0,34m e)3,44dm (T.R.E.) Trs amigos comparam um terreno de 10.800m 2. Qual a poro de cada um se o primeiro entrou com R$ 16.000,00, o segundo com R$ 20.000,00 e o terceiro com R$ 24.000,00?
a) 3.000m2; 3.800m2 e 4.000m2 c) 2.880m2; 3.700m2 e 4.220m2 e) 2.880m2; 3.600m2 e 4.320m2 b) 3.880m2; 2.600m2 e 4.320m2 d) 3.800m2; 3.680m2 e 4.120m2

3)

4)

5)

6)

7)

8)

(T.J) Duas pessoas associam-se entrando a primeira com R$ 60.000,00 e a segunda com R$ 40.000,00. sabendo-se que a primeira afundou a firma e a segunda participou durante 6 meses, qual a parte do lucro que coube a cada uma se lucraram, no fim de 1(um) ano R$ 96.000,00? 168

MATEMTICA PASSO A PASSO

a) R$ 72.000,00 e R$ 24.000,00 c) R$ 74.000,00 e R$ 22.000,00 e) R$ 52.000,00 e R$ 44.000,00

b) R$ 50.000,00 e R$ 46.000,00 d) R$ 70.000,00 e R$ 26.000,00

9)

(T.J.) Uma pequena empresa foi assim constituda: Ana investiu um capital de R$ 1.200,00: decorridos seis meses, ingressou Bia, com um capital de R$ 800,00: passados mais seis meses, foi admitida Carla, com um capital de R$ 2.000,00. Aps dois (2) anos de funcionamento, a empresa apresentou um lucro de R$ 1.400,00. Considerando que o contrato de formao da empresa estabelece que 4% do lucro apurado destinam-se constituio de um fundo de reserva, assinale a opo correta, em relao a diviso do restaurante do lucro obtido pela empresa: a) Ana recebeu R$ 600,00; b) Bia recebeu 50% da quantia que coube a Ana; c) Carla recebeu mais de R$ 500,00: d) Ana e Carla receberam juntas mais de 80% do lucro rateado; e) Se no existisse o fundo de reserva, cada scio teria recebido 4% a mais do que efetivamente recebeu.

10) (BB) Se 78 dividido em 3 partes proporcionais a 1, 1/3 e 1/16, ento a parte do meio ser: a) 9 1 b) 13 c) 17 1 d) 18 1 e) 26 3 3 3 11) (TTN) Dividi-se 315 em trs partes, A, B, C, que so ao mesmo tempo diretamente proporcionais a 3, 2 e 5 e inversamente proporcionais a 5, 3 e 6, respectivamente. O maior valor dessas partes : a) 225 b) 156 c) 145 d) 100 e) 125

12) (TJ) Uma torneira, que jorra 20 litros dagua por minuto, enche um tanque em 6 horas. Qual o tempo em que encher o mesmo tanque uma torneira que deite 30 litros dgua por minuto? 169

PROF. WELLINGTON BRITO

a) 10 horas b) 09 horas c) 08 horas d) 04 horas e) 02 horas

13) (TJ) Um pneu de boa qualidade roda em mdia 40.000 km/ano e custa R$ 56,00. Um pneu de qualidade inferior roda 32.000 km/ano. Nessas condies, interessante adquirir o pneu de qualidade inferior at o preo mximo de:
a) R$ 50,00 b) R$ 45,00 c) R$ 46,80 d) R$ 45,20 e) 44,79

14) (TTN) Um navio, com guarnio de 300 homens, necessita de 120.000 litros de gua para efetuar uma viagem de 20 dias. Aumentando a guarnio em 50 homens e a gua em 6.000 litros. Determine qual poder ser a durao da viagem. a) 24 dias b) 22 dias c) 20 dias d) 18 dias e) 16 dias 15) (TJ) Um navio cargueiro, com 30 homens de tribulao, encontrou alguns nufragos durante a viagem e reduziu a rao de cada homem de 48 dag para 288g. Quantos eram os nufragos? a) 40 b) 35 c) 30 d) 20 e) 25 16) (BB) 15 operrios, trabalhando 8 horas por dia, em 30 dias manufaturam 900 pares de sapatos. Quantos pares sero manufaturados por 8 operrios, trabalhando 40 dias de 6 horas, sabendo-se que os novos sapatos apresentam o dobro da dificuldade dos primeiros? a) 85 b) 135 c) 240 d) 480 e) 960 17) (BB) Trabalhando 10 horas, durante 15 dias, 8 pedreiros fizeram uma parede de concreto de 48m 2. Se tivessem trabalhando 12 horas dirias, e se o nmero de operrios fosse reduzido de 2, quantos dias levariam para fazer outra parede cuja rea fosse o dobro daquela?
a) 33d b) 33d 8h c)33d 4h d)33d 6h e)33d 5h

18) (PRF) Para construir um muro, Joo levaria 30 dias e Carlos 25 dias. Os dois comearam a trabalhar juntos, mas aps 6 dias, Joo deixa o trabalho; 2 dias aps a sada deste, Carlos tambm o abandona, Antonio, sozinho, consegue 170

MATEMTICA PASSO A PASSO

termina-lo em 24 dias. Para realizar a construo do muro, sozinho, Antonio levaria: a) 50 dias b) 45 dias c) 40 dias d) 35 dias e) 30 dias 19) (SEFAZ) Se x = y = z e 2x + 3y z = 42, 6 3 7 ento 3x + 2y + Z igual a: a) 91 b)93 c) 95 d) 97 e)99 20) (ANTT) Um adesivo colado em um caminho de carga indica: carga mxima 1 ton, o que significa que aquele caminho pode transportar, com segurana, no mximo uma tonelada de carga. O caminho ser abastecido com caixas de um certo produto.Cada caixa tem um peso bruto de 4.250g.Nesse caso, ele poder transportar, no mximo, a seguinte quantidade de caixas. a) 23 b) 24 c)205 d) 235 e) 2350 21) (ANTT) A cada 1200m rodados em viagem, o automvel de Pascoal gasta 0,09 litro de combustvel. Numa viagem, Pascoal gastou 54,9 litros de combustvel. O percurso teve ento a seguinte quantidade de quilmetros: a) 776 b) 732 c) 688 d) 654 e)586 22) (BNB) Em uma fbrica de automveis, 3 (trs) robs, trabalhando 8(oito) horas por dia, constroem em 6 (seis) dias, 36 unidades de uma pea nobre utilizada na construo automobilstica. Uma equipe de 5 (cinco) robs trabalhando 6 (seis) horas por dia, constri 15 unidades da citada pea em: a) 2 dias b) 5 dias c) 4 dias d) 3 dias e) 6 dias 23) (BB) Um bloco de concreto de 3 metros de comprimento, 1,5 metros de largura e 60 cm de espessura pesa 6.300 kg. Quanto pesar um outro bloco do mesmo concreto com 2,2 m de comprimento, 80 cm de largura e 90 cm de espessura?
a) 3.686kg b) 3.690kg c) 3.696kg d) 3.966kg e) 0 3.969kg

24) (BB) A uma caixa de gua que mede 2,5 m de largura, 0,5 dam de comprimento e 0,004km de altura, acham-se ligadas duas torneiras que fornecem, respectivamente 0,9kl e 20, 8hl de gua por hora. H um escape continuo que perde 0,8 171

PROF. WELLINGTON BRITO

dal por minuto. Determinar em quantas horas a caixa ficar cheia, funcionando conjuntamente ambas as torneiras e o escape?
a) 22 horas b) 21 horas c) 20 horas d) 19 horas e) 18 horas

25) (TTN) Dividir o nmero 570 em trs partes, de tal forma que a primeira esteja para a segunda assim como 4 est para 5, e a segunda esteja para a terceira assim como 6 est para 12. Nestas condies, a terceira parte vale: a) 120 Respostas: 1) 2) 3) 4) 5) B D B D C 06) C 07) E 08) A 09) B 10) C 11) E 12) D 13) E 14) D 15) D 16) C 17) C 18) A 19) B 20) D 21) B 22) A 23)C 24)C 25)D b) 150 c) 320 d) 300 e) 250

PERCENTAGEM Introduo

172

MATEMTICA PASSO A PASSO

Em nosso dia-a-dia comum observarmos expresses como estas: Desconto de at 30% na grande liquidao de vero. Os jovens perfazem um total de 50% da populao brasileira. A inflao registrada em dezembro foi de 1.93%. O rendimento da caderneta de poupana foi de 1,99% em dezembro. Todas estas expresses envolvem uma razo especial chamada percentagem. Taxa Percentual

Suponhamos que um aluno tenha acertado, em um exame, 12 das 15 questes apresentadas. A razo entre o nmero de questes acertadas e o nmero total de questes : 12 = 4 = 0,8 = 8 = 80 = .... 15 5 10 100

Quando uma razo apresentada com o conseqente 100 neste caso, 80 , ela chamada razo centesimal. 100

80 100

Uma outra forma de representarmos as razes centesimais, muito usada principalmente no universo econmico-financeiro, substituir o conseqente 100 pelo smbolo % (que lemos: por cento). Assim: 80 = 80% (lemos: oitenta por cento) 100 Esse numeral (80%) denominado taxa percentual. Elementos do Clculo Percentual

12 = 80 15 100 Neste exemplo, chamando o 12 de percentagem, o 15 de principal e o 80 de taxa, temos: Percentagem = Principal taxa 100

Vimos que:

Da, obtemos as seguintes definies: Taxa o valor que representa a quantidade de unidades tomadas em cada 100. 173

PROF. WELLINGTON BRITO

Percentagem o valor que representa a quantidade tomada de outra, proporcionalmente a uma taxa. Principal o valor da grandeza da qual se calcula a percentagem. O principal, a percentagem e a taxa so os elementos do clculo percentual. Observao: Na prtica, muito comum: ---- empregarmos as palavras desconto, comisso, multa, parte, quota, abatimento, prejuzo, lucro etc. em lugar de percentagem; ---- designarmos a taxa percentual simplesmente por percentagem. Assim, tanto faz dizermos, em uma situao qualquer, que o lucro foi de R$ 80 ou de 20%

Problemas de percentagem O principal por P; A percentagem por p; A taxa por r; p P =r 100

Representando:

Temos, genericamente:

Questes Comentadas

1) Escreva a razo 3 em forma de taxa percentual. 4 Resoluo: 25 Temos: 3 4 = x 100 x = 3 x 100 = 4 1 75

Logo, a resposta e: 75 % 2) Um vendedor tem 3% de comisso nos negcios que faz. Qual sua comisso numa venda de R$ 3.600? Resoluo: 174

MATEMTICA PASSO A PASSO

Temos: Assim: p 3.600 =

P = 3.600 r =3 3 100 p= 3.600 x 3 100 = 108

Logo, a comisso de: R$ 108 3) Em um colgio 26% dos alunos so meninas. Quantos alunos possui o colgio, se elas so em nmero de 182? Resoluo: Temos: Assim: 182 P p = 182 r = 26 = 26 100 P = 182 x 100 26 = 700

Logo, o colgio possui : 700 alunos 4) Um automvel foi adquirido por R$ 5.000 e vendido com um lucro de R$ 400. Qual a percentagem de lucro? Resoluo: Temos: Assim: 400 p = 5.000 r = 400 r = 5.000 100 r = 400 x 100 5.000 = 8

Logo, o lucro foi de: 8%

EXERCCIO - PERCENTAGEM ( I)

1) Exprima sob a forma de taxa percentual as razes: 175

PROF. WELLINGTON BRITO

a)

2 25

b) 19 40

c) 1 4

2) Em uma liquidao, uma camisa que custava R$ 24 foi vendida com 25% de abatimento. De quanto foi o abatimento? 3) Um corretor recebe R$ 2.800 pela venda de duas casas, tendo sido de 5% a taxa de comisso. Qual o valor de venda das propriedades? 4) Uma pessoa devia R$ 20.000 e pagou R$ 7.400. Quantos por cento da dvida foram pagos? Respostas: 1) a) 8% 2) R$ 6,00 b) 47,5% c) 25% 3) R$ 56.000,00 4) 37%

Taxa Unitria Vimos que a taxa percentual se refere a 100, isto : 25 = 25% 100 Porm, na resoluo de muitas questes, mais prtico (e, algumas vezes, necessrio) tomarmos como valor referencial a unidade, obtendo o que chamamos de taxa unitria (simbolizada por i). Assim: 25 i 25 = = 0,25 i = 100 1 100 Temos, ento: 25 i = 0,25 = = 25% 100

Questes comentadas
1) Qual a taxa unitria correspondente a 20%? Resoluo: Temos: 20% = 20 100 = 0,2 Logo, i = 0,2

2) Qual a taxa percentual correspondente a 0,05? Resoluo: 176

MATEMTICA PASSO A PASSO

Temos:

0,05 =

5 100

= 5%

Logo, i = 5%

3) Calcule 30% de 15% Resoluo: Temos: Como: 30% = 0,3 e 15% = 0,15 0,045 = 4,5% Ento: 30% de 15% = 0,3 de 0,15 = 0,3 x 0,15 = 0,045 a resposta : 4,5% 4) Um comerciante vendeu um objeto por R$ 540 com um lucro de 15% sobre esse valor. Quanto ganhou? Resoluo: Temos: P = 540 i = 15% = Como: p = i p = Pi P 15 = 0,15 100

Vem: p = 540 x 0,15 = 81 Logo, o comerciante ganhou: R$ 81 5) Um terreno tem 70% de sua rea plantada, que corresponde a 154 ha. Qual a rea total do terreno? Resoluo: Temos: p = 154 i = 70% Como: Vem: P p = 70 = 0,7 100 P = p i

= i Pi = p

P = 154 = 220 0,7

Logo, a rea total de : 220 ha

6) Em uma turma de 60 alunos, foram reprovados 9. Quantos por cento dos alunos foram reprovados? 177

PROF. WELLINGTON BRITO

Resoluo: Temos: P = 60 p= 9 Como: Vem: p P i= = i ou i = p p 9 = 0,15 = 15 = 15 % 60 100

Logo, foram reprovados : 15% dos alunos EXERCCIO - PERCENTAGEM (II)

1) Exprima, sob a forma de taxa percentual, cada uma das seguintes razes: a) b) 1 20 2 5 c) 5 2 d) 3 1 4 e) 37 80 f) 0,24 g) 0,125 h) 0,012

2) Escreva as taxas percentuais abaixo como razes, sob a forma mais simples possvel: a) 80% c) 25,2% e) 18,6% f) 2 % 3 g) 0,054% h) 2 1 % 4

b) 66% d) 0,48%

3) Calcule: a) 20% de 300 c) 9% de 50 e) 0,4% de 550 4 1 b) 15% de R$ 160 d) 6,5% de 1.200 kg f) % de 750
2

4) Calcule quantos por cento: a) R$ 121 so de R$ 484; b) 936 g so de 15.600g; c) 912,5 g so de 73 kg; d) 45 so de 180 dm3

178

MATEMTICA PASSO A PASSO

5)

Calcule a quantia da qual: a) R$ 42 representam 5% c) R$ 33 representam 5,5% b) R$ 280 representam 8% d) R$ 320 representam 1,25%

6) Meio representa quantos por cento de


7) 8) 9) Qual o nmero cujos 7% valem 28?

5 8

Por quanto devo vender um objeto que me custou R$ 70 para obter um lucro de 30%? Uma nota promissria, cujo valor era de R$ 5.000, foi paga com um desconto de R$ 250. Qual a taxa de desconto?

10) Em So Paulo colhem-se 1.268.000 sacas de caf. Se 25% desta produo destinam-se ao consumo interno, qual a quantidade de sacas para este consumo? 11) Um jornal recebia por dia R$ 42.000 de anncios. Os preos dos anncios foram aumentados em 6%. Qual ser a nova receita diria do jornal? 12) Em quanto por cento aumentou a populao de um cidade que era de 67.200 habitantes e agora de 92.400 habitantes? 13) Um terreno foi vendido por R$ 9.600, recebendo o intermedirio de 3% de comisso. Calcule a comisso. 14) Em uma escola, 40% dos alunos so meninas. O total dos alunos 750. Quantos so os meninos? 15) Em uma cidade, 35% da populao constituda de homens e 40% de mulheres. Qual a populao da cidade, se o nmero de crianas de 8.000? 16) Vendi uma mercadoria recebendo 25% de entrada e o restante em trs prestaes de R$ 160 e uma de R$ 180. Qual o preo da mercadoria? 17) Um vendedor recebe 3% de comisso sobre as vendas que efetua. Qual a quantia a receber pelas vendas de R$ 8.000, R$ 3.700 e R$ 9.500?

179

PROF. WELLINGTON BRITO

18) Em um dos Grandes Prmios de Formula 1 largaram 24 carros e terminaram a competio 10 carros. De quanto por cento foi o nmero de carros que no terminaram a corrida? 19) Um comerciante comprou 120 bons a R$ 8 cada um. Vendeu a metade a R$ 10 e o restante a R$ 12. De quanto por cento foi o lucro? 20) Um comerciante pagou 20% de uma dvida. Determine a dvida inicial, sabendo que com R$ 43.680 ele pagou 35% do restante. 21) Uma pessoa entregou a um banco a quantia de R$ 562 para pagamento de uma ordem a ser expedida por telegrama. O custo do telegrama foi de R$ 2 e a comisso, de 1 % . Qual o valor da ordem? 8 22) Tm-se duas misturas de lcool com gua;uma contm 24 de lcool e120 de gua e a outra, 21 de lcool e 112 de gua. Qual a mais forte e em quanto por cento?

23) Uma casa, que est alugada por R$ 9.600 ao ano, foi comprada
por R$ 98.000. O proprietrio gastou com ela, durante o ano, R$ 1.180 em impostos e reparos. Qual foi a taxa de rendimento do capital empregado? 24) Comprei 6 peas de tecido de 50 m a R$ 9 o metro. Quero vendlas com um lucro de 30%. Vendo a tera parte razo de R$ 11 o metro. Por quanto devo vender o metro do tecido restante? 25) Um comerciante adquiriu 3 sacos de 60kg de certo cereal, razo de R$ 48 o saco. Obteve, por ter pago vista, um desconto de 5% e teve uma despesa de transporte se R$ 5. Revendendo o cereal a R$ 1 o quilograma, qual ser a percentagem de lucro? 26) Em uma partida de futebol, um dos times obteve os seguintes resultados quanto aos chutes a gol: - bolas chutadas fora: 10; - bolas defendidas pelo goleiro adversrio:6; - bolas na trave: 2; - gols: 2;

180

MATEMTICA PASSO A PASSO

a) Qual a percentagem dos gols em relao s bolas chutadas a gol? b) Qual a percentagem das bolas chutadas fora? c) Qual a percentagem das bolas defendidas pelo goleiro adversrio?

27) Um relojoeiro adquire um lote de 120 relgios razo de R$ 80


cada um. Vende a R$ 95 cada um e o restante a R$ 102 cada um. De quanto por cento foi o lucro?

28) Uma dona de casa compra um pedao de carne com osso e paga
R$ 3. Ao desoss-lo, percebe que os ossos correspondem a 12% do peso total. Sabendo que o preo do quilo dessa carne de R$ 2 e que, durante o cozimento, a carne perde 15% de seu peso, qual o peso do pedao de carne cozida? 29) Em um concurso prestado por certo nmero de candidatos houve 18% de aproveitamento, ou seja, 117 aprovados; num outro, a que concorreram 350 candidatos, houve 22% de aproveitamento. Determine quantos candidatos se submeteram ao primeiro concurso e quantos foram reprovados no segundo. 30) Uma pessoa deseja adquirir uma televiso catalogada por R$ 460. Se o pagamento for vista,a loja oferecer um desconto de 5%.Como a pessoa no pode faz-lo, paga 2/5 vista e o restante em 3 prestaes,sofrendo um aumento de 25% sobre a parte relativa s prestaes. a) b) Qual o preo vista da televiso? Qual o valor de cada prestao?

Respostas
1) a) 40% b) 5% c) 250% d) 325% 15) 32.000 16) R$ 880 17) R$ 636 18) 58,33% 19) 35,5%

e) 46,25% f) 24% g) 12,5% h) 1,2% 2) a) 4 5 e) 93 f) b) 33 50 1 g) c) 63 250 27 d) 3 625 h) 9

181

PROF. WELLINGTON BRITO

500 3) a) 60 d) 78kg 4) 5) 6) 7) a) 25%

150

50.000 c) 4,5

400

20) R$ 156.000 21) R$ 559 22) A primeira; 1,25% 23) 8,59%

b) R$ 24,00 e) 2,2 b) 6%

f) 33,75 c) 1,25% d) 25%

a)R$ 840 b)R$ 3.500 c) R$ 600 d) R$ 25.600 24) R$ 12 80% 400 25) 26,9% 26)a)10% b) 50% c) 30% 27) 21,7% 28) 1,122kg 29) 650 e 273 30)a) R$437 b) R$115

8) R$ 91 9) 5%

10) 317. 000 11) R$ 44.520 12) 37,5% 13) R$ 288 14) 450 meninos

OPERAES SOBRE MERCADORIAS- PASSO A PASSO Introduo

O que vamos estudar neste captulo so os problemas de percentagem ligados s operaes de compra e venda de mercadorias, isto , vamos aprender a fazer clculos de lucro ou prejuzo sobre os preos de custo e de venda de mercadorias. Vendas com Lucro

A venda de mercadorias pode oferecer um lucro e este lucro pode ser sobre o preo de custo ou sobre o preo de venda. Observao: 182

MATEMTICA PASSO A PASSO

Preo de custo de uma mercadoria compreende o preo de aquisio, acrescido das despesas diretas sobre a compra e sobre a venda e, ainda, das despesas de administrao e funcionamento da empresa. Lucro Sobre o Preo de Custo Passo a Passo

Consideremos o seguinte problema: Um comerciante vendeu mercadorias com um lucro de 8% sobre o preo de custo. Determine o preo de venda, sabendo que essas mercadorias custaram R$ 500. Sabemos que: preo de venda = preo de custo + lucro Como o lucro de 8% sobre o preo de custo, isto : Lucro = 0,08 do preo de custo, temos: preo de venda = preo de custo + 0,08 x preo de custo = = ( 1 + 0,08) x preo de custo = = 1,08 x 500 = 540 Logo, o preo de venda de : R$ 540

Frmula: Chamando de : V C L i o preo de venda o preo de custo o lucro a taxa unitria do lucro

Vem:

V=C+L

Como : L = i x C 183

PROF. WELLINGTON BRITO

Temos: V = C + i x C Logo : V=(1+i)C

que nos d o preo de venda, conhecidos o custo e a taxa de lucro sobre o custo. Lucro Sobre o Preo de Venda Passo a Passo

Comprou-se um objeto por R$ 60 e deseja-se ganhar 25% sobre o preo de venda. Qual deve ser este preo? Sabemos que: preo de venda lucro = preo de compra Como o lucro de 25% sobre o preo de venda, isto : Lucro = 0,25 do preo de venda Temos: preo de venda 0,25 x preo de venda = preo de custo ( 1 0,25) x preo de venda = preo de custo Ou: Ou ainda: Preo de venda = preo de custo = 60 0,75 Logo, o preo de venda deve ser de : R$ 80 Frmula: Temos Como: Vem: Logo VL=C L=ixV VixV=C (1i)V=C V= C 0,75 = 80

1i Que nos d o preo de venda, conhecidos o preo de custo e a taxa de lucro sobre o preo de venda. Vendas Com Prejuzo Analogamente ao que ocorre com o lucro, uma mercadoria pode ser vendida com prejuzo sobre o preo de custo ou sobre o preo de venda. 184

MATEMTICA PASSO A PASSO

Prejuzo Sobre o Preo de Custo Passo a Passo Considere o seguinte problema: Um objeto foi vendido com um prejuzo de 40% sobre o preo de custo. Sabendo que esse objeto custou R$ 30, qual foi o preo de venda ? Sabemos que: preo de venda = preo de custo prejuzo Como o prejuzo de 40% sobre o preo de custo, isto : prejuzo = 0,4 do preo de custo Temos: preo de venda = preo de custo 0,4 x preo de custo= = ( 1 0,4) x preo de custo = = 0,6 x preo de custo = 0,6 x 30 = 18 Logo, o preo de venda foi de : R$ 18 Frmula: Chamando de P o prejuzo, vem: V= CP Como: Temos: Logo: P= i x C V = C iC

V = (1 i ) C

que nos d o preo de venda, conhecidos o custo e a taxa do prejuzo sobre o custo.

Prejuzo Sobre o Preo de venda Passo a Passo Uma casa que custa R$ 96.000 foi vendida com um prejuzo de 20% sobre o preo de venda. Calcule o preo de venda. 185

PROF. WELLINGTON BRITO

Sabemos que: preo de venda + prejuzo = preo de custo Como o prejuzo de 20% sobre o preo de venda, isto : prejuzo = 0,2 do preo de venda, temos: preo de venda + 0,2 x preo de venda = preo de custo ou: ( 1 + 0,2) x preo de venda = preo de custo ou ainda: preo de venda = preo de custo = 96.000 = 80.000

1,2 1,2 Logo, o preo de venda ser de: R$ 80.000 Frmula: Como: Temos: Logo: V+P = C e P=i x V V + iV = C ( 1 + i ) V = C C 1+i que nos d o preo de venda, conhecidos o preo de custo e a taxa do prejuzo sobre o preo de venda. Questes Comentadas 1) Vendi um objeto por R$ 276 e ganhei na venda 15% sobre o preo de custo. Quanto custou o objeto? Resoluo: Temos: V = 276 i = 15% = 0,15 Como: V = C(1+ i ) ou C( 1 + i) = V Vem: C ( 1 + 0,15) = 276 1,15 x C = 276 C = Logo, o objeto custou: R$ 240 2) Comprei uma mercadoria por R$ 480. Sendo minha inteno vend-la com um lucro de 20% sobre o preo de venda, qual deve ser este ltimo ? 186 276 1,15 C = 240

V=

MATEMTICA PASSO A PASSO

Resoluo: Temos: C = 480 r = 20% = 0,2 Como: Vem: V = V = C 1i 480 = 480 V = 600 1 0,2 0,8

Logo, o preo de venda deve ser de: R$ 600 3) Um terreno foi comprado por R$ 5.000 e vendido por R$ 6.500. De quanto por cento foi o lucro sobre o preo de compra? Resolua Temos: C = 5.000 V = 6.500 Lembrando que: V = C(1 + i ) ou C( 1 + i ) = V 6.500 65 Vem: 5.000 ( 1+ i ) = 6.500 1 + i = i = -1 5.000 50 50 50 Logo, o lucro sobre o custo foi de: 30% i= 65 50 i = 15 i = 0,3

4) Quanto custou um objeto vendido por R$ 248 com um prejuzo de 20% sobre o preo de custo? Resoluo: Temos: V = 248 i = 20% = 0,2 Como: V = C(1 i ) ou C( 1 i ) = V Vem: C ( 1 - 0,2) = 248 0,8 x C = 248 C = Logo, o objeto custou: R$ 310 5) Um terreno foi vendido por R$ 50.600, dando um prejuzo de 8% sobre o preo de venda. Quanto havia custado? Resoluo: 187 248 0,8 C = 310

PROF. WELLINGTON BRITO

Temos:

V = 50.600 i = 8% = 0,08 C 1+i ou C = 1+i V

Lembrando que: V =
Vem:

C = 50.600 C = 50.600 C = 50.600 x 1,08 C = 54.648 1+ 0,08 1,08

Logo, o terreno havia custado: R$ 54.648 EXERCCIO OPERAES SOBRE MERCADORIA - (I) 1) Um comerciante comprou determinada mercadoria por R$ 650. Por quanto dever revende-la para obter um lucro de 30%? 2) Um aparelho de som foi vendido por R$ 360. Qual o lucro obtido, sabendo que o mesmo foi calculado na base de 25%? 3) Um objeto comprado por R$ 80 foi revendido por R$ 104. Qual a taxa pela qual se calculou o lucro sobre o preo de custo? 4) Um objeto foi vendido, com prejuzo de 10%, pelo preo de R4 36. Quanto havia custado? 5) Uma agncia vendeu um carro por R$ 8.500. Sabendo que na venda teve um prejuzo de 15% sobre o preo de venda, quanto custou esse carro? Respostas: 01) R$ 845 02) R$ 72 03) 30% 04) R$ 40 05) R$ 9,775 Descontos Sucessivos Passo a Passo

Nesse item,vamos aprender a calcular os descontos sucessivos sobre uma importncia resultante de um negcio efetuado. Consideremos o seguinte problema: Uma firma distribuidora oferece, sobre o valor de uma fatura, os descontos sucessivos de 10%, 4% e 5%. Sabendo que o valor da fatura de R$ 48.000, qual ao valor lquido da mesma? Basta, evidentemente, calcularmos os lquidos parciais correspondentes aos abatimento oferecidos, respeitando a ordem das taxas, at obtermos o liquido final. Assim, chamando o valor liquido de L, temos: 188

MATEMTICA PASSO A PASSO

P = 48.000 i1 = 10% = 0,1 Como:


p1 = P x i1 p1 = 48.000 x 0,1= 4.800 L1 = 48.000 4.800 = 43.200 p2 = L1 x i2 p2 = 43.200 x 0,04 = 1.728 L2 = 43.200 1.728 = 41.472 p3 = L2 x i3 p3 = 41.472 x 0,05 = 2.073,6 L3 = 41.472 2.073,6 = 39.398,4

i2 = 4% = 0,04

i3 = 5% = 0,05

O valor lquido da fatura de: R$ 39.398 Frmula do Desconto sucessivo L = P ( 1 i1 ) ( 1 i2 ) ( 1 i3 )... ( 1 in ) Onde: i1 , i2 , i3 ... in , so taxas sucessivas. Observao: Para aumentos sucessivos, temos M = P ( 1 + i1 ) ( 1 + i2 ) ( 1 + i3 )... ( 1 + in )

Questes Comentadas
1) Uma firma distribuidora oferece, sobre o valor de uma fatura, os descontos sucessivos de 10%, 4% e 5%. Sabendo que o valor da fatura de R$ 48.000, qual o valor liquido da mesma? Resoluo: Temos: Assim: P = 48.000 i1 = 10% = 0,1 i2 = 4%= 0,04 i3 = 5% = 0,05 L = 48.000 ( 1 0,1 ) ( 1 0,04) ( 1 0, 05 ) = = 48.000 x 0,9 x 0,96 x 0,95 = 39,398, 4 O valor lquido da fatura de: R$ 39.398

2)

Sobre um artigo de R$ 2.500 incide um imposto federal de 10% e um estadual de 4%.Qual o preo final desse artigo? Resoluo: 189

PROF. WELLINGTON BRITO

Temos:

P = 2.500 i1 = 10% = 0,1

i2 = 4% = 0,04

Assim: M= 2.500 ( 1 + 0,1 ) ( 1 + 0,04) = 2.500 x 1,1 x 1,04 = 2.860 Logo, o preo final de: R$ 2.860 EXERCCIO Operaes sobre mercadoria (II) 1) Por quanto devo vender um objeto que me custou R$ 40 para ganhar 15% sobre o custo? 2) Vendendo por R$ 56 um objeto que custou R$ 50, qual ser a percentagem de lucro? 3) Um objeto foi revendido por R$ 701, dando um prejuzo de 20% sobre o custo. Quanto havia custado? 4) Quanto por cento sobre o custo se perdeu ao se vender por R$ 238 um objeto que custou R$ 280? 5) Uma casa foi vendida por R$ 53.700, dando um lucro de 35% sobre o custo. Quanto havia custado/ 6) Calcule o preo de venda de um objeto que comprei por R$ 450, tendo uma perda de 15% sobre o preo de compra. 7) Calcule o preo de venda de um objeto comprado por R$ 84, para ganhar 30% sobre o preo de venda. 8) Calcule o preo de venda de um objeto que comprei por R$ 540, tendo perdido 20% do preo de venda. 9) Vendendo um imvel por 120.000, tive um prejuzo de 18% sobre o preo de venda. Por quanto comprei? 10) Vendi um objeto por R$ 280, com um lucro de 20% sobre o preo de venda. Qual o preo de compra? 11) Quanto por cento ganhei sobre o preo de venda de um objeto que me custou R$ 360 e foi vendido por R$ 450 12) De quanto por cento foi meu prejuzo sobre a venda de um objeto que me custou R$ 280 e foi vendido por R$ 250? 13)Vendi um objeto por R$ 120. Se tivesse vendido por mais R$ 20, meu lucro seria de 50% do preo da nova venda. Qual foi o meu lucro? 190

MATEMTICA PASSO A PASSO

14) Calcule o prejuzo de um comerciante que vendeu certas mercadorias por R$ 26.410, perdendo, nessa transao, a quantia equivalente a 5% sobre o preo de custo. 15) Se eu tivesse mais 50% da quantia que tenho poderia pagar uma dvida de R$ 5.000 e ainda ficaria com R$ 700. Quanto tenho? 16) Certa mercadoria foi vendida por R$ 3.232, com o prejuzo de 8,7% sobre o preo de compra. Por quanto deveria ser vendida para dar lucro de 12% sobreo seu preo de custo? 17) Em um exerccio de tiro ao alvo um soldado fez 40% a mais do que outro. Se os dois juntos fizeram 720 pontos, quanto fez cada soldado? 18) Calcule o liquido de uma duplicata no valor de R$ 8.600 que sofreu a reduo de 15% sobre esse valor total e, em seguida, outro abatimento de 8% sobre o lquido da primeira reduo. 19)comprei 2.000 kg de feijo, a R$ 1 o quilo; vendi 600 kg com um lucro de 25% sobre o preo de compra e o resto com 12% de lucro sobre o preo de venda da primeira parte. Calcule o lucro total 20) Sobre o preo de compra de uma mercadoria incide uma despesa de 15%. Por quanto devemos vender essa mercadoria, comprada por R$ 540, para que tenhamos um lucro de 25% sobre o preo de compra, repassando a despesa para o comprador? 21) Uma pessoa comprou um automvel de R$ 15.800 (preo de tabela) com desconto de 2,5%. No dia seguinte, vendeu o automvel pelo valor de 2% acima do preo da tabela. Qual foi a taxa percentual de lucro total dessa pessoa? 22) O que significa a expresso 4% dos 5% de uma grandeza? 23) Um comerciante comprou 450 unidades de um certo eletrodomstico, ao custo de R$ 420 a unidade. Vendeu 340 unidades com 30% de lucro. Depois vendeu o restante com certo prejuzo. Sabendo que a venda de todo estoque, nas condies acima, deixou R$ 38.660 de lucro lquido, calcule o preo pelo qual foi vendida, em cada caso, a unidade do eletrodomstico. 24) Um objeto foi vendido com 25% de lucro e outro com 30%. Por quanto foi vendido cada um, se os dois foram vendidos por R$ 2.142? 191

PROF. WELLINGTON BRITO

25) Um comerciante comprou vrias peas de tecido por R$ 38.200 e uma certa quantidade de arroz por R$ 29.000. Vendeu o tecido com 8% de prejuzo e o arroz com 12% de lucro. Ao todo, ganhou ou perdeu? Quantos por cento? 26) Um comerciante pagou 30% de uma dvida; do restaurante, pagou 20% e com R$ 28.000 liquidou a dvida. Determine o valor da dvida. 27) Um objeto foi vendido com 15% de prejuzo e outro com 35% de lucro. Por quanto foi vendido cada um, se os dois foram vendidos por R$ 748? 28) Certa mercadoria foi vendida por R$ 7.475, com lucro de 15%: em seguida, foi revendida por R$ 8.447. De quanto por cento foi o lucro final sobre o valor inicial dessa mercadoria? 29) Uma pessoa empregou seu capital, sucessivamente, em quatro empresas. Na primeira apurou 100% e em cada uma das outras perdeu 15%. Quanto ganhou sobre o capital primitivo? 30) sobre uma fatura de R$ 150.000 foram feitos descontos sucessivos de 8%, 5% e 2%. Qual o valor lquido da fatura? Respostas: 1) R$ 46 2) 12% 3)
R$ 876 11) 20% 12) 12% 13) R$ 50 14) R$ 1.390 21) 4,62% 22) 0,2% da Grandeza 23) 340und.a R$ 546 e110und a R$ 382 24) R$ 1.050 e R$ 1.092 25) Ganhou 0,63% 26) R$ 50.000

4) 15% 6) R$ 383 7) R$ 120 8) R$ 450 10) R$ 224

5) R$ 39.778 15) R$ 3.800


16) R$ 3.965

17) 420 e 300 27) R$ 289 e R$ 459 18) R$ 6.725 28) 29,95% 29) 22,825% 30) R$ 128.478

9) R$ 141.600 19) R$ 710


20) R$ 776

TEORIA DOS CONJUNTOS 192

MATEMTICA PASSO A PASSO

Conceitos Primitivos:

Na teoria dos conjuntos trs noes so aceitas sem definio, isto , so consideradas noes primitivas: a) conjunto b) elemento c) pertinncia entre elemento e conjunto Representao Relao De Pertinncia A noo matemtica de conjunto praticamente mesma que se usa na linguagem comum: o mesmo que agrupamento, classe, coleo, sistema. Eis alguns exemplos : 1) Conjunto das vogais 2) Conjunto dos algarismos romanos 3) Conjunto dos nmeros mpares positivos Cada membro ou objeto que entra na formao do conjunto chamado elemento. Assim, nos exemplos anteriores, temos os elementos: 1) a, e, i, o, u 2) I, V, X, L, C, D, M 3) 1, 3, 5, 7, 9, 11, ... Indicamos um conjunto, em geral, com uma letra maiscula, A, B, C,..., e um elemento com uma letra minscula, a, b, c, d, x, y,... . Sejam A um conjunto e x, um elemento. Se x pertence ao conjunto A, escrevemos: xA Para indicar que x no elemento do conjunto A, escrevemos: xA habitual representar um conjunto pelos pontos interiores a uma linhafechada e no entrelaada. Assim, na representao abaixo temos:
A

a b c

* Nota: No caso de usarmos um crculo para representar um conjunto, estaremos usando d o assim chamado diagrama de Euler-Venn.

a A, b A e d A Conjunto Unitrio - Conjunto Vazio

Chama-se conjunto unitrio aquele que possui um nico elemento. 193

PROF. WELLINGTON BRITO

Exemplos: 1) Conjunto dos divisores de 1, inteiros e positivos: {1} 2) Conjunto das solues da equao 3x + 1= 10: {3} Chama-se conjunto vazio aquele que no possui elemento algum. O smbolo usual para o conjunto vazio . Obtemos um conjunto vazio quando descrevemos um conjunto por meio de uma propriedade P logicamente falsa. Exemplos: 1) { x x x } = 2) { x x impar e mltiplo de 2 } = EXERCCIO CONJUNTOS ( I )

1. D os elementos dos seguintes conjuntos: A = { x x letra da palavra matemtica} B = { x x cor da bandeira brasileira } 2. Complete com (V) se verdadeiro ou (F) se falso as afirmaes: a) 0 ( ) b) 0 ( ) c) {1} { 1,2 } ( ) d) { 1, 2 } ( ) e) {1,2} {1, 2} ( ) f) { 1, } ( ) g) { 1 } { 1, {1} } ( h) { 1 } { 1, } ( i) { 1, {1} } ( ) ) )

3. Quais dos conjuntos abaixo so unitrios? 9 ex> 6 B = { x 0 . x = 2} 4 5 C = { x x inteiro e x2 = 3} D = { x 2x + 1 = 7 } 4. Quais dos conjuntos abaixo so vazios? A={x0.x=0} C = { x x divisor de zero } B = x x > 9 e x < 6 D = { x x divisvel por zero} A= x x < 4 5 Respostas: 1. A = { m, a, t, e, i, c } B = { Verde, Amarelo, Azul, Branco} 2. a) V b) V c) F d) F e) F f) V g) V h) F i)F 3. D = { 3 } 4. B = D = Conjuntos Iguais Dois conjuntos A e B so iguais quando todo elemento de A pertence a B e reciprocamente, todo elemento de B pertence a A. 194

MATEMTICA PASSO A PASSO

Em smbolos:

A = B ( x) (x x B )

Exemplos: 1) { a, b, c, d } = { d, c, b, a } 2) { 1, 3, 5,7,9, ...} = { x x inteiro, positivo e impar} 3) { x 2x + 1 = 5 } = { 2 } * Nota: Observemos que na definio de igualdade entre conjuntos no intervm a noo de ordem entre os elementos; portanto: { a, b, c, d } = { d, c, b, a} = { b, a, c, d} Subconjuntos Conjuntos Das Partes Incluso

Um conjunto A subconjunto de um conjunto B se, e somente se , todo B elemento de A pertence tambm a B. A Com a notao A B indicamos que A subconjunto de B ou A est contido em B ou A parte de B. * Notas: 1) Quando A B, tambm O smbolo denominado sinal de podemos escrever B A, Incluso. Em smbolos, a definio que l-se B contm A fica assim. 2) Com a notao A B A B (x) ( x A x B) indicamos que A no est contido em B, isto Exemplos: , a negao de A B. 1) {a,b} {a, b, c,d} 2) {a} {a, b} 3) {a, b} {a, b} Conjunto Das Partes 3) evidente que A B somente se existe ao menos um elemento de A que no pertence a B.

195

PROF. WELLINGTON BRITO

Dado um conjunto A, chama-se o conjunto das partes de A - notao P (A) aquele que formado por todos os subconjuntos de A. Em smbolos: Exemplos 1) Se A = {a}, os elementos de P (A) so e {a}, isto : P (A) = { , {a} }. 2) Se A = {a, b}, os elementos de P (A) so , {a},{b},{a, b} isto : P (A) = { , {a},{b},{a, b} }. 3) Se A = {a, b, c,}, os elementos de P (A) so , {a},{b}, {c}, {a,b}, {a,c},{b,c} e {a,b,c},isto : P (A) = { , {a},{b}, {c},{a, b},{b,c}, {c,a}, {a,b,c} }. *Nota: A quantidade de subconjuntos determinada pela potncia: n[ p (A) ] = 2n(A) Propriedades Da Incluso Sendo A, B e C trs conjuntos arbitrrios,valem as seguintes propriedades: 1) A 2) A A (reflexiva) 3) (A B e B A ) A = B (anti-simtrica) 4) (A B e B C ) A C (transitiva) P (A) = { x x A}

Questes Comentadas
1. Dados A = { 1, 2, 3, 4 } e B = { 2, 4 }, a) escreva com os smbolos da teoria dos conjuntos as seguintes sentenas: 1) 3 elemento de A 4) B igual a A 2) 1 no est em B 5) 4 pertence a B 3) B parte de A b) classifique as sentenas anteriores em falsa ou verdadeira. Resoluo: 1) 3 A ( V ) 3) B A ( V ) 5) 4 B ( V ) 2) 1 B ( V ) 4) B = A ( F ) 2. Sendo A = {1,2}, B = {2,3}, C= {1,3,4} e D = {1,2,3,4}, 196

MATEMTICA PASSO A PASSO

classifique em V ou F cada sentena abaixo e justifique: a) A D b) A B Resoluo: c) B C d) D B e) C = D f) A C e) F, pois 2 D e 2 C f) V, pois 2 A e 2 C

a) V, pois 1 A, 1 D, 2 A e 2 D b) F. pois 1 A e 1 B

c) F, pois 2 B e 2 C d) V, pois 2 B, 2 D, 3 B e 3 D EXERCCIO CONJUNTOS (II) Diga se verdadeira (V) ou falsa (F) cada uma das sentenas abaixo. a) 0 { 0, 1, 2, 3, 4 } b) {a} {a,b} c) {0} d) 0 e) {a} f) a {a, {a} } g) {a} {a, {a} } h) { , {a} } i) { , {a}} j) {a,b} {a,b,c,d}

Operaes Entre Conjuntos Unio Passo a Passo Dados dois conjuntos A e B, chama-se reunio de A e B o conjunto formado pelos elementos que pertencem a A ou a B. A B = { x x A ou x B }

* Nota: O conjunto A B (l-se A reunio B ou A B ) formado pelos elementos que pertencem a pelo menos um dos conjuntos A e B. Exemplos: 1) {a, b} {c,d} = {a,b,c,d} 2) {a,b} {a,b,c,d} = {a,b,c,d} 3) {a,b,c,} {c,d,e} = {a,b,c,d,e} Propriedades da reunio Sendo A, B e C conjuntos quaisquer, valem as seguintes propriedades: 1) A A = A ( idempotente) 2) A = A (elemento neutro)

3) A B = B A (comutativa) 4) (A B ) C = A (B C) (associativa)

197

PROF. WELLINGTON BRITO

Interseco Passo a Passo Dados dois conjuntos A e B, chama-se interseo de A e B o conjunto formado pelos elementos que pertencem a A e a B.

A B = { x x A e x B } * Nota: Se x A B, isso significa que x pertence a A e tambm x pertence a B. O conectivo colocado entre duas condies significa que elas devem ser obedecidas ao mesmo tempo. Exemplos: 1) {a,b,c,} { b,c,d,e} = {b,c} 2) {a,b} {a,b,c,d} = {a, b} 3) {a,b,c} {a,b,c} = {a, b, c} 4) {a, b} {c, d} =

Propriedades da interseo

Sendo A, B, e C conjuntos quaisquer, valem as seguintes propriedades: 1) A A = A 2) A U = A (idempotente) ( elemento neutro)

3) A B = B A ( comutativa ) 4) A ( B C) = (A B) C (associativa)

Questes Comentadas
1. Dados os conjuntos A = {1,2,3}, B= {3,4} e C = { 1,2,4}, determine o conjunto X tal que X U B = A U C e X B = . Resoluo a) X U B = { 1,2,3,4}, ento os possveis elementos de X so: 1, 2, 3 e 4. b) X B = 3 X e 4 X Concluso: X = {1,2} 2. Dados:A ={1,2} e B={2,3,4},determine o conjunto X tal que A X = {1}, B X = {3} e A U B U X = { 1,2,3,4,5}. ( A = {1,2} e A X = {1}) (1 X e 2 X ) (B = {2,3,4,} e B X = {3} ) (3 X, 2 X e 4 X) (A U B U X = { 1,2,3,4,5}, 5 A e 5 B) (5 X) Logo, X = {1,3,5} 198

MATEMTICA PASSO A PASSO

EXERCCIO CONJUNTOS ( III )

1) Dados os conjuntos A = {a,b,c}, B = {c,d} e C = {c, e}, determine A U B, A U C, B U C e A U B U C . 2) Dados os conjuntos A = {a,b,c,d,}, B = {b,c,d,e} e C = {c,e,f}, descreva A B, A C, B C e A B C. 3) Classifique em V ou F: a) (A U B) b) (A U B ) A d) (A U B ) (A U B) e) B (A U B)

c) A ( A U B) f ) (A U B) (A U B U C) admitindo que A, B e C so conjuntos quaisquer 4) Determine o conjunto X tal que: {a,b,c,d} U X = {a,b,c,d,e}, {c, d} U X = {a,c,d,e} e {b, c,d} X = {c} 5) Sabe-se que A U B U C = { n 1 n 10}, A B= {2,3,8}, A C = {2,7}, B C ={2,5,6} e A U B = { n 1 n 8}. Determine C. 6) Determine o nmero de conjuntos X que satisfazem a relao {1,2} X {1,2,3,4}. 7) Assinale no diagrama abaixo, um de cada vez, os seguintes conjuntos: a) A B C b) A (B U C) c) A U (B C) B A C

d) A U B U C 8) Sejam os conjuntos A com 2 elementos, B com 3 elementos, C com 4 elementos. Qual o nmero mximo de elementos de (AB)C? 9) Classifique em V ou F: a) (A B ) b) A (A B ) c) A (A B) d) (A B) (A B) e) (A B) B f) (A B) ( A B C) 199

PROF. WELLINGTON BRITO

admitindo que A, B e C so conjuntos quaisquer. Respostas: 3. a) V b) F c) F d) V e)V f)V 4. X = { a, c, e} 5. C = { 2, 5, 6, 7, 9, 10} 6. 4, so eles: { 1,2 }, { 1,2,3}, {1,2,4}, {1,2,3,4} 8. 2 elementos 9. a) V b) F f) V c) F d) V e)V

Diferena Passo a Passo

Dados dois conjuntos A e B, chama-se diferena entre A e B o conjunto formado pelos elementos de A que no pertencem a B. AB ={xxA e x B} Exemplos 1) {a,b,c} {b, c,d,e} = {a} 2) {a,b,c} {b,c} = {a} 3) {a,b} {c,d,e,f} = {a,b} 4) {a,b} {a,b,c,d,e} =

4) Complementar Passo a Passo Dados dois conjuntos A e B, tais que B A, chama-se Complementar de B em relao a A o conjunto A B, Isto , o conjunto dos elementos de A que no pertencem a B. Com o smbolo
B AB

OU B

Indicamos o complementar de B em relao a A * Nota


B

Temos que
A B

s definido para B A, a temos: = AB 200

MATEMTICA PASSO A PASSO

Exemplos 1) Se A = { a, b, c, d. e} e B = { c, d, e}, ento: A


A B B

= { a, b}

2) Se A = { a, b, c, d} = B, ento: =
B

3) Se A = { a, b, c, d} e B = , ento: A = { a, b, c, d } = A

Questes Comentadas
1) Dados A = {a, b, c,} , B = {a, c,e,f} e C = { d, e, f, g} e considerando o conjunto Universo U = {a,b,c,d,e,f,g,h} determine: a) Ac (B C) Resoluo a) Ac = U A = {d,e,f,g,h} B C = {e, f} Logo Ac ( B C) = {d,g,h} b) A U C = {a,b,c,d,e,f,g} B U C = {a,c,d,e,f,g} ( B U C) c = U (B U C) = {b,h} b) (A U C) ( B U C) c

Logo A U C ( B U C) c = {a,c,d,e,f,g} 2) No conjunto universo U = {a,b,c,d,e,f} consideremos o subconjunto A = {a,b,c}. Determinar o subconjunto X tal que A X = { b, c}, A X e Ac X = . Resoluo (A = { a,b,c}, A X = {b,c} e A X ) (b X, c X e a X) (Ac = {d,e,f} e Ac X = ) (d X, e X e f X ) Ento X = {a} U Em diagrama: A b x a c 201 d e f

Ac

PROF. WELLINGTON BRITO

3) Sendo conhecidos n (A) = nmero de elementos do conjunto A, n(B) = nmero de elementos do conjunto B e n (A B) = nmero de elementos de AB,determinar o nmero de elementos de AU B. Resoluo Notemos que A U B formado pelos elementos A
A (A B) B A B (A B)

que pertencem s a A: h n (A) n ( A B) elementos. que pertencem s a B: h n (B) n ( A B) elementos. que pertencem a A e a B: h n (A B) elementos. Ento: n(A U B)=[ n (A) n ( A B) ]+ [ n(B) n (A B)] + n (A B) n (A U B) = n (A) + n(B) - n (A B) 4) Num grupo de motoristas h 28 que dirigem carro, 12 que dirigem moto e 8 que dirigem carro e moto. Quantos motoristas h nesse grupo? Quantos s dirigem carro? Resoluo C = Conjunto dos que dirigem carro M = Conjunto dos que dirigem moto Nmero total de motoristas: n(C U M) = n(C) + n (M) n (C M) = 28 + 12 8 = 32 Nmero dos que dirigem s carro: n (C) n (C M) = 28 8 = 20 C Tambm podemos resolver o problema construindo M 8 o diagrama ao lado. Marcamos os 8 elementos 4 20 comuns, depois completamos o conjunto C (tem 28 elementos como 8 so comuns h mais 20 que pertencem s a C) e o conjunto M (tem 12 elementos como 8 so comuns h mais 4 que pertencem s a M) 5) Numa classe de 36 alunos temos: 19 jogam futebol, 25 jogam vlei, 13 jogam basquete, 12 jogam futebol e vlei, 8 jogam vlei e basquete, 8 jogam futebol e basquete e 4 praticam os trs esportes. Determine: a) quantos alunos da classe no praticam estes esportes? b) quantos praticam exatamente um destes esportes? c) quantos praticam exatamente dois destes esportes? 202 3 8 9 4 4 4 4

MATEMTICA PASSO A PASSO

Resoluo Construmos o diagrama comeando pelos 4 elementos que praticam os trs esportes ( F V B). Depois completamos F V (12 elementos), V B (8 elementos) e F B(8 elementos. Finalmente completamos F (19 elementos), V (25 elementos) e B ( 13 elementos). Como marcamos 4+8+4+4+3+9+1= 33 elementos e a classe tem 36 alunos, h 3 que no praticam nenhum dos esportes. As respostas So: a) 3 b) 3 + 9 + 1 = 13 c) 8 + 4 + 4 = 16 U F 3 9 V 8 4 4 4 1 3 B

EXERCCIO CONJUNTOS ( IV)

1) Seja E = { a, {a}}. Diga quais das proposies abaixo so verdadeiras. a) a E b) {a} E c) a E d) {a} E e) E f) E

2) Sejam os conjuntos A = {a, b, c, d}, B = {c,d,e,f,g} e C = { b,d,e,g}. Determine: a) A B c) C B e) A ( B C) b) B A d) (A U C) B f) (A U B ) (A C) 3) Dados os conjuntos A={1,2,3,4,5}, B={1,2,4,6,8} e C={2,4,5,7}, obtenha um conjunto X tal que X A e A X = B C. 4) Sendo A um conjunto qualquer, determine: a) A A a)BCA b) A b) ) C Bc c) A c) C Ac 5) Dados A = {1,2,3}, B = {1,2,3,4,5} e C = {2,3} determine:

203

PROF. WELLINGTON BRITO

6) Sombreie o conjunto pedido em cada diagrama a) A (B C)


A

b) A (B U C) B
A

B C

7) Sombreie o conjunto pedido em cada diagrama. a) (A B) U ( B A ) A B b) ( A U B ) ( A B) A B

8) Denominamos diferena simtrica dos conjuntos A e B ao conjunto A B ( leia: A delta B) dado por A B = (A B ) U ( B A ). Dados A={0,3,6,9,12,15,18} e B = { 0,2,4,6,8,10,12} determine A B. 9) Considere no conjunto universo U = {1,2,3,4,5,6,7,8,9,10} os subconjuntos A = {2,3,5,7} e B = {1,3,5,7,9}. Determine: a) Ac b) Bc c) (A B) c d) (A U B) c 10)Classifique em verdadeiro ou falso, supondo que A e B so subconjuntos quaisquer de um universo U: a) A B = A Bc b) A B c = A B c) Ac B c = B A d)O complementar de Ac A, isto (Ac) c = A. e) (A B) c = (A B c ) c = Ac U B.

11) Em uma escola que tem 415 alunos, 221 estudam ingls, 163 estudam francs e 52 estudam ambas as lnguas. Quantos alunos estudam ingls ou francs? Quantos alunos no estudam nenhuma das duas? 12) Uma populao consome trs marcas de sabo em p: A, B e C. Feita uma pesquisa de mercado, colheram-se os resultados tabelados abaixo:
Marca nmero de A B C A e B B e C C e A A, B e C Nenhuma das trs 41 28 5 115

consumidores 109 203 162 25

204

MATEMTICA PASSO A PASSO

Fornea: a) o nmero de pessoas consultadas b) o nmero de pessoas que s consomem a marca A; c) o nmero de pessoas que no consomem as marcas A ou C; d) o nmero de pessoas que consomem ao menos duas marcas 13) Em certa comunidade h indivduos de trs raas: branca, preta e amarela. Sabendo que 70 so brancos, 350 no so pretos e 50% so amarelos, responda: a) quantos indivduos tem a comunidade? b) quantos so os indivduos amarelos? 14) De todos os empregados de uma firma, 30% optaram por um plano de assistncia mdica. A firma tem a matriz na capital e somente duas filiais, uma em Santos e outra em Campinas. 45% dos empregados trabalham na matriz e 20% dos empregados trabalham na filial de Santos. Sabendo que 20% dos empregados da capital optaram pelo plano de assistncia mdica e que 35% dos empregados da filial de Santos o fizeram, qual a porcentagem dos empregados da filial de Campinas que optaram pelo plano? 15) Determine os conjuntos A, B, e C que satisfazem as seguintes seis condies: 1) A U B U C = { z,x,v,u,t,s,r,q,p} 2) A B = {r,s} 3) B C = {s, x} 4) C A = {s, t} 5) A U C = {p,q,r,s,t,u,v,x} 6) A U B = {p,q,r,s,t,x,z} 16) Considerando os conjuntos A, B e C, representados ao lado, e sabendo que: n (A U B) = 24 n (A B) = 4 n (B U C) = 16 n (A C ) = 11 n (B C ) = 10, calcule: a) n (A B) b) n (A B C) c) n (B ( C U A) ) d) n ((A B) C) e) n ( B (A B) ) 205 A

C B

PROF. WELLINGTON BRITO

Respostas 1) a, b, d, f 2) a) {a,b} b) {e, f, g} c) {b} d) {a,b} e) { a, b, c} f) {a,c,e,f,g} 3) X = {1,3,5} 4) 5) 8) a) b) A c) a) {4,5} b) {1,4,5} c {1} A B = {2,3,4,8,9,10,15,18} c){1,2,4,6,8,9,10} d){4,6,8,10}

9) a){1,4,6,8,9,10} b){2,4,6,8,10} 11) 332 e 83

10) a) V b) V c) V d) V e) V 12) a) 500 b) 61 c) 257 d) 84 13) a) 560 b) 280 14) 40% 15) A = { p,q,r,s,t } B = { r,s,x,z } C = { s,t,u,v,x } 16) a) 8 b) 1 c) 7 d) 3 e) 12

206

MATEMTICA PASSO A PASSO

QUESTES DE CONCURSOS E VESTIBULARES


01) (Guarda-M) Numa academia de ginstica foi feita uma pesquisa para saber o nmero de pessoas matriculadas em alongamento (A), hidroginstica (H) e musculao (M), chegando-se ao seguinte resultado:
Atividade Fsica Nmero de Pessoas A H M AeH 25 AeM 28 HeM 41 A,H e M 5 Outras Atividades 115

109 203 162

Com base nessas informaes, pode-se concluir que a pesquisa foi feita com: a) 500 pessoas b) 573 pessoas c) 600 pessoas d) 688 pessoas 02) (UECE) Se A um conjunto finito, seja n(A) o nmero de elementos de A. sejam X, Ye Z trs conjuntos tais que: n ( X ) = 100, n (Y) = 90, n (z) = 80, n (X ( Y U Z) ) = 50, n ( X Y Z) = 10 e n ( X Y ) = n( X Z) = n (Y Z) Nestas condies o nmero de elementos que pertencem a mais de um conjunto : a) 70 b) 80 c) 90 d) 100 03) (FGV) Dados os conjuntos A = {a,b, c, d}, B = {b,c,d,e}, C= {a, c, f}, ento [(A B) U (B C) U (A B)] [ (A C) U (B A C)] : a) { a, b,c,d,e} b) { a, b, c,d} c) {a,c} 04) (FATEC) Assinale a alternativa verdadeira. Se A e B so dois conjuntos, no vazios, e o conjunto vazio, ento: a) { x x A e x B} = A U B b) B ( A B) c) A = {} d) B A = x implica C B A = x e) A A B 05) (CESGRANRIO) Sejam os conjuntos U = { 1,2, 3, 4} e A = {1, 2 }. O conjunto B tal que B A = {1} e B A = U : a) b) {1} c) {1,2} 207 d) {1,3,4} e) U d) {a, b} e) {b, c, d}

PROF. WELLINGTON BRITO

06) (PUC-RS) Dados os conjuntos A = { a,b,c}, B = {a,d} e C = {a,b,d}, o conjunto X tal que A U C = B U X e B X = : a) {a} b) {b} c) {c} d) {a,b} e) {b,c} 07) (U.F.RS) O conjunto A subconjunto de B e A B, A U (B A) : a) B b) A c) d) A B e) A B 08) (U.F.RN) A parte hachurada do grfico abaixo corresponde a: a) ( A B) B A b) (A C) B B c) (B C) A d) (A C) A e) (A B) C C 09) (F.SANTANA) Na figura abaixo, esto representados os conjuntos A, B e C no vazios. A regio sombreada representa o conjunto: a) (A B) C b) ( A U B U C) C c) ( A B) C d) (B U C) A e) A B C C A B

10)(U.F.PE) Considere os seguintes conjuntos: A = {1,2, {1,2 }} a) A B = { 2 } b) B C = {{1}} c) B C = A B d) B A e) A P (A) = {{1,2}}, onde P (A) o conjunto dos subconjuntos de A B= {{1}, 2} e C = {1,{1}, {2}} Assinale abaixo a alternativa falsa:

208

MATEMTICA PASSO A PASSO

11) (FGV) Numa pesquisa de mercado, foram entrevistadas vrias pessoas acerca de suas preferncias em relao a 3 produtos:A,B e C. Os resultados da pesquisa indicaram que: 210 pessoas compram o produto A. 210 pessoas compram o produto B. 250 pessoas compram o produto C. 20 pessoas compram os 3 produtos. 100 pessoas no compram nenhum dos 3 produtos. 60 pessoas compram os produtos A e B. 70 pessoas compram os produtos A e C. 50 pessoas compram os produtos B e C. Quantas pessoas foram entrevistadas ? a) 670 b) 970 c) 870 d) 610 e) 510 12) (FGV) No problema anterior, calcular quantas pessoas compram apenas o produto A; apenas o produto B; Apenas o produto C. a) 210, 210; 250 b) 150; 150; 180 c) 100;120;150 d) 120;140;170 e) n.d.a

13) (FGV) Numa universidade com N alunos, 80 estudam Fsica, 90 Biologia, 55 Qumica, 32 Biologia e Fsica, 23 Qumica e Fsica, 16 Biologia e Qumica e 8 estudam nas trs faculdades.Sabendo-se que esta Universidade somente mantm as trs faculdades, quantos alunos esto matriculados na Universidade? a) 304 b) 162 c) 146 d) 154 e) n.d.a. 14) (PUC) Em um exame vestibular, 30% dos candidatos eram da rea de Humanas. Dentre esses candidatos, 20% optaram pelo curso de Direito. Do total dos candidatos, qual a porcentagem dos que optaram por Direito? a) 50% b) 20% c) 10% d)6% e) 5% 15) (PUC) Dentre os inscritos em um concurso pblico, 60% so homens e 40% so mulheres. J tm emprego 80% dos homens e 30 % das mulheres. Qual a porcentagem dos candidatos que j tm emprego? a) 60% b) 40% c)30% c) x C A C B d) x A C B 209 d) 24% e) 12% 16) (PUC) sejam A, B U. Se x C (A B), ento: a) x A B b) X A B e) x C A B

PROF. WELLINGTON BRITO

17) (G.Municipal) Um conjunto A possui o dobro do nmero de elementos de um conjunto B e o conjunto B possui mais elementos que o conjunto C. Sabendo-se que o conjunto A possui x subconjuntos a mais que o conjunto B e que o conjunto B possui 15 subconjuntos a mais que o conjunto C, o valor de x : a) 60 b) 120 c) 180 d) 240

18) (F.Carlos Chagas-SP) Se A = { ; 3;{3}; {2;3}}, ento: a) { 2 ;3} A b) 2 A c) A d) 3 A e) {3} A

19) (MACK-SP) Seja o conjunto A = {3, {3}} e as proposies: (1)3A ento: a) apenas (1) e (2) so verdadeiras b) apenas (2) e (3) so verdadeiras c) apenas (1) e (3) so verdadeira d) todas as proposies so verdadeiras. e) nenhuma proposio verdadeira. ( 2 ) {3} A ( 3 ) {3} A

20) (FEC-SP) Dados os conjuntos: M = {3;5;6;}, N = {5;6;7} e P={6;7;8}, podemos afirmar que: a) M N = b) 8 P c) 3 M N d)n.d.a.

21) (UF-Uberlndia) Se A = {3;4;5;6;} e B {7;8;9}, ento: a) {7} B b) A B = {} d) A B = { x R 3 x 9} c) {5;6} A e) BA = B

22) (MACK-SP) Sendo A = {{1}, {2}, {1,2}} pode-se afirmar que: a) {1} A b) {1} A c) {1} {2} A d) 2 A e){1} {2} A

23)(FUNVEST-SP) Seja A B a diferena simtrica dos conjuntos A e B, definida pela igualdade: A B = ( A B ) (B A ). Se A = {a,b,c} e B {b,c,d,e,f} ento A B o conjunto: a) {a,d,e,f} b) {b,c,d,f} c) d) {a} e) A B

210

MATEMTICA PASSO A PASSO

24) (UFOP MG) Numa sala de aula com 60 alunos, 11 jogam xadrez, 31 so homens ou jogam xadrez e 3 mulheres jogam xadrez. Conclui-se, portanto que: a) 31 so mulheres d) 23 homens no jogam xadrez. b) 29 so homens e) 9 homens jogam xadrez. c) 29 mulheres no jogam xadrez 25) (STA.Casa-SP) Feito exame de sangue em um grupo de 200 pessoas, constatou-se o seguinte: 80 delas tm sangue com fator Rh negativo, 65 tm sangue tipo O e 25 tm sangue tipo O com fator Rh negativo. O nmero de pessoas com sangue de tipo diferente de O e com fator Rh positivo : a) 40 b) 65 c) 80 d) 120 e) 135 26) (UFC) Depois de n dias de frias, um estudante observa que: (1) Choveu 7 vezes, de manh ou tarde; (2) quando chove de manh no chove a tarde; (3) houve 5 tardes sem chuva; (4) houve 6 manhs sem chuva; Ento n igual a: a) 7 b) 9 c) 10 d) 11 e) 13

27) (UNIFOR) Relativo ao conjunto X = {, {1}, {1,2}, {1,2,3}} e seus elementos, correto afirmar que: a) {1} X c) {1,2} [1,2,3} e) 1 X b) X d) {1,2,3} X 28) (UNIFOR) Um conjunto X tem 32 subconjuntos, dois dos quais so: A = {a,b,c } e B = { a, b,d,e }. Nessas condies, verdade que: a) X B = {a,b} b) X = {a, b,c,d} c) X = A B d) X = B A e) X A = { d, e} Respostas: 01) A 02) A 03) C 04) D 05) D 06) E 07) A 08) B 09) A 10) D 11) D 12) C 13) B 14) D 15) A 16) C 17) D 18) E 19) D 20) D 21) C 211 22) E 23) A 24) C 25) C 26) B 27) B 28) E

PROF. WELLINGTON BRITO

RACIOCNIO LGICO - QUANTITATIVO O Estudo da Lgica Na Grcia antiga, h mais de dois mil anos, viveram inmeros pensadores cujas idias permanecem vivas at os dias de hoje. Aristteles, que viveu no sculo IV antes de Cristo , foi um deles. Esse filsofo pode ser considerado o primeiro a se preocupar com o estabelecimento de regras para Proposio. Ele fez um estudo minucioso de certos tipos de proposies, estabelecendo Regras para distinguir os que so verdade daqueles que no o so. Estudar matemtica pode ser um exerccio permanente de Lgica. Cada proposio que fazemos, por mais complicada que parea, pode ser sempre justificada a partir de outras mais simples, encandeadas adequadamente. Proposio Chama-se proposio ou sentena toda orao declarativa que pode ser classificada em verdadeira ou falsa. Observemos que toda proposio apresenta trs caractersticas obrigatrias: 1) sendo orao, tem sujeito e predicado; 2) declarativa ( no exclamativa nem interrogativa ); 3) tem um, e somente um, dos dois valores lgicos: ou verdadeira (V) ou falsa (F). Exemplos So proposies a) b) c) d) e) Nove diferente de cinco. (9 5) Sete maior que trs. ( 7 > 3) Dois um nmero inteiro. ( 2 ) Trs divisor de onze. ( 3/11 ) Quatro vezes cinco igual a vinte. ( 4 x 5= 20)

Dessas proposies, todas so verdadeiras exceto d. No so consideradas proposies as frases: f) g) h) Trs vezes cinco mais um. ( 3 x 5 + 1) A raiz quadrada de dois um nmero racional? (2 Q ?) O triplo de um nmero menos um igual a onze. ( 3x 1 = 11) 212

MATEMTICA PASSO A PASSO

A frase f no tem predicado, a frase g interrogativa e a frase h no pode ser classificada em verdadeira ou falsa. Negao A partir de uma proposio p qualquer, sempre podemos construir outra, denominada negao de p e indicada com o smbolo ~ p. Exemplos a) p: Nove diferente de cinco (9 5) ~ p: Nove igual a cinco ( 9 = 5) b) p: Sete maior que trs. ( 7 > 3) ~ p: Sete menor ou igual a trs ( 7 3) c) p: Dois um nmero inteiro. ( 2 ) ~ p: Dois no um nmero inteiro. (2 ) d) p: Trs divisor de onze: ( 3/11 ) ~ p: Trs no divisor de onze. ( 3 11) e) p: Quatro vezes cinco igual a vinte. ( 4 x 5= 20) ~ p: Quatro vezes cinco diferente de vinte. ( 4 x 5 20) Para que ~ p seja realmente uma proposio devemos ser capazes de classifica-la em verdadeira (V) ou falsa (F). Para isso vamos postular (decretar) o seguinte critrio de classificao: A proposio ~ p tem sempre o valor oposto de p, isto , ~ p verdadeira quando p falsa e ~ p falsa quando p verdadeira. Esse critrio est resumido na tabela ao lado, denominada tabela-verdade da proposio ~ p Assim, reexaminando os exemplos anteriores, temos que ~ p verdadeiro no exemplo d e ~ p falsa nos demais.

~p F V

V F

EXERCCIO RACIOCNIO LGICO ( I )

1) Quais das sentenas abaixo so proposies? No caso das proposies, quais so verdadeiras? a) 5 x 4 = 20 b) 5 4 = 3 c) 2 + 7 x 3 = 5 x 4 + 3 d) 5(3 + 1) = 5 x 3 + 5 x 1 213 e) 1 + 3 1 + 6 f) ( 2)5 ( 2)3 g) 3 + 4 > 0 h) 11 4 x 2

PROF. WELLINGTON BRITO

2) Qual a negao de cada uma das seguintes proposies? Que negaes so verdadeiras? a) 3 x 7 = 21 b) 3 x (11 7 ) 5 c) 3 x 2 + 1 > 4 d) 5 x 7 2 5 x 6 Respostas 1) So proposies: a, b, c, d, e f, g So verdadeiras: a, c, d, e,g 2) a) 3 x 7 21( F) e) f)

() ()
1 7< 1 2 2 2 < 1

h) 3 7

g) ( 4 ) 7

d) 5 x 7 2 > 5 x 6 (V) e) f) 2 1 (V) g) ( 4) < 7 (V)

() ()
1 7 2 h) 3

1 3(F) 2

()

b) 3 (11 7 ) = 5 ( F ) c) 3 x 2 + 1 4 (F) Os Quantificadores

7 ( V)

Em relao ao conjunto A = { 6, 8, 9, 10, 12} podemos dizer que: qualquer que seja o elemento de A, ele um nmero natural, existe elemento de A que um nmero par, existe um nico elemento de A que impar. No existe elemento de A que nmero primo. Em Matemtica dispomos de smbolos prprios para representar as expresses grifadas acima. Estes smbolos, chamados quantificadores, so os seguintes: ( leia: qualquer que seja) ( leia: existe) ( leia: existe um nico) ( leia: no existe) Colocando-se x A ao lado de cada um deles, temos: x A : qualquer que seja x pertencente a A, ( ou para todo x pertencente a A) x A : existe x pertencente a A. 214

x A: existe um nico x pertencente a A. x A : no existe x pertencente a A.

MATEMTICA PASSO A PASSO

Ento, no caso do conjunto A = { 6,8,9,10,12} temos: x A, x natural. x A x par. x A x mpar. x A x primo. J a sentena (x A, x par) falsa, porque 9 A e 9 no par. Em outras palavras a sentena (x A, x par) falsa porque (x A x no par) verdadeira. Dizemos que a sentena (x A x no par) a negao lgica da sentena (x A, x par). Quando uma sentena a negao lgica da outra, sendo uma delas verdadeira a outra falsa. A negao lgica de uma sentena tipo (x, x tem a propriedade P) a sentena (xx no tem a propriedade P). Por exemplo, a negao de todo sorvete gostoso existe sorvete que no gostoso. A negao lgica de uma sentena do tipo ( x x tem a propriedade P ) a sentena ( x tem a propriedade P) ou, equivalentemente, (x, x no tem a propriedade P). por exemplo, para negar que existe menino de cabelos verdes, podemos dizer no existe menino de cabelos verdes ou nenhum menino tem cabelos verdes, ou ainda, todo menino no tem cabelos verdes. Implicao e Equivalncia Se for verdade que todo brasileiro entende de futebol, ento tambm verdade que todo maranhense entende de futebol (porque, afinal, os maranhenses tambm so brasileiros). Isto significa que da afirmativa a, todo brasileiro entende de futebol,podemos tirar como concluso b: todo maranhense entende de futebol. ( lgico que tambm podemos tirar outras concluses como, por exemplo, todo paulista entende de futebol, todo gacho entende de futebol e at todo carioca entende de futebol.). Quando de uma afirmao a podemos tirar uma concluso b dizemos que a implica b. Indicamos: a b ( leia: a implica b, ou se a ento b) Se tambm de b podemos tirar como concluso a, dizemos que a e b so equivalentes. Neste caso indicamos: 215

a b ( leia: a equivalente a b, ou a se e somente se b) Exemplos: 1) sendo x um nmero inteiro, que pode ser positivo, nulo ou negativo, temos que: x = 2 x2 = 4 Notemos que de x2 = 4 no podemos tirar a concluso de que x = 2 (porque poderamos ter x = - 2). Assim x 2 = 4 no implica x = 2 , logo x2 = 4 no equivale a x = 2. Quando a no implica b escrevemos: a b ( leia: a no implica b) Quando a no equivale a b escrevemos: a b ( leia: a no equivale a b) 2) Imaginemos agora que E um subconjunto de um conjunto F e seja x um elemento qualquer. Podemos afirmar que se x E, ento x F. xExF F X
E

PROF. WELLINGTON BRITO

E tambm podemos afirmar que se x F, ento x E. x FxE A afirmativa x F a negao de x F. Costumamos representar a negao de uma afirmativa a pelo smbolo ~ a ( leia : no a). De modo geral, quando a b Tambm temos que : ~ b ~ a E
F

Na verdade, vale a equivalncia: ( a b ) (~ b ~ a) 3) Com os smbolos estudados podemos escrever as definies de subconjuntos e da igualdade de conjuntos como segue: A B x, ( x A B) x baiano x brasileiro A=B(ABeBA) x no brasileiro x no baiano E temos tambm que: A B x x A e x B
brasileiros baianos

216

MATEMTICA PASSO A PASSO

EXERCCIO RACIOCCIO LGICO ( II ) 1) Sendo A = { 2,3,5,7,11,13,17,19}, classifique em verdadeiro ou falso: a)x A, x menor que 20. f)x,(xAx maior que 10). b)x,(xA x nmero primo).g) x A x maior que 10. c) x Ax impar. h) x A x maior que 10. d) x A x par. i) x A x negativo. e) x Ax maior que 10. j)x,(x nmero primoxA). 2) Seja A o conjunto de todos os cariocas e B o conjunto de todas as pessoas inteligentes. Admitindo que verdadeira a frase todo carioca inteligente, como se representam num diagrama os conjuntos A e B? 3) A negao da sentena A B (todo elemento de A pertence a B) a sentena A B (existe elemento de A que no pertence a B). Ento, qual a negao da frase todo carioca inteligente ? 4) considerando os conjuntos A e B do exerccio 2, e supondo que exista carioca que no inteligente podemos ter os seguintes casos: I II III A B A B A B

Associe cada caso a uma das seguintes sentenas: a) Nenhum carioca inteligente b) Existe carioca inteligente, carioca no inteligente e inteligente que no carioca c) Existe carioca no inteligente, mas todo inteligente carioca 5) Sendo a e b nmeros quaisquer,classifique em verdadeiro ou falso: a) a b = 0 a = b b) a + b = 0 (a = 0 e b 0) . 6) D a negao (lgica) de cada sentena. a) Existe menina feia. b) Todo menino gosta de futebol. 217

PROF. WELLINGTON BRITO

c) Nenhuma menina gosta de futebol. d) Tudo que bom engorda. 7) Em todo sbado que no chove, Ricardo anda de bicicleta. Se no sbado passado Ricardo no andou de bicicleta, o que voc pode concluir? 8) Considere a afirmativa a: todo aluno que gosta de Matemtica tambm gosta de poesia. a) Qual a negao lgica de a? b) Se a verdadeira, o que se pode concluir a respeito de um aluno que no gosta de poesia? c) Se a verdadeira e Adriana no gosta de Matemtica, pode-se concluir que Adriana no gosta de poesia? Respostas 1) a) V b) V e) F f) F i) V j) F c) V g) V d) V h) F 2) B

3) Existe carioca que no inteligente. 4) a) II 5) a) V b) I b) F c) III

6) a) Nenhuma menina feia. b) Existe menino que no gosta de futebol (ou tambm: nem todo menino gosta de Futebol.) c) Existe menina que gosta de futebol. d) Nem tudo o que bom engorda. (ou tambm: existe o que bom e no engorda.) 7) Sbado passado choveu. 8) a) Existe aluno que gosta de Matemtica e no gosta de poesia. b) Que o aluno no gosta de Matemtica c) No. 218

MATEMTICA PASSO A PASSO

Os Conectivos Passo a Passo

A partir de proposies dadas podemos construir novas proposies mediante o emprego de dois smbolos lgicos chamados conectivos: Conectivos (l-se: e) e o conectivo (l-se: ou). Conectivo v ( ou ) Colocando o conectivo v entre duas proposies p e q, obtemos uma nova proposio, p v q, denominada disjuno das sentenas p e q. Exemplos 1) p: 5 > 0 ( cinco maior que 0) q: 5 > 1 ( cinco maior que um) p q: 5 > 0 ou 5 > 1(cinco maior que zero ou maior que um) 2) p: 3 = 3 ( trs igual a trs) q: 3 < 3 ( trs menor que trs) p q: 3 3 (trs menor ou igual a trs) 3) p: 10 um nmero primo q: 10 um nmero composto q q: 10 um nmero primo ou composto 4) p: 34 < 26 q: 22 < ( - 3 )5 p q: 34 < 26 ou 22 < ( - 3 )5 Vamos postular um critrio para decidir o valor lgico (V ou F) de uma disjuno a partir dos valores lgicos (conhecidos) das proposies p e q: A disjuno p q verdadeira se ao menos uma das proposies p ou q verdadeira; se p e q so ambas falsas, ento p q falsa. Esse critrio est resumido na tabela ao lado, Denominada tabela-verdade da proposio p q. p V V F F q V F V F pq V V V F

219

PROF. WELLINGTON BRITO

Revendo os exemplos anteriores, temos: 1) p: 5 > 0 ( V ) q: 5 > 1 ( V ) ento: p q: 5 > 0 ou 5 > 1(V) 2) p: 3 = 3 ( V ) q: 3 < 3 ( F ) ento: p q: 3 3 ( V ) 3) p: 10 nmero primo ( F ) q: 10 nmero composto ( V ) ento: p q: 10 nmero primo ou composto ( V ) 4) p: 34 < 26 (F) q: 22 < ( - 3 )5 ( F ) ento: p q: 34 < 26 ou 22 < ( - 3 )5 ( F ) Conectivo ( e ) Colocando o conectivo, entre duas proposies p e q, obtemos uma nova proposio, p q, denominada conjuno das sentenas p e q. Exemplos 1) p: 2 > 0 q: 2 1 p q: 2 > 0 e 2 1 2) p: 2 < 1 q: ( 2)2 < ( 1)2 p q: 2 < 1 e ( 2)2 < ( 1)2 3) p: um quadrado de lado a tem diagonal 2a q: um quadrado de lado a tem rea a2 p q: um quadrado de lado a tem diagonal 2a e rea a2 4) p: 2 / 5 ( 2 divisor de 5) q: 3 / 5 ( 3 divisor de 5) p q: 2 / 5 e 3 / 5 ( 2 e 3 so divisores de 5) Vamos postular um critrio para estabelecer um valor lgico ( V ou F) de uma conjuno a partir dos valores lgicos(conhecidos ) das proposies p e q: A conjuno p q verdadeira se p q so ambas 220

verdadeiras; se ao menos uma delas for falsa, ento p q falsa. Esse critrio est resumido na tabela ao lado, p q p q em que so examinadas todas as possibilidades para p e q. V V V Essa tabela denominada V F F tabela-verdade da proposio p q. F V F F F F Reexaminando os exemplos anteriores, temos: 1) p: 2 > 0 (V) q: 2 1 (V) ento: p q: 2 > 0 e 2 1 ( V ) 2) p: 2 < 1 (V) q: ( 2)2 < ( 1)2 ( F ) ento: p q: 2 < 1 e ( 2)2 < ( 1)2

MATEMTICA PASSO A PASSO

(F)

3) p: um quadrado de lado a tem diagonal 2a ( F ) q: um quadrado de lado a tem rea a2 (V) Ento:p q: um quadrado de lado a tem diagonal 2a e rea a2 ( F ) 4) p: 2 / 5 ( F ) q: 3 / 5 (F) ento: p q: 2 / 5 e 3 / 5 Questes Comentadas Admitindo verdadeiras as premissas: (1) O professor no erra. (2) Joo distrado. (3) Quem distrado erra.

(F)

Classifique em V ou F as seguintes concluses: a) Joo no professor. b) Nenhum professor distrado. Resoluo P = Conjunto dos professores D = Conjunto dos distrados E = Conjunto dos que erram (1)PE= ( 2 ) Joo D (3)DE 221
P

Joo D

a) V (pois D P = e Joo D, logo Joo P ) b) V ( pois D P = ) EXERCCIO - RACIOCNIO LGICO ( III ) 1) Classifique em verdadeira ou falsa cada uma das seguintes proposies compostas: a) 3 > 1 e 4 > 2 e) 1 < 3 ou 5 b) 3 > 1 ou 3 = 1 2 4 11 Respostas: c) 2/4 ou 2/(4 + 1) d) 3( 5 + 2 ) = 3 x 5 + 3 x 2 e 3/7 f) ( 1)6 = 1 e 25 < ( 2)7 g) 16 = 6 ou mdc (4,7) = 2

PROF. WELLINGTON BRITO

2) Partindo das premissas: (1) Todo reprter esperto. (2) Todo reprter formado em jornalismo. (3) Jamil esperto. (4) Adelaide jornalista. Pode-se concluir que: a) Adelaide esperta? b) Jamil reprter? c) H jornalistas espertos?

3) Forme a negao de cada frase. a) Juliana alta e loira b) Sandro pratica natao e corrida. 4) Forme a negao de cada sentena. a) Osmar palmeirense ou vascano b) Simone gosta de ler ou de ouvir msica. 5) (GV-SP) Um grupo de 4 pessoas ser formado, escolhendo-se entre 3 homens ( F,G,H) e 4 mulheres ( W, X,Y,Z). O grupo dever ter pelo menos 2 homens e as seguintes condies devero ser respeitadas: F se recusa a trabalhar com Y G se recusa a trabalhar com W Y se recusa a trabalhar com Z a) Se Y pertence ao grupo, quais sero os outros membros? b) Classifique em Verdadeiro ou Falso: I Se F no escolhido, W tambm no o . II Se H no escolhido, Z o . III Se G no escolhido, W o . Respostas:
1) a) V b) V c) V d)F e)V f)F g) F 2) a) no b) no c) sim 3) a) Juliana no alta ou no loira. b) Sandro no pratica natao ou no pratica corrida. 4) a) Osmar no palmeirense e no vascano.

222

MATEMTICA PASSO A PASSO

b) Simone no gosta de ler e no gosta de ouvir msica. 5) a) 6, H e X b) I) V II) V III) F.

Os Condicionais Passo a Passo

Ainda a partir de proposies dadas podemos construir novas proposies mediante o emprego de outros dois smbolos lgicos chamados condicionais: o condicional se...ento... (Smbolo: ) e o condicional ... se, e somente se, ....( Smbolo ). Condicional ( se... ento...)

Colocando o condicional entre duas proposies p e q, obtemos uma nova proposio, p q, que l-se: se p, ento q, p condio necessria para q, q condio suficiente para p. No condicional p q, a proposio p chamada antecedente e q chamada conseqente. Exemplos 1) p: dois divisor de quatro ( 2/4 ) q: quatro divisor de vinte ( 4/20 ) p q: se dois divisor de quatro, ento quatro divisor de vinte (2/4 4/20) 2) p: dois vezes cinco igual a dez ( 2 x 5 = 10) q: trs divisor de dez (3/10) p q: se dois vezes cinco igual a dez, ento trs divisor de dez (2x5=10 3/10) 3) p: cinco menor que dois ( 5 < 2) q: dois nmero inteiro ( 2 ) p q: se cinco menor que dois, ento dois nmero intero (5 < 2 2 ) 4) p: um meio menor que um tero q: trs igual a cinco ( 3 = 5) p q: se um meio menor que um tero, ento trs igual a cinco 1 < 1 3=5 2 3 Vamos postular um critrio de classificao para a proposio p q baseado nos valores lgicos de p e q.

1 < 1 2 3

O condicional p q falso somente quando p verdadeira e q falsa; caso contrrio, p q verdadeiro. 223

PROF. WELLINGTON BRITO

Esse critrio est resumido na tabela ao lado, denominada tabela-verdade da proposio p q. Revendo os exemplos dados temos: 1) p V e q V, ento p q V 2) p V e q F, ento p q F 3) p F e q V, ento p q V 4) p F e q F, ento p q V

pq V F V V

V V F F

V F V F

Condicional ( se, e somente se)

Colocando o condicional entre duas proposies p e q, obtemos uma nova proposio, p q, que l-se: p se, e somente se, q, p condio necessria e suficiente para q, q condio necessria e suficiente para p ou se p, ento q e reciprocamente. Exemplos 1) p: 2 / 12 q: 2 x 7 /12 x 7 p q: 2/12 2 x 7/12 x 7 3) p: 6 = 12 : 3 q: 3.6 = 18 p q: 6 = 12 : 3 3 x 6 = 18

2) p: 3 = 6 4) p: 4 3 2 4 q: 4 x 5 3 x 5 q: 3 x 4 6 x 2 p q: 4 3 4 x 5 3 x 5 p q: 3 = 6 3 x 4 6 x 2 2 4 Vamos postular para o condicional classificao: p q o seguinte critrio de

O condicional verdadeiro somente quando p e q so ambas verdadeiras ou ambas falsas; se isso no acontecer, o condicional falso. Assim a tabela-verdade da proposio p q: a que est ao lado. Revendo os exemplos dados temos: 1) p V e q V, ento p q V 2) p V e q F, ento p q F 224 p V V F F q V F V F

pq
V F F V

3) p F e q V, ento p q F 4) p F e q F, ento p q V EXERCCIO RACIOCNIO LGICO ( IV ) 1) Escreva a tabela-verdade contendo duas proposies p e q com possveis resultados dos conectivos e condicionais. 2) Classifique em verdadeira ou falsa cada uma das proposies abaixo. a) 2 1 = 1 5 + 7 = 3 x 4 e) 2/8 mmc (2,8) = 2 b) 22 = 4 ( - 2)2 = 4 c) 5 + 7 x 1 = 10 3 x 3 = 9 d) mmc (3,6) = 1 4 nmero primo f) 6 2 6 2 0 g) 3 < 2 3 x 7 = 2 x 5 5 7

MATEMTICA PASSO A PASSO

3) Admitindo que p e q so verdadeiras e r falsa, determine o valor ( V ou F) de cada proposio abaixo. a) p r b) p q c) r p d) (p r) q e) p ( q r) f) p ( q r ) g) ~ p ~ q h) ~ p r

4) Sendo a proposio p ( r s) falsa e a proposio (q ~ s ) p verdadeira, classifique em verdadeira ou falsa as afirmaes p, q, r, s. Respostas: 2) a) V b) v c) V d)V e) F f) F g) V d) V e) F f) V g) V h) V 3) a) F b) V c) V

4) p (V), q (V), r ( F), s ( F). Tautologias

Seja v uma proposio formada a partir de outras (p,q,r,...) mediante o emprego de conectivos ( ou ) ou de modificador ( ~ ) ou de condicionais ( ou ). Dizemos que v uma tautologia ou proposio logicamente verdadeira quando v tem o valor lgico V (verdadeira) independentemente dos valores lgicos de p, q, etc. 225

PROF. WELLINGTON BRITO

Assim a tabela verdade de uma tautologia v apresenta s V na coluna de v. Exemplos 1) ( p ~ p ) (q p) uma tautologia, pois: p p) V V V F F F F F V V V V q

~p

p ~ p q p ( p ~ p) (q

2) ~ (p q ) (~ p ~ q) uma tautologia, pois: p q p q ~(p q) ~ p ~ q ~ p ~ q ~ (p q)(~p

~q)
V V V F F F F V

Proposies Logicamente Falsas

Seja uma proposio formada a partir de outras ( p, q, r,...) mediante o emprego de conectivos ( ou ) ou de modificador ( ~ ) ou de condicionais ( ou ). Dizemos que uma proposio logicamente verdadeira quando tem o valor lgico F (falsa) independentemente dos valores lgicos de p, q, etc. Assim a tabela verdade de uma proposio logicamente falsa apresenta s F na coluna de . Exemplos 1) p ~ p proposio logicamente falsa, pois: p V F

~p
F V

p ~ p F F

226

MATEMTICA PASSO A PASSO

2) ( p ~ q ) (~ p q ) p q ~ p ~qp ~q ~ p q V V V F F V F F V F V F V V F F F V

( p ~ q ) (~p q)
F F F

Como Negar Proposies Passo a Passo para negar

Vamos destacar os resultados obtidos proposies compostas, conectivas e condicionais. Negao de uma disjuno ( p q)

Tendo em vista que ~ (p q) ( ~ p ~ q), podemos estabelecer que a negao de p q a proposio ~ p ~ q. Exemplos 1) p: o tringulo ABC issceles. q: o tringulo ABC eqiltero. p v q: o tringulo ABC issceles ou eqiltero ~ (p q) : o tringulo ABC no issceles e no eqiltero. 2) p: a = 0 q: b = 0 (p q): a = 0 ou b = 0 ~ (p q): a 0 e b 0 Negao de uma conjuno (p q) Tendo em vista que ~ (p q) ~ p ~ q, podemos estabelecer que a negao de p q a proposio ~ p ~ q. Exemplos 1) p: a 0 q: b 0 p q: a 0 ou b 0 227

2) p: 2/4 q: 3/9 p q: 2/4 e 3/9

~ (p q): a = 0 ou b = 0

PROF. WELLINGTON BRITO

~ (p q): 2 4 ou 3 9

Negao de um condicional

J que ~ (p q) p ~ q, podemos estabelecer que a negao de p q a proposio p ~ q. Exemplos: 1) p: 2

q: 2 p q: 2 2

~ (p q): 2 e 2 2) p: 52 = ( 5) 2 q: 5 = 5 p q: 52 = ( 5)2 5 = 5 ~ (p q): 52 = ( 5)2 e 5 - 5 Negao de proposies quantificadas a)Uma sentena quantificada com o quantificador universal do tipo (x ) (p (x)), negada assim: substitui-se o quantificador pelo existencial e nega-se p(x), obtendo: (x) (~ p(x)). a) (x ) ( x + 3) = 5 ( ~ ) (x) ( x + 3 5) b) Uma sentena quantificada com o quantificador existencial, do tipo (x)( p(x)), negada assim: substitui-se o quantificador pelo universal e nega-se p(x), obtendo:(x)( ~p (x)). Exemplos: 1 1 1 1)sentena:( a) 2)sentena:( a) a + R a 2 3 negao ( a)

( (

1 a

) )

negao:( a) a +

( (

1 2

<

) )

) )

Questes Comentadas

1) A negao de x A B x A v x B. Assim, a negao da sentena Chico cantor e compositor a sentena Chico no cantor ou no compositor. 228

2) A negao de x A B x A x B. Assim, a negao da sentena Regina gosta de cinema ou gosta de teatro a sentena Regina no gosta de cinema e no gosta de teatro. 3) Verifique, por meio da tabela-verdade, a equivalncia da proposio: ( p q ) ( ~ q ~ p), Resoluo: ( p q ) ( ~ q ~ p) p V V F q V F V p q V F V

MATEMTICA PASSO A PASSO

q F V F

~
F F V

q V F V

Resposta: A coluna ( p q ) apresenta o mesmo resultado da coluna ( ~ q ~ p),o que significa dizer que verdadeiro o bi condicional ( p q ) ( ~ q ~ p). 4) Se Beto briga com Glria, ento Glria vai ao cinema, se Glria vai ao cinema, ento Carla fica em casa. Se Carla fica em casa, ento Raul briga com Carla. Ora, Raul no briga com Carla. Logo: a) Carla no fica em casa e Beto no briga com Glria. b) Carla fica em casa e Glria vai ao cinema. c) Carla no fica em casa e Glria vai ao cinema. d) Glria no vai ao cinema e Beto briga com Glria e) Glria no vai ao cinema e Beto briga com Glria. Resoluo: Se Raul no briga com Carla ento: Carla no fica em casa Se Carla no fica em casa, ento Glria no vai ao cinema Se Glria no vai ao cinema, ento Beto no briga com Glria 229

PROF. WELLINGTON BRITO

Logo a nica opo correta a . Carla no fica em casa e Beto no briga com Glria. 5) Se Carlos mais velho do que Pedro, ento Maria e Julia tem a mesma idade. Se Maria e Julia tem a mesma idade, ento Joo mais moo do que Pedro. Se Joo mais moo do que Pedro, ento Carlos mais velho do que Maria. Ora, Carlos no mais velho do que Maria. Ento: a) Carlos no mais velho do que Leila, e Joo mais moo do que Pedro. b) Carlos mais velho que Pedro, e Maria e Julia tem a mesma idade. c) Carlos e Joo so mais moos do que Pedro. d) Carlos mais velho do que Pedro, e Joo mais moo do que Pedro. e) Carlos no mais velho do que Pedro, e Maria e Julia no tem a mesma idade. Resoluo: Se Carlos no mais velho do que Maria, ento Joo no mais moo que Pedro Se Joo no mais moo que Pedro, ento: Maria e Julia no tem a mesma idade Se Maria e Julia no tem a mesma idade, ento Carlos no mais velho que Pedro Logo,a nica opo correta : Carlos no mais velho do que Pedro, e Maria e Julia no tem a mesma idade. EXERCCIO RACICNIO LGICO ( V )
i) Todo nmero inteiro primo impar. j) Todo tringulo issceles eqiltero k) Existe um losango que no quadrado l) Existe um nmero cuja raiz quadrada zero m) Todo tringulo que tem trs ngulos

1) Diga qual a negao de cada proposio abaixo . a) mdc (2,3) = 1 ou mmc (2,3) 6 b) 3 = 6 ou 3 x 10 6 x 5 5 10 c) 3 1 e 3 7 7 d) 22 = 4 4 = 2 e) ( 3 )2 = 9 9 3

230

f) 2 5 32 52
x

MATEMTICA PASSO A PASSO

congruentes tem trs lados congruentes.


2

g) (x ) ( x > 2 3 > 3 ) h) ( x ) ( x < 0 )

2) Classifique em V ou F as negaes construdas no exerccio anterior. 3) Verifique, por meio das tabelas-verdades, a validade das equivalncias abaixo. a) da conjuno c) da conjuno relativamente disjuno p qqp p ( q r ) ( p q) (p r ) ( p q ) r p ( q r) p ( q r ) (p q) (p r ) ppp p(pq)p pvp p(pq)p p ff b) da disjuno d) da negao pqqp (p)p (p q ) r p (q r) ( p q) p q p pp ( p q) p q p vv pfp em que p, q, r so proposies quaisquer, uma tautologia e uma proposio logicamente falsa. 4) A negao da frase Todas as mulheres so honestas . a) nenhuma mulher desonesta d) Nenhuma mulher honesta b) Todas as mulheres so honestas e) Algumas mulheres so desonestas. c) Algumas mulheres so honestas 5) Numa gaveta h 20 meias pretas e 20 marrons, qual o nmero mnimo de meias que uma pessoa deve retirar, no escuro, para ter a certeza de formar um par da mesma cor? a) 2 b) 20 c) 3 d) 4 e) 40 6) Timteo tem na sua cmoda, 18 meias azuis, 12 amarelas, 8 cor de laranja,30 verdes e 2 roxas.As meias esto todas misturadas Timteo, pega em algumas, s escuras, se lhes ver a cor. Em quantas meias deve pegar para ter a certeza de conseguir, pelo menos, um par da mesma cor? a) 6 b) 5 c) 4 d)3 e)2 7) Trens, malas, maior. Estas palavras seguem uma Regra Lgica. Das palavras seguintes, qual poder continuar a srie? 231

PROF. WELLINGTON BRITO

a) Parti b) aulas

c) calma b) boio

e)menor

8)Esta srie de palavras segue uma Regra Lgica: gua, aor, corpo, pranto, cristal, fantstico. Das palavras abaixo, qual poder continuar a srie: a) Honrado c) Constituinte e) Profcuo b) Abstmio d) Equivalente 9) Um caramujo resolve subir um muro de 12 metros de altura da seguinte maneira: durante o diaele sobe 3 metros e durante a noite, ao dormir, desce 2 metros. Sabendo-se que iniciou a subida da base, ao amanhecer do 1 dia, quantos dias gastar o caramujo para chegar ao topo? a) 9 dias e meio b) 10 dias c) 10 dias e meio d) 11 dias e)12 dias

10) Assinale a opo que contm a seqncia correta das quatro bolas, de acordo com as afirmativas abaixo. I A bola amarela est depois da branca II A bola azul est antes da verde III - A bola que est imediantamente aps a azul maior do que a que est antes desta. IV - A bola verde a menor de todas. a)branca,amarela,azul e verde b)branca,azul,amarela e verde c) branca, azul, verde e amarela. d)azul,branca,amarela e verde e)azul,branca,verde e amarela.

* Nos exerccios 11 a 13, assinale a opo que contm o numeral correto, sabendo que as seqncias seguem uma ordem lgica. 11) 4, 11, 17, 22, 26, 29, 31, _____ a) 31 a) 18 a) 71 b) 30 b) 8 b) 132 c) 32 c) 17 c) 72 d) 29 d) 7 d) 144 e) 33 e) 4 e) 73 12) 67, 64, 59, 52, 43, 32, 19,_____ 13) 2, 5, 10, 14, 28, 33, 66, _______ 14) Anteontem Maria tinha 17 anos. No ano que vem, ela vai fazer 20 anos. Que dia hoje? a)1 de Abril b) 31 de dezembro 232 c) 1 de Janeiro

MATEMTICA PASSO A PASSO

d) dia do seu aniversrio e) um dia antes do seu aniversrio 15) Se a praia no est movimentada, ento os pssaros voam. Se a praia est movimentada, ento o pssaro no canta. Ora, o pssaro canta, logo: a) A praia est movimentada e o pssaro voa. b) A praia est movimentada e o pssaro no voa. c) A praia no est movimentada e o pssaro voa. d) A praia no est movimentada e o pssaro no voa. e) Se o pssaro canta, ento eles no voam. 16) Um crime cometido por uma pessoa e h quatro suspeitos: Ari, Belo, Caio e Denis. Interrogados, fazem as seguintes declaraes: Ari: Belo o culpado. Belo: Denis o culpado. Caio: Eu no sou culpado. Denis: Belo mente quando diz que sou culpado. Sabendo-se que apenas um dos quatro no falou a verdade, quem o culpado do crime cometido? a) Ari b) Belo c) Caio d) Denis 17) Trs meninos, cujos nomes so Andr, Beto e Carlos, tem as seguintes caractersticas: um dos trs louro, outro moreno e o outro ruivo. Andr mente sempre que Beto diz a verdade. Carlos mente quando Beto mente. Cada um dos meninos faz uma afirmao: Andr afirma: Eu sou brasileiro ou no sou brasileiro. Carlos afirma: Beto ruivo. Beto afirma: Eu sou loiro ou Carlos ruivo.

Considerando as caractersticas e as afirmaes citadas, correto concluir que Andr, Beto e Carlos so, respectivamente caracterizada como: a) Louro, ruivo, moreno b) Ruivo, louro, moreno c) Louro, moreno, ruivo. d) Ruivo, moreno, louro e) Moreno, louro, ruivo.

233

PROF. WELLINGTON BRITO

18) Considere que, em um pequeno grupo de pessoas G envolvidas em um acidente, haja apenas dois tipos de indivduos: aqueles que sempre falam a verdade e os que sempre mentem. Se no conjunto G, o individuo P afirmar que o individuo Q fala a verdade, e Q afirmar que P e eles so tipos opostos de indivduos, ento, nesse caso, correto concluir que: a) apenas P fala a verdade. b) apenas Q fala a verdade. c) P e Q falam verdade d) P e Q mentem. e) As afirmaes so inconsistentes. 19) H trs suspeitos de um crime: A governanta o cozinheiro e o mordomo. Sabe-se que o crime foi efetivamente por um ou por mais de um deles, j que podem ter agido individualmente ou no. Sabe-se ainda que: Logo: a) b) c) d) e) Se o cozinheiro inocente, ento a governanta culpada; Ou o mordomo culpado ou a governanta culpada, mas no os dois; O mordomo no inocente A governanta e o cozinheiro so culpados Somente o cozinheiro inocente Somente a governanta culpada O cozinheiro e o mordomo so os culpados Somente o mordomo culpado.

20) Marcos e Paulo pertencem a um grupo de mentirosos programados. Marcos mente sempre na tera, quarta e quinta, dizendo a verdade nos outros dias da semana. Paulo mente sempre na sexta, sbado e domingo, dizendo a verdade nos outros dias. Certo dia, dialogando entre eles, afirmam. Marcos: Eu mentirei amanh assim como ontem. Paulo: Hoje tera-feira Em que dia da semana ocorreu esse dilogo? a) segunda-feira c) quarta-feira e) domingo b) tera-feira d) sbado

234

MATEMTICA PASSO A PASSO

21) Se Fred fala francs, ento Albert no alemo. Ou Albert a alemo, ou den espanhol. Se Pedro no portugus, ento Fred francs. Ora, nem den espanhol nem Isa Italiana. Assim: a) Pedro portugus e Fred francs. b) Pedro portugus e Albert Alemo c) Pedro no portugus e Albert alemo d) Eden espanhol ou Fred francs. e) den espanhol ou Albert no alemo. 22) Se W = 2a + 3b, ento W = 4p + 3r . Se W = 4p + 3r, ento W = 2s 3r. Por outro lado, W = 2a + 3b, ou W = 0. Se W = 0, ento W + S = 5. Ora, W + S 5. Ento a) 2s 3r = 0 d) 2a + 3b 2s 3r b) 4p + 3r 2s 3r e) W = 2s 3r c) W 2a + 3b 23) Paulo guarda suas gravatas em uma nica gaveta em seu quarto. Nela encontra-se sete gravatas azuis, nove amarelas, uma preta, trs verdes e trs vermelhas. Uma noite, no escuro, Paulo abre a gaveta e pega algumas gravatas. O nmero mnimo de gravatas que Paulo deve pegar para ter certeza de ter pegado ao menos duas gravatas da mesma cor : a) 6 b) 8 c) 18 d) 23 e)22 Respostas 1) a) mdc(2,3) 1e mmc(2,3) = 6 j) Existe um triangulo issceles e no eqiltero. b) 3 6 5 10 ou 3 x 10 = 6 x 5 k) Todo losango quadrado. l)Todo nmero tem raiz quadrada diferente de zero. m)Existe um tringulo eqingulo e no eqiltero 2)a)F b)F c)V d)F e)F f)F g)F h)V i)V j)V k)F L)F m)F 4)C 5) C 6) A 7) D 8) C 9)A 10)B 11)C 12)E 13) C 14)C 15)C 16)B 17)D 18)D 19)D 20)B 21)B 22)E 23) A

c) 3 < 1 ou 3 < 7 7 d) 22 = 4 e 4 2 e) ( 3 )2 = 9 e 9 = 3 f) 2 > 5 e 32 > 52 g) ( x ) ( x > 2 e 3x 32 h) ( x ) ( x 0 ) i) Existe um nmero inteiro primo e par

235

PROF. WELLINGTON BRITO

QUESTES DE CONCURSOS E VESTIBULARES

1) (PUC-SP) A negao da proposio x ( A B) : a) x ( A B ) b) x A ou x B c) x A e x B d) x A ou x B e) x A e x B 2) (UF-BA) A negao de Hoje segunda-feira e amanh no chover : a) Hoje no segunda-feira e amanh chover b) Hoje no segunda-feira ou amanh chover c) Hoje no segunda-feira, ento amanh chover d) Hoje no segunda-feira nem amanh chover e) Hoje segunda-feira ou amanh no chover 3) (FEI-SP) Dadas as proposies: (1) Toda mulher boa motorista. (2) Nenhum homem bom motorista. (3) Todos os homens so maus motoristas. (4) Pelo menos um homem mau motorista. (5) Todos os homens so bons motoristas. a negao de (5) : a) (1) b) (2) c) (3) d) (4) e) n.d.a

4) (PUC-RS) A sentena (x x a = b) a negao de: a) x x a b b) x x a > b c) x x a < b d) x, x a = b e) x, x a b 5) (UNESP) Uma pessoa que gosta de todas e apenas das pessoas que no gostam de si mesmas: a) gosta de si mesma. b) no gosta de si mesma. c) no existe. d) gosta de algum. e) no gosta de ningum. 236

MATEMTICA PASSO A PASSO

6) (FATEC-SP) Considere verdadeiras as trs seguintes afirmaes: I - Todos os amigos de Joo so amigos de Mrio II- Mrio no amigo de qualquer amigo de paulo III - Antonio s amigo de todos os amigos de Roberto. Se Roberto amigo de Paulo, ento a) Antonio amigo de Mrio b) Joo amigo de Roberto c) Mrio amigo de Roberto d) Antonio no amigo de Joo. e) n.d.a 7) (FEI-SP) Dadas as premissas: Todos os corintianos so fanticos Existem fanticos inteligentes, pode-se tirar a concluso seguinte: a) Existem corintianos inteleigentes. b) Todo corintiano inteligente. c) Nenhum corintiano inteligente. d) Todo inteligente corintiano. e) No se pode tirar concluso. 8) (MACK-SP) Duas grandezas x e y so tais que: se x = 3, ento y = 7. Pode-se concluir que: a) se x 3, ento y 7. b) se y = 7, ento x = 3. c) se y 7, ento x 3. d) se x = 5, ento y = 5. e) nenhuma das concluses anteriores valida 9) (U.F.-GO) A negao de x 2 : a) x 2 b) x 2 c) x < 2 d) x < 2 e) x 2 10) (FUVEST) Cada um dos cartes abaixo tem de um lado um nmero e do outro lado uma letra. A B C 2 3

Algum afirmou que todos os cartes que tm uma vogal numa face tm um nmero par na outra. Para verificar se tal afirmao verdadeira: a) necessrio virar todos os cartes b) suficiente virar os dois primeiros cartes c) suficiente virar os dois ltimos cartes. d) suficiente virar os dois cartes do meio. e) suficiente virar o primeiro e o ultimo carto. 237

11) (PUC-RS) Sejam p e q duas proposies. A negao de p q equivale a: a) p q b) p q c) p q d) p q e) p q 12)(VUNESP)A negao de para todo real x existe um real y tal que y < x equivalente a: a) existe um real x tal que x y para todo real y. b) no existe um real x tal que x y para todo real y. c) existe um real x tal que y x para todo real y. d) no existe um real x tal que y x para todo real y e) para todos reais x, y, com x < y, existe um real z com x < z < y. 13) (U.F.BA) A proposio p q q r verdadeira, se: a) p e q so verdadeiras e r, falsa. d) p, q e r so verdadeiras. b) p e q so falsas e r, verdadeira. e) p, q e r so falsas. c) p e r so falsas e q, verdadeira. 14) (U.F.RS) A negao da proposio para todo y, existe um x tal que y = sen(x) : a) Para todo y, existe um x tal que y = sen(x). b) Para todo y e para todo x, y = sen(x). c) Existe um y e existe um x tal que y = sen(x). d) Existe um y tal que, para todo x, y = sen(x). e) Existe um y tal que, para todo x, y sen(x). 15)(U.F.RS)A negao da proposio( x R) ( y R) [xy = 1] : a) ( x R ) ( y R ) [xy = 1] b) ( x R) ( y R ) [ xy 1] c) ( x R ) ( y R) [ xy 1] d) ( x R) ( y R) [xy 1] e) ( x R ) ( y R ) [xy 1]

PROF. WELLINGTON BRITO

16)(UFC) Trs bolas A, B, C, foram pintadas: uma verde, uma de amarelo e uma de azul, no necessariamente nesta ordem. Leia atentamente as declaraes abaixo: I) B no azul II) A azul III) C no amarela Sabendo-se que apenas uma das declaraes acima verdadeira, podemos afirmar corretamente que: a) A bola A verde, a bola B amarela e a bola C azul. b) A bola A verde, a bola B azul e a bola C amarela. c) A bola A amarela, a bola B azul e a bola C verde d) A bola A amarela, a bola B verde e a bola C azul e) A bola A azul, a bola B verde e a bola C amarela. 238

MATEMTICA PASSO A PASSO

17) (UECE) Em cada crculo, os nmeros esto colocados de acordo com um raciocnio lgico matemtico. 6 12 23 44 7 14 26 48 5 10 20 40 Complete o ltimo crculo e encontre a soma dos seus trs nmeros. a) 250 b) 255 c) 260 d) 265 e)270 18)(UECE) Os nmeros colocados nos quadrados seguem uma organizao lgica. 38 20 N X 3 a) 10 b) 11 c)12 4 X 1 d) 13 e)14 18

Observando os nmeros, atentamente, determine o nmero N. 19)(MPU) Ana guarda suas blusas em uma nica gaveta em seu quarto. Nela encontram-se sete blusas azuis, nove amarelas, uma preta, trs verdes e trs vermelhas. Uma noite, no escuro, Ana abre a gaveta e pega algumas blusas. O nmero mnimo de blusas que Ana deve pegar para ter certeza de ter pegado ao menos duas blusas da mesma cor : a) 6 b)4 c) 2 d) 8 e)10 20) (MPU) Sabe-se que Joo estar feliz a condio necessria para Maria sorrir e condio suficiente para Daniela abraar Paulo. Sabe-se, tambm, que Daniele abraar Paulo condio necessria e suficiente para a Sandra abraar Srgio. Assim, quando Sandra no abraa Srgio. a)Joo est feliz, e Maria no sorri, e Daniela abraa Paulo. b)Joo no est feliz, e Maria sorri, e Daniela no abraa Paulo. c)Joo est feliz, e Maria sorri, e Daniela no abraa Paulo. d)Joo no est feliz,e Maria no sorri,e Daniela no abraa Paulo. e) Joo no est feliz, e Maria sorri, e Daniela abraa Paulo.

239

PROF. WELLINGTON BRITO

21) (MPU) Fernanda atrasou-se e chega ao estdio da Ulbra quando o jogo de vlei j est em andamento. Ela pergunta s suas amigas, que esto assistindo partida, desde o inicio, qual o resultado at o momento. Suas amigas dizem-lhe: Amanda: Neste set, o escore est 13 a 12 Berenice: O escore no est 13 a 12, e a Ulbra j ganhou o primeiro set. Camila: Este set est 13 a 12, a favor da Ulbra. Denise: O escore no est 13 a 12, a Ulbra est perdendo este set, e quem vai sacar a equipe visitante Eunice: Quem vai sacar a equipe visitante, e a Ulbra est ganhando este set. Conhecendo suas amigas, Fernanda sabe que duas delas esto mentindo e que as demais esto dizendo a verdade. Conclui, ento, corretamente que: a) o escore est 13 a 12, e a Ulbra est perdendo este set, e quem vai sacar a equipe visitante. b) o escore est 13 a 12 e a Ulbra vai sacar, e a Ulbra venceu o primeiro set. c) o escore no est 13 a 12, e a Ulbra est vencendo este set, e quem vai sacar a equipe visitante, d) o escore est 13 a 12, e a Ulbra est vencendo este set, e quem vai sacar a equipe visitante. e) o escore no est 13 a 12, e a Ulbra no est vencendo este set, e a Ulbra venceu o primeiro set. 22) (MPU) Scrates encontra-se em viagem por um distante e estranho pas, formado por apenas duas aldeias, uma grande e outra pequena. Os habitantes entendem perfeitamente o portugus, mas falam apenas no idioma local, desconhecido por Scrates. Ele sabe, contudo, que os habitantes da aldeia menor sempre dizem a verdade, e os da aldeia maior sempre mentem. Sabe, tambm, que Milango e Nabungo so palavras no idioma local que significam sim e no, mas no sabe qual delas significa sim e nem, conseqentemente, qual significa no. Um dia, Scrates encontra um casal acompanhado de um jovem. Dirigindo-se a ele, e apontando para o casal, Scrates pergunta: Meu bom jovem, a aldeia desse homem maior do que a dessa mulher? Milango reponde o jovem. E a tua aldeia maior do que a desse homem? voltou Scrates a perguntar. 240

Milango tornou o jovem a responder E, diz-me ainda, s tu da aldeia maior? perguntou Scrates Nabungo disse o jovem. Scrates, sorrindo, conclui corretamente que: a) o jovem diz a verdade, e o homem da aldeia grande e a mulher da grande. b) O Jovem mente, e o homem da aldeia grande e a mulher da grande. c) O jovem mente, e o homem da aldeia grande e a mulher da pequena d) o jovem diz a verdade, e o homem da aldeia pequena e a mulher da pequena. e) O jovem mente, e o homem da aldeia pequena e a mulher da pequena 23)(MPU) Cinco irmo exercem, cada um, uma profisso diferente. Luis paulista, como o agrnomo e mais moo do que o engenheiro e mais velho do que o Oscar. O agrnomo, o economista e Mrio residem no mesmo bairro. O economista, o matemtico e Luis so, todos, torcedores do Flamengo. O matemtico costuma ir ao cinema com Mrio e Ndio. O economista mais velho do que Ndio e mais moo do que Pedro; este, por sua vez mais moo do que o arquteto. Logo. a) Luis arquiteto e o engenheiro mais velho do que o agrnomo e Pedro mais velho do que o matemtico. b) Oscar engenheiro, e o matemtico mais velho do que o agrnomo, e Luis mais velho do que o matemtico. c) Mrio engenheiro, e o matemtico mais velho do que o agrnomo, e o economista mais novo do que Luis. d) Pedro matemtico, e o arquiteto mais velho do que o engenheiro, e Oscar mais velho do que o agrnomo. e) Ndio engenheiro, e o arquiteto mais velho do que o matemtico, e Mrio mais velho do que o economista. 24) (MPU) Caio, Dcio, der, Felipe e Gil compraram, cada um, um barco. Combinaram, ento dar aos barcos os nomes de suas filhas. Cada um tem uma nica filha, e todas tem nomes diferentes. Ficou acertado que nenhum deles poderia dar a seu barco o nome da prpria filha e que a cada nome das filhas corresponderia um e apenas um barco. Dcio e der desejavam, ambos, dar seus barcos o nome de Las, mas acabaram entrando em um acordo: o nome de Las ficou para o 241

MATEMTICA PASSO A PASSO

PROF. WELLINGTON BRITO

barco de Dcio e der deu a seu barco o nome de Mara. Gil convenceu o pai de Olga a pr o nome de Paula em seu barco ( isto , no barco dele, pai de Olga). Ao barco de Caio, coube o nome de Nair e ao barco do pai de Nair, coube o nome de Olga. As filhas de Caio, Dcio, der, Felipe e Gil so, respectivamente. a)Mara,Nair,Paula, Olga, Lais. b)Lais,Mara,Paula, Olga, Nair. c)Lais,Mara,Olga, Nair, Paula. d) Paula, Olga, Las, Nair, Mara e) Nair, Las, Mara, Paula, Olga. 25) (MPU) Ana, Bia, Cl, Da e Ema esto sentadas, nessa ordem e em sentido horrio, em torno de uma mesa redonda. Elas esto reunidas para eleger aquela que, entre elas, passar a ser a representante do grupo. Feita a votao, verificou-se que nenhuma fra eleita, pois cada uma delas havia recebido exatamente um voto. Aps conversarem sobre to inusitado resultado, concluram que cada um havia votado naquela que votou na sua vizinha da esquerda (isto , Ana votou naquela que votou na vizinha da esquerda de Ana, Bia votou naquela que votou na vizinha da esquerda e Bia, e assim por diante). Os votos de Ana, Bia, Cl, Da e Ema fora, respectivamente, para: a) Emma, Ana, Bia, Cl, Da. d) Da, Ema, Ana, Bia, Cl b) Cl, Da, Ema, Ana, Bia. c) Cl, Bia, Ana, Ema, Da. 26)(MPU) Em torno de uma mesa quadrada, encontram-se sentados quatro sindicalista. Oliveira, o mais antigo entre eles, mineiro. H tambm um paulista. Um carioca e um baiano. Paulo est sentado direita de Oliveira. Norton; direita do paulista. Por sua vez, Vasconcelos, que no carioca, encontra-se frente de Paulo. Assim, a) Paulo paulista e Vasconcelos baiano. b) Paulo carioca e Vasconcelos baiano. c) Norton baiano e Vasconcelos paulista d) Norton carioca e Vasconcelos paulista. e) Paulo baiano e Vasconcelos paulista. 242 e) Da, Ana, Bia, Ema, Cl.

MATEMTICA PASSO A PASSO

27) (MPU) Quando no vejo Carlos, no passeio ou fico deprimida. Quando chove, no passeio e fico deprimida. Quando no faz calor e passeio, no vejo Carlos. Quando no chove e estou deprimida,no passeio. Hoje, passeio. Portanto hoje. a) vejo Carlos, e no estou deprimida, e chove, e faz calor. b) no vejo Carlos, e estou deprimida, e chove, e faz calor. c) vejo Carlos, e no estou deprimida, e no chove, e faz calor. d) no vejo Carlos,e estou deprimida,e no chove,e no faz calor. e) vejo Carlos, e estou deprimida, e no chove, e faz calor. 28) (MPU) Se fulano culpado, ento Beltrano culpado. Se Fulano inocente, ento ou Beltrano culpado ou Sicrano culpado, ou ambos, Beltrano e Sicrano, so culpados. Se Sicrano inocente, ento Beltrano inocente. Se Sicrano culpado ento Fulano culpado. Logo, a) Fulano inocente, e Beltrano inocente, e Sicrano inocente. b) Fulano culpado, e Beltrano culpado, e Sicrano inocente. c) Fulano culpado e Beltrano inocente, e Sicrano inocente. d) Fulano inocente, e Beltrano culpado, e Sicrano culpado. e) Fulano culpado, e Beltrano culpado, e Sicrano culpado. 29)(MPU) Uma curiosa mquina tem duas teclas, A e B, e um visor no qual aparece um nmero inteiro x.Quando se aperta a tecla A, o nmero do visor substitudo por 2x + 1. Quando se aperta a tecla B, o nmero do visor substitudo por 3x 1. Se no visor est o nmero 5, o maior nmero de dois algarismos que se pode obter, apertando-se qualquer seqncia das teclas A e B, . a) 87 b) 95 c)92 d)85 e)96 30)(MPU) A operao x definida como o triplo do cubo de x, e a operao x definida como o inverso de x. Assim, o valor da operao. 3 (2 ) 21 igual a a) 15 b) 20 c) 25 d) 45 e) 30

243

PROF. WELLINGTON BRITO

31)(MPU) Um colgio oferece a seus alunos a pratica de um ou mais dos seguintes esportes: futebol, basquete e vlei. Sabe-se que, no atual semestre, 20 alunos praticam vlei e basquete 60 alunos praticam futebol e 65 praticam basquete; 21 alunos no praticam nem futebol nem vlei O nmero de alunos que praticam s futebol idntico ao nmero dos alunos que praticam s vlei; 17 alunos praticam futebol e vlei; 45 alunos praticam futebol e basquete; 30, entre os 45, no praticam vlei O nmero total de alunos do colgio, no atual semestre, igual a: a) 99 b)93 c)103 d)110 e)114 32) (MPU) Voc est frente de duas portas. Uma das conduz a um tesouro; a outra, a uma sala vazia. Cosme guarda uma das portas, enquanto Damio guarda a outra. Cada um dos guardas sempre diz a verdade ou sempre mente, ou seja, ambos os guardas podem sempre mentir, ambos podem sempre dizer a verdade ou um sempre dizer a verdade e 5 outro sempre mentir. Voc no sabe s ambos so mentirosos, seambos so verazes, ou se um veraz e o outro mentiroso. Mas, para descobrir qualdas portas conduz ao tesouro, voc pode fazer trs ( e apenas trs) perguntas aos guardas, escolhendo-as da seguinte relao: P1: O outro guarda da mesma natureza que voc ( isto , se voc mentiroso ele tambm o ,e se voc veraz tambm o )? P2: Voc o guarda da porta que leva ao tesouro? P3: O outro guarda mentiroso? P4: voc veraz? Ento, uma possvel seqncia de trs perguntas que logicamente suficiente para assegurar, seja qual for a natureza dos guardas, que voc identifique corretamente a porta que leva ao tesouro, . a) P2 a Cosme, P2 a Damio, P3 a Damio b) P3 a Damio, P2 a Cosme, P3 a Cosme c) P3 a Cosme, P2 a Damio, P4 a Cosme d) P1 a Cosme, P1 a Damio, P2 a Cosme e) P4 a Cosme, P1 a Cosme, P2 a Damio 244

MATEMTICA PASSO A PASSO

33) (AFTN) Trs amigas, Tnia, Janete, e Anglica, esto sentada lado a lado em um teatro. Tnia sempre fala a verdade; Janete s vezes fala a verdade ; e Anglica nunca fala a verdade. A que est sentada esquerda diz: Tnia quem est sentada no meio. A que est sentada no meio diz. Eu sou Janete. Finalmente, a que est sentada direita diz: Anglica quem est sentada no meio. A que est sentada esquerda, a que est sentada no meio e a que est sentada direita so respectivamente: a) Janete, Tnia e Angelica. b) Janete, Anglica e Tnia. c) Anglica, Janete e Tnia d) Anglica, Tnia e Janete e) Tnia, Anglica e Janete. 34) (AFTN)Jos que ir ao cinema e assistir ao filem Fogo contra fogo, mas no tem certeza se o mesmo est sendo exibido. Seus amigos, Maria, Luis e Julio, tm opinies discordantes sobre o filme est ou no em cartaz. Se Maria estiver certa, ento Jlio est enganado. Se Jlio estiver enganado, ento Luis est enganado, ento o filme no est sendo exibido ; Ora, ou o filme Fogo contra fofo est sendo exibido, ou Jos no ir ao cinema. Verificou-se que Maria est certa. Logo a) o filme Fogo contra fogo est sendo exibido; b) Luis e Jlio no esto enganados; c) Jlio est enganado, mas no Luis d) Luis est enganado, mas no Jlio; e) Jos no ir ao cinema Respostas: 01) E 02) B 03) D 04) E 05) C 06) D 07) E 08) A 09) C 10)E 11)A 12)A 13)D 14)E 15)C 16)C 17)B 18)B 19) A 20) D 21) D 22) B 23) C 24) B 25) B 26) A 27) C 245 28) E 29) B 30) C 31) A 32) D 33) B 34) E

PROF. WELLINGTON BRITO

FUNO POLINOMIAL

Par Ordenado

A noo de par ordenado ser aqui adotada como conceito primitivo. Podemos formar idia de par ordenado, imaginando-o como um conjunto de dois elementos considerados numa dada ordem. Para lembrar que a ordem est sendo considerada, na representao do par ordenado utilizamos parnteses e no chaves como nos conjuntos em geral. Assim, ( X,Y ) par ordenado de 1 termo X e 2 termo Y. Impomos que dois pares ordenados so iguais somente se tiverem os primeiros termos iguais entre si e tambm os segundos termos iguais entre si: (a, b) = (c, d) ( a = c e b = d)

Exemplos: 1) (a , b) = (5,1) ( a = 5 e b = 1) 2) (a , b) = (3,3) ( a = 3 e b = 3) Nota: Observe que num par ordenado podemos ter os dois termos iguais. Produto Cartesiano Dados os conjuntos A = { 1,2, 3} e B = { 1, 2, 3, 4} vamos formar pares ordenados que tm o 1 termo em A e o 2 termo em B. Observe no esquema que cada flecha determina um par. 1 termo 2 termo Par Ordenado A 1 2 3 B 1 2 2 3 4 3 1 2 3 4 ----------------------------------------------------( 3, 1 ) ( 3, 2 ) ( 3, 3 ) ( 3, 4 ) 1 1-------------- ( 1, 1 ) 2-------------- ( 1, 2 ) 3 -------------- ( 1, 3 ) 4 -------------- ( 1, 4 ) 1-------------2 -------------3 -------------4 --------------( 2, 1 ) ( 2, 2 ) ( 2, 3 ) ( 2, 4 )

246

MATEMTICA PASSO A PASSO

O Conjunto formado pelos pares ordenados obtidos denominado produto cartesiano de A por B e o indicamos por A x B (leia: A cartesiano B). Temos, ento, A x B = {(1,1),(1,2 ), (1,3 ), (1,4 ), (2,1 ), (2,2 ), (2,3 ), (2,4 ), (3,1 ), (3,2 ), (3,3 ), (3, 4 )} De maneira geral, o produto cartesiano de dois conjuntos A e B o conjunto A x B formado pelos pares ordenados que tm 1 termo em A e 2 termo em B. AxB={(x,y)xAeyB} Nota: Observamos que os elementos dos conjuntos A x B so pares ordenados. No exemplo anterior A tem 3 elementos, B tem 4 elementos e AxB, tem 3 x 4 = 12 elementos. Podemos constatar que quando A tem m elementos e B tem n elementos, o conjunto A x B formado por m . n pares ordenados. Exemplos 1) Dados A = { a, e, i } e B = { m, n } determinar os conjuntos A x B, B x A, A2 = A x A e B2 = B x B. Temos: A x B = { (a,m), (a,n), (e,m), (e,n), (i,m), (i,n) B x A = { (m,a), (m,e), (m,i), (n,a), (n,e), (n,i)} Note que: A x B BxA (por exemplo,o par (a, m) A x B mas (a,m) B x A) A2 = A x A = {(a,a), (a,e), (a,i), (e,a), (e,e), (e,i), (i,a), (i,e), (i,i) } B2 = B x B = {(m.m), (m,n), (n,m), (n,n)} 2) Se A tem 5 elementos e B tem 7 elementos, ento A x B tem 5 x 7 = 35 elementos. Relao Retomando os conjuntos A = { 1,2,3) e B = { 1,2,3,4} vamos formar agora o conjunto R dos pares ordenados que tm 1 termo em A e 2 termo em B, tais que o 1 termo menor que o 2. A 1 B 1 2 2 3 3 4 R = { ( 1,2), (1,3), (1,4), (2,3), (2,4), (3,4) } 247

PROF. WELLINGTON BRITO

Este conjunto R, que subconjunto de A x B, exemplo de uma relao de A em B. De modo geral, denominamos relao de A em B a todo subconjunto de A x B. R uma relao de A em B R A x B Exemplo Dado A = {1,2,3,4,5} e B = {1,3,5,7,9} determinar as relaes de A em B: a) R = { (x, y) A x B x + y = 6 } A 1 2 3 1 B 3 5 b) S = { (x, y ) A x B xy 6 } A relao R formada pelos pares (x, y),

x A e y B, com a soma dos termos . 4 7 x + y = 6 Estes pares so (1,5), (3,3) e (5,1). 5 9 Logo, R = {(1,5),(3,3),(5,1)} pares com soma = 6 EXERCCIO FUNO (I) 1) Dados A = {1,2,3} e B = { 4,5}, forme todos os pares ordenados que tm o primeiro termo em A e o segundo em B. 2) Sendo A = {a,b} e B = { x, y, z}, determine os conjuntos: A x B e B x A. 3) Determine A x B e B x A nos casos: a) A = {1,2,3,4,5 } e B = {9} c) A = { 7 } e B = {5} b) A = { -1 , 1 } e B = { -1, 0, 1} d) A = { 3, 6, 9 } e B = 4)Se um conjunto A tem 5 elementos e B tem 10 elementos, quantos elementos tem A x B? e B x A? Os conjuntos A x B e B x A so iguais? 5)Dados A= {1,2,3,4,5,6,7,8} e B = {2,4,6,8,10}, forme as seguintes relaes de A em B: a) R = {(x, y) A x B x + y = 12} b) S = {(x, y) A x B x + y 15} 6)Dados A = {3,6,9,12} e B = {1,3,5,7,9}, determine: a)R = {(x, y) A x B x < y } c)T= {(x, y) A x Bx2 + y2 < 50} b) S = {(x, y) A x B 2xy < 25} 7)Determine as seguintes relaes no conjunto dos nmeros naturais: a) R = {(x, y) N x N 2x + y = 10 } b) S = {(x, y) N x N x2 + y2 = 25} 248 c) T = {(x,y) A x B x + y < 8}

8) Quantos pares pertencem relao R= {(x, y) 2x2 + y2 = 25} ? 9) Se R={ ( x, y ) N2 x + y = 10} e S = { ( x, y ) N2 x y = 2 }, determine R S. 10) Calcule a e b de modo que se verifique a igualdade dos pares ordenados em cada caso: a) ( a, 2b) = (3, 4) d) (a + 2b, 2a + 3) = (6,5) b) ( a + 1, 2b 1 ) = ( -1 ,0 ) e) (2a + b, a b) = (12, 5) c) ( 2a, ab) = (8,12) f) (3a + 4b, 5a + 3b ) = ( 21, 13)

MATEMTICA PASSO A PASSO

FUNO
Definio Dados dois conjuntos no vazios A e B, uma funo de A em B uma relao que a cada elemento x de A faz corresponder um nico elemento y de B. Notemos que: A B 1) todo elemento de A deve ser associado a algum elemento de B; x y 2) para um dado elemento de A associamos um nico elemento em B. Empregamos a seguinte linguagem: a) o conjunto A o domnio da funo; b) o conjunto B o contradomnio da funo; c) o elemento y de B, associado ao elemento x de A, denominado imagem de x; d) o subconjunto de B, formado pelos elementos que so imagens dos elementos de A, denominado conjunto-imagem (ou apenas imagem) da funo. Para dar nomes s funes costumamos usar as letras f, g, h e outras. Empregamos tambm a seguinte notao; f : A B ( leia: f de A em B), para indicar uma funo f de A em B; y = f(x) (leia:y = f de x), para indicar que y a imagem de x; D ou D(f) (leia: D de f), para indicar o domnio da funo f; Im ou Im(f) (leia: imagem de f), para indicar o conjunto-imagem de f.

249

PROF. WELLINGTON BRITO

Exemplos 1) Dado A = { 1,2,3,4} consideremos a funo f : A R definida por f(x) = 2x. Temos: Para x= 1, f (1) = 2 . 1 = 2 A Para x = 2, f(2) = 2 . 2 = 4 Para x = 3, f(3) = 2 . 3 = 6 Para x = 4, f(4) = 2 . 4 = 8 R
1 2 4 6 8

A imagem da funo Im(f) = { 2, 4 , 6, 8 }. 2) Determinar a imagem da funo f: D R definida por f(x) = x3 x + 10, sendo D = { 2, 2 0, 1, 2 }. Temos: 1, 3 3 = Para x = 2, f( 2) = ( 2) ( 2) + 10 = 8 + 2 + 10 = 4 2 3 Para x = 1, f( 1) = ( 1) ( 1) + 10 = 1 4 = 1 + 1 + 10 = 10 0 10 Para x = 0 , f(0) = 03 0 + 10 = 10 4 1 16 3 Para x = 1, f(1) = 1 1 + 10 = 10 2 3 Para x = 2, f(2) = 2 2 + 10 = = 8 2 + 10 = 16 Logo, Im(f) = { 4, 10, 16} EXERCCIO FUNO ( II )

1) Seja f : R R a funo definida por f(x) = 3x2 + 1. Calcule: a) f(5) b) f( 5) c) f () 2) Seja f:R R a funo definida por f(x) = a) f( 1) 3) Se f(x) = 1 x 1 x+1 b) f 2 . Calcule X +1
2

c) f ( 2 ) 2 , qual o valor de f(1) + f(2) + f(3)?

( )
1

4) Seja A={0,1,2,3,4} e f:A R a funo definida por f(x)=(x + 1) 2. Determine a imagem de f. 250

MATEMTICA PASSO A PASSO

5) Determine a imagem de cada funo: a) f: A R dada por f(x) = x + 1 e A = x 1 3 , 1 2 , 1, 2, 3

b) f: D R dada por f(x) = x 1 + 1 e D = { 2, 1, 0, 1, 2}. 6) Na funo f:R R definida por f(x) = 7x 3, para que valor de x tem-se f (x) = 18? 7) Dada f(x)= x + 1, calcule o valor de x para o qual se tem f(x) = 2. 8) Uma f (x) = x 1 tem imagem 2x + 1 Im = { 3, 1, 1, 3, 5}. Qual o domnio de f? funo definida por

9) Examine cada relao e diga se ou no uma funo de A em B. Justifique. a) A b) A x B x x x x x x x x x c) 7/3 c) A x x x x x x x x x x B

d) A

Respostas 1) a) 76 b) 76 2) a) 1 3) 3/4 4) Im(f) = {1,4,9,16,25} 5) a) Im (f) = 10 , 5 , 2 3 2 b) Im (f) = {1,2,3,4} 6) x = 3 7) x = 3 9) a) funo b) no funo:existe elementos em A sem corresponder em B c)no funo:existe elemento em A com mais de um correspondente em B. d) funo b) 8/5 c) 2/3

251

PROF. WELLINGTON BRITO

8) D(f) = 2 , 0 , 2 , 4 , 2 7 5 3 Funo Polinomial do 1 Grau Dados os nmeros reais a e b, sendo a 0, podemos considerar a funo que a todo nmero real x faz corresponder o nmero ax + b: f: R R, com f(x) = ax + b ( x R) Esta funo denominada funo polinomial do 1 grau (ou tambm funo afim). O grfico uma reta no paralela a nenhum dos eixos coordenados. Exemplos y 1) f(x) = 2x + 1, onde a = 2 e b = 1. 5 -------- Temos: 4 Para x = 0, y = f(0) = 2 . 0 + 1 = 1 3 ----Para x = 1 ,y = f(1) = 2 . 1 + 1 = 3 2Para x = 2, y = f(2) = 2 . 2 + 1 = 5 1 Para x = 1, y = f( 1) = 2( 1) + 1 = 1 -2 -1 0 1 2 3 4 O grfico a reta desenhada na figura ao lado -1Observe que D(f) = Im(f) = R. -22) f(x) = 2x + 3, onde a = 2 e b = 3. Temos: Para x = 1, y = f( 1) = 2( 1)+ 3 = 5 Para x = 0 ,y = f(0) = 2 . 0 + 3 = 3 Para x = 1, y = f(1) = 2 . 1 + 3 = 1 Para x = 2, y = f(2) = 2 . 2 + 3 = 1 O grfico a reta desenhada na figura ao lado Observe que D(f) = Im(f) = R. -2 y 6--5 43 21--- -1 0 1 2 3 4 5 -1------ x

Funo Crescente e funo decrescente

Observando o grfico de f(x) = 2x + 1 (exemplo 1) notamos que, da esquerda para a direita, isto , aumentando os valores de x, vo tambm aumentando as ordenadas y dos pontos do grfico. Por isso, dizemos que essa funo crescente.

252

MATEMTICA PASSO A PASSO

J na funo f(x) = 2x + 3 (exemplo 2) notemos que, da esquerda para a direita, vo diminuindo as ordenadas y dos pontos do grfico. Por isso, dizemos que essa funo decrescente y y f(x2) ------------f(x1) ----0 x1 x2 x f(x1) ----f(x2) ------------0 x1 x2 x

funo crescente x1 < x2 f(x1) < f(x2) Nota:

funo decrescente x1 < x2 f(x1) > f(x2)

Se a > 0, a funo f(x) = ax + b crescente Se a < 0, a funo f(x) = ax + b decrescente.

Exemplos 1) Em f(x) = 2x + 1 temos a = 2, portanto a > 0. A funo crescente. 2) Em f(x) = 2x + 3 temos a = 3, portanto a< 0. A funo decrescente. Raiz de uma funo Denominamos raiz (ou zero) de uma funo f a todo valor de x para o qual se tem f(x) = 0. No caso da funo afim temos b f(x) = 0 ax + b = 0 ax = b x = a b Logo, a raiz de f(x) = ax + b o nmero a

Nota: Notemos que, para x = b , temos y = f b a a e portanto a raiz da funo a abscissa do ponto onde o grfico corta o eixo dos x. 253

( ) (

= 0 b ,0 a

PROF. WELLINGTON BRITO

Exemplo Determinar a raiz de f(x) = 2x + 1. Temos: 2x + 1 = 0 2x = 1 x = 1 1 2 2 Portanto, a raiz . O grfico corta o eixo dos x no ponto .

3 -----21- 1/2 -1 1;0 2 0 1

------- -1

Sinais da funo afim Estudar os sinais de uma funo y = f(x) significa estabelecer, para cada x D(f), qual das sentenas verdadeira: y > 0, caso a > 0 y y>0 y<0 b a x y>0 b a y<0 x y = 0 ou y < 0 y caso a < 0 Para a funo afim f(x) = ax + b temos dois casos a considerar.

Exemplos 1) Sinais da funo f(x) = 2x + 1. Temos


1 f(x) = 0 2x + 1 = 0 x = 2

y y > 0 1 2 (x> 1 ) 2 x

a = 2 a > 0 f crescente. Ento: x =_ 1 y = 0 2 _ 1 x < y < 0 2

(x < _ 1 ) 2 y < 0

254

MATEMTICA PASSO A PASSO

x >

_ 1 2

y > 0

2) Sinais da funo f(x) = 2x + 3. Temos: f(x) = 0 2x + 3 = 0 x = 3 2 a = 2 a < 0 f decrescente. Ento: x= 3 y=0 2 < 3 y>0 x 2 x > 3 y<0 2 Funo Constante

y>0

(x > 3 ) 2 3 2 y<0 x

(x < 3 ) 2

Dado um nmero real c, podemos considerar uma funo que a todo nmero real x faz corresponder o nmero c: f: R R, com f(x) = c (x R) Esta funo denominada funo constante. O grfico a reta paralela ao eixo dos x, passando pelos pontos de ordenadas y = c. Observe que o domnio D(f) = R e a imagem Im (f) = {c}. y c

Exemplo O grfico da funo f: R R, dada por f(x) = 2, a reta paralela ao eixo dos x pelos pontos de ordenadas y = 2. Note que: Para x = 0 , temos y = f(0) = 2 Para x = 1, temos y = f(1) = 2 Para x = 2, temos y = f(2) = 2 Para x = 3, temos y = f(3) = 2 32255

PROF. WELLINGTON BRITO

1-1 0 1 2 3 4 x

EXERCCIO FUNO - ( III )


1) Faa o grfico de cada funo polinomial abaixo, definida em R. a) f(x) = 2x 4 c) f(x) = 5 2x b) f(x) = x + 2 d) f(x) = x 1 2 3 2) Represente no mesmo sistema de coordenadas os grficos de x y = x , y = 2x e y = . 2 3) Represente no mesmo sistema de coordenadas os grficos de y=x+1 e y=x1. 4) Dada f(x) = x 1 , calcule: 5 2 a) f 5 ( 2) b) f 2 ( 5)

5) Se f(x) = 2x + 1 , qual o valor de f(1) f( 1) ? 3 2 6) Diga se funo crescente ou funo decrescente: a) y = 2x + 8 d) y = 2x 6 g) y = 1 + x 2 x 3x b) y = 3x 9 e) y = 1 h) y = 1 5 2 c) y = 4x + 6 f) y = 2x 1/2 7) Calcule a raiz e d os sinais da funo: a) y = 4x 8 b) y = x + 2 2 c) f(x) = x d) f(x) = 3x 5 8) Determine m para que f seja crescente nos casos: a) f(x) = ( m 1 ) x 1 b) f(x) = (2m + 1)x + (m 1) 9) Para que valores de m a funo f decrescente? a) f(x) = (5m 2) x + 4 b) f(x) = 1 ( 3 m ) x 256

MATEMTICA PASSO A PASSO

10) Dada f(x) = 3x 1 , determine 4 a) os valores de x para os quais f(x) 1 b) os valores de x para os quais f(x) > 0 c) 11) se f(x) = 3x e g(x) = 1 x , para que valores de x temos 4 2 10 f(x) > g(x)? 12) O grfico de y = 2x + b corta o eixo dos x no ponto ( 3;0). Qual o valor de b? 13) Calcule o valor de a sabendo que o grfico de y = ax + 3 passa no ponto ( 1; 1). 14) Calcule a e b sabendo que o grfico de y = ax + b passa nos pontos (4;0) e (0;2). 15) Determine a funo polinomial do 1 grau cujo grfico passa nos pontos(1;-1)e(2;1). 16) Faa o grfico de cada funo. D o domnio e a imagem de cada uma. a) f: R R, f(x) = 3 b) f: R R, f(x) = 1 c) f: R R, f(x) = 5 2 d) f:R + R, f(x) = 1, 5

a) b) c)

17) Uma funo f: R R definida por f(x) = 1 para todo x 0, e f(x) = 1 para todo x < 0. Indicamos: 1, se x 0 F(x) = 1, se x < 0 D os valores de f(1), f(2), f(3), f(0), f( 1), f( 2) e f(( 3). Faa o grfico de f. Qual a imagem de f? 18) Examine o grfico de cada relao, d o domnio e a imagem e diga se ou no grfico de uma funo. a) y 5432 1 0 1 2 3 4 5 6 x 257 c) 3 2 1 -3 0 -2 -1 1 2 3 y

PROF. WELLINGTON BRITO

1 2

b) 4 3 2 1

d)

y 2 1 -1 0 1 -1 -2

-2 x 5) 2 3 b) raiz: x = 4 x=4 y=0 x<4 y>0 x>4 y<0 b) m > 1 2 b) m < 3 b) x > 1 3

0 1 2 3 4 5 Respostas 4) a) 0 b) 21 50

6) crescente : a) b) e) f) g) decrescente : c) d) h) c) raiz: x = 0 x=0 y=0 x<0 y<0 x>0 y>0 d) raiz: x = 0 x=0 y=0 x<0 y>0 x>0 y<0

7) a) raiz : x = 2 x=2 y=0 x<2 y<0 x>2 y>0 8) a) m > 1 9) a) m < 2 5 10) a) x 5 3 11) x > 10 17 12) b) = 6

14) a = 1 , b = 2 2 17) a) f(1) = 1, f(2) = 1, f(3) = 1 f (0) = 1, f( 1) = 1 , f( 2) = 1 e f( 3) = 1 c) Im(f) = { 1, 1} 18) a) D(f) = { 1;6} b) D(f) = { 0;5} 258

13) a = 2

15) f(x) = 2x 3

MATEMTICA PASSO A PASSO

Im(f) = {1,4} Im(f) = {0,4} No funo funo c) D(f) = { -3, 3} d) D(f) = { -2;2} Im(f) = {-2,2} Im(f) = {-2,2} No funo No funo INEQUAES DO 1 GRAU Introduo: Chamamos inequaes do 1 grau s sentena: ax + b > 0, ax + b 0, ax + b < 0, ax + b 0 onde a e b so nmeros reais conhecidos, a 0, e x incgnita. Convm notar, porm que fazendo-se y = ax + b, resolver cada inequao acima significa determinar para quais valores reais de x temos, respectivamente: y > 0, Exemplo: 1) Resolver a inequao 2 x 1 0. y 0 (x 2) y 0, y < 0, y 0. y Isto pode ser feito analisando-se os sinais da funo y = ax + b.

Temos: x x 1 0 2x2 2 2 S = { x R x 2} Note que sendo y = x

0 1

1 2

Grfico de y = x 1 1, 2

2 temos y 0 para x 2. Inequao Produto: Passo a Passo f(x) . g(x) 0, f(x) . g(x) < 0, f(x) . g(x) 0

Dada as funes f: R r e g: R r, cada uma das inequaes: f (x) . g(x) > 0, denominada inequao-produto.

259

PROF. WELLINGTON BRITO

Para resolver tais inequaes procuraremos estudar os sinais dos fatores f(x) e g(x) e, ento, determinar para cada x o sinal do produto f (x) . g(x). Feito isto, podemos estabelecer facilmente o conjunto-verdade da inequao dada.

Questes Comentadas
1) Resolva a inequao (2x 5) (10 3x ) 0. Fazendo f(x) = 2x 5 e g(x) = 10 3x temos: 1) Sinais de f(x) 5 2x 5 = 0 x = 2 2 a = 2 a > 0 f crescente. 2) Sinais de g(x) 10 3x = 0 x = 10 g >0 10 3 5 2 f(x) g(x) f(x) . g(x) + + + + 10 3 + 3 a = 3 a < 0 g decrescente 3) Sinais de f(x) . g(x) f<0 y

y f>0 5 2

g<0

(Note que 5 < 10 ) Como queremos f (x) . g (x) 0, 2 3 o conjunto-verdade { x R 5/2 x 10/3 } 2) Resolver a inequao x ( x 1) (2x 1) < 0. Fazendo y1 = x, 1) Sinais de y1 y2 = x 1 e y3 = 2x 1 temos: 2) Sinais de y2 260 3) Sinais de y3

MATEMTICA PASSO A PASSO

y y1 > 0 y1 < 0 0 x y2 < 0

y y2 > 0 1 x y3 < 0

y y3 > 0 1 2

4) Sinais de y1 . y2 . y3 0 y1 y2 y3 y 1 . y2 . y3 1 2 1

+ + + + + + + + iiiiiiiiiiiiiiii iiiiiiiiiiiiiiiii Como queremos y1 . y2 . y3 < 0, o subconjunto-verdade 1 { x R x < 0 ou < x < 1} 2 EXERCCIO INEQUAO DO 1 GRAU ( I ) 1) Resolva as inequaes do 1 grau: a) 3x + 8 x + 2 b) 2(1 + 2x) 3 (1 x ) > 0 2) Resolva as inequaes: a) 6x 2 6x 3 < 5 3 2 3) Resolva as equaes: a) (4x 1) (3x + 1) > 0 b) (2x + 6) (5 x ) > 0 4) Resolva as inequaes: a) x (x + 3 ) ( 4 x ) > 0 c) x x 1 4 2 d) 3(4x 7 ) (4x 9 ) 8x 11 b) 3x + 1 5 6x + 1 2 > 0

c) ( x + 2) (2x 1) 0 d) (10x 3 ) (5 2x) 0 b) (x + 1) (x 2 ) (2x + 1) < 0

5) Resolva a inequao 3x (x + 3) (3x + 3) 0. 6) Determine os nmeros inteiros x que verificam 261

x ( x + 2) (x 2) (x 4) 0.

PROF. WELLINGTON BRITO

7) D o conjunto-soluo em da inequao (5x + 3) (4x 6) ( 3x + 14) > 0. 8) Quantos so os elementos do conjunto {x N (x 1)(7 x ) >0}?

Respostas: 1) a) S = { x R x 3} b) S = { x R x > 1 } 7 2) a) S = { x R x > 25 } 6

c) S = { x R x 4} d) S = R b) S = { x R x < 1 } 8 c) S = {x R x 1 ou x 2} 2 d) S = { x R 3 x 5 } 10 2

3) a) S = { x R x < 1 ou x > 1 } 3 4 b) S = { x R 3 < x < 5} 4) a) S = {x R x < 3 ou 0 < x < 4}

b) S = { x R x < 1 ou 1 < x < 2} 2 5) a) S = { x R 3 x 1 ou x 0} 6) { 2 , 1,0,2,3,4} 7) S= {2,3,4} 8) 5 elementos. Inequao Quociente: Passo a Passo

Dadas as funes f: R R e g: R R, cada uma das inequaes: f (x) g (x) > 0, f (x) g (x) 0, f (x) g (x) 262 < 0, f (x) g (x) 0

MATEMTICA PASSO A PASSO

denominada inequao quociente. Para resolve-las procedemos como no caso da inequao-produto, fazendo um quadro de sinais para o quociente f (x) . Devemos nos g (x) lembrar ao dar a resposta que, em qualquer dos casos acima, devemos ter g(x) 0.

Questes Comentadas
1) Resolver a inequao 3x + 2 0. 2x 3

Faamos f(x) = 3x +2 e g(x) = 2x 3. Temos: Sinais de f(x)


y f>0 f<0 2 3 x g<0 3 2 x

sinais de g(x)
y g>0 f(x) g(x)

sinais de f(x)/g(x)
_ 2 3 3 2 +

+ + +

f(x)/g(x) +

Como queremos

0, o conjuntoverdade g(x) 2 3 xR x ou x > 3 2 2x + 1

f(x)

< 1. 4x 1 Primeiro vamos coloc-la na forma de inequao quociente, deixando zero no segundo membro: 2x+1 2x+1 2x+1 (4x1) 2x + 2 < 0 <1 1< 0 <0 4x1 4x1 4x 1 4x 1 2) Resolver a inequao 4x 1 263

PROF. WELLINGTON BRITO

Agora faamos f(x) = 2x + 2 e g(x) = 4x 1. Temos: sinais de f(x)


y f>0 1 f<0 x g<0 1 4

sinais de g(x)
y g>0 x

sinais de f(x)/g(x)
_ 1 4 1 + + + +

f(x) g(x) f(x)/g(x)

Como queremos

f(x) g(x)

< 0, o conjunto soluo 1

ou x > 1 4 EXERCCIO INEQUAO DO 1 GRAU (II) a) 2x 8 4x 3 c) >0 b) 5x 1 10x 1 d) x 4x 5 b) 0

x R x <

1)Resolva as inequaes

6 3x x+4

>0

<0

2) Resolva as inequaes a)

x 8x 1

>2

3x 2 >0 (2x + 1) (1 x )

Respostas: 1) a) S= { x R x < 3/4 ou x > 4} c) {x R 4 < x < 2} b) S= { x R 1/10 < x 1/5 } d) {x R x < 5/4 ou x > 0} 2) a) {x R 1/8 < x < 2/15} b) {x R x < 1/2 ou 2/3 < x < 1} Sistema De Inequaes: Passo a Passo

Um sistema de inequaes um conjunto de duas ou mais inequaes consideradas simultaneamente (ligadas pelo conectivo e). O conjunto-verdade do sistema a interseco dos conjuntosverdades das inequaes que o formam.

Questes Comentadas
1) Resolver o sistema 264 x1 x+1 3 2 4 (I)

MATEMTICA PASSO A PASSO

1 x + 2 0 (II) 3 Primeiro resolvemos cada inequao do sistema: (I) x 1 x +1 x1 x+1 6 46 24 3 2 3 2 2(x 1 ) 3 (x + 1) 24 2x 2 3x 3 24 x 29 x 29 (II) 1 x+2 3 0 3 (x + 2) 0 3 x 2 0 x 1 x 1

Agora determinamos o conjunto soluo do sistema, que a interseco de (I) e (II). 29 (I) 1 (I) (II) 29 S = { x R x 29} = ] ; 29] 2) Resolver as inequaes simultneas 2x + 4 3x + 8 2x + 12 Note que devemos ter 2x + 4 3x + 8 e 3x + 8 2x + 12. Portanto temos que resolver o sistema Temos: 2x +4 3x + 8 (I) (II)

3x + 8 2x + 12 (II)

(I) 2x + 4 3x + 8 2x 3x 8 4 x 4 x 4 (II) 3x + 8 2x + 12 3x 2x 12 8 x 4 4 (I) 4 (II) 4 4 265

(I) (II)

S = { x R 4 x 4} = [ 4;4] EXERCCIO INEQUAO DO 1 GRAU ( III ) Resolva os sistemas. 1) 3x 6 > 0 3) 2(x 1) ( 2x 1 ) 0 3x + 12 > 0 5x 2( x 2 ) 0 2) 5x + 1 < 3x + 2 6x 2 < 8x +4 4) 1 3(2 x) 2 (5x 1 ) 6x 3(x + 1 ) 1 7x

PROF. WELLINGTON BRITO

Resolva as inequaes simultneas. 5) x 1 3 2x 3x 2 6) 1 < 6x 1< 6( 1 x ) 7) x < x + 1 < 4x 2 2


Respostas: 1) S = { x R 2 < x < 4} 3) S= {x R x 4/3 } 6) S= { x R 0 < x < 7/12} 2) S= { x R 3 < x < 1/2 } 4) S = 5) S= { x R 1 x 4/3 } 7) S={ x R x > 1}

FUNO DO 2 GRAU

Equao do 2 Grau Chamamos equao do 2 grau sentena: ax2 + bx + c = 0

onde a, b, e c, so nmeros conhecidos, a 0 e x a incgnita. Exemplos 1) 2x2 + 3x + 1 = 0, onde a = 2, b = 3 e c = 1. 2) x2 2x = 0, onde a = 1, b = 2 e c = 0. 3) 4 x2 1 = 0, onde a = 4, b = 0 e c = 1. Nota: Estaremos considerando equaes no conjunto universo U = R, exceto quando for citado outro universo. Assim, estaremos sempre interessados em calcular as razes reais da equao. Resoluo das equaes incompletas: Passo a Passo As equaes do 2 grau que tm b = 0 ou c = 0 so chamadas equaes incompletas.

Questes Comentadas
1) Resolver a equao x2 2x = 0 Comeamos colocando x em evidencia: x . (x 2) = 0. 266

MATEMTICA PASSO A PASSO

Lembrando que um produto igual a zero somente se pelo menos um dos fatores zero, devemos ter: x = 0 ou x 2 = 0 Logo, o conjunto soluo S = { 0; 2}. 2) Resolver a equao 4x2 1 = 0. Temos: 4x2 1 = 0 x2 = 1 x = 1 = 1 4 4 2 O conjunto-soluo S = _ 1 ; 1 2 2 3) Resolver a equao 2x2 + 8 = 0 Nenhum nmero real raiz da equao, porque x2 0, x R. Ento, o conjunto-soluo S = Equaes Completas: Passo a Passo Vamos resolver a equao ax2 + bx + c = 0 com a 0. Temos: x = b b2 4ac 2a Temos: 2x2 + 8 = 0 x2 = 4

Nota: Denominamos b2 4ac de discriminante da equao do 2 grau, que representamos pela letra grega ( leia: Delta).

Questo Comentada
Resolver a equao 2x2 + 3x + 1 = 0 Como a = 2, b = 3 e c = 1 temos: x= 3 32 4 . 2 . 1 2.2 3 1 4 3 1 4 x = = 3 98 4 3 + 1 4 = = 2 4 = 1 2

x =

x = 3 1 = 4 = 1 4 4 1 2 267 ; 1

O conjunto soluo S =

PROF. WELLINGTON BRITO

Condies de Existncia de Razes Reais:

Observando a deduo da frmula de Bscara podemos concluir que: a equao do 2 grau tem razes reais se, e somente se, 0 As razes so dadas por: x= b 2a Temos ainda: > 0 as duas razes so nmeros reais e distintos. = 0 as duas razes so nmeros reais iguais. < 0 no existem razes reais.

Exemplos. 1) Na equao 2x2 + 4x + 1 = 0, = b2 - 4ac = 42 4 . 2 . 1 = 16 8 = 8 Como > 0, a equao possui duas razes reais e distintas. 2) Na equao 9x2 + 12x + 4 = 0, = b2 - 4ac = 122 4 . 9 . 4 = 144 144 = 0 Como = 0, a equao possui duas razes reais e iguais. 3) Na equao 2x2 + 5x + 9 = 0, = b2 - 4ac = 52 4 . 2 . 9 = 25 72 = 47 Como < 0, a equao no possui razes reais. O conjunto- soluo em R = EXERCCIO EQUAO DO 2 GRAU 1) Resolva as equaes (no conjunto universo U = R ): a) x2 36 = 0 e) 2x2 1 = 0 i) x2 + 5x = 0 2 2 b) 9x 4 = 0 f) 3x 16 = 0 j) 2x2 11x = 0 2 2 c) 5x 1 = 0 g) 25x 8 = 0 k) 3x2 4x = 0 x d) 2x2 + 18 = 0 h) 9x2 + 1 = 0l) x2 + =0 2 2) Resolva as equaes: a) x2 + 2x 15 = 0 e) 12x2 5x 3 = 0 2 b) 2x 5x + 2 = 0 f) x2 6x + 7 = 0 2 c) x 6x 7 = 0 g) x2 + 2x 2 = 0 268

MATEMTICA PASSO A PASSO

d) 6 x2 + 5x 1 = 0

h) 4x2 4x 17 = 0

3) Resolva a equao x3 + x2 ( x 1) + x ( x 2 ) 3 ( x 3) = 7 4) Resolva as equaes: x2 x 2 x+1 x2 1 a) + = 0 b) =0 12 2 3 2 3 5) Calcule e diga, quantas razes reais distintas possui cada equao. a) 9x2 + 6x + 2 = 0 b) 9x2 + 6x + 1 = 0 c) 9x2 + 6x 1 = 0 d) x2 + 5x + 4 = 0 e) x2 + 4x + 4 = 0 f) x2 + 3x + 4 = 0

6) Para que valores de m a equao 9x2 + 6x + m = 0 admite razes reais? 7) Para que valores de k a equao x2 + k x + 4 = 0 possui razes reais e iguais? 8) Para que valores de m a equao 2x2 + x + m = 0 possui razes reais distintas? 9) Para que valores de p a equao px2 + 2x + 3 = 0 no possui razes reais? 10)Ache c de modo que a equao x2 + 8x 2c = 0 tenha razes reais. 11) Calcule os valores de r para os quais a equao x2 + 2rx + (r + 2) = 0 possui razes reais iguais. Respostas: 1- a) S = { 6, 6 } e) S = 2 , 2 2 b) S = 2 , 2 3 3 c) S = 5 , 5 5 5 d) S = 2- a) S = { 5, 3} b) S = 1 , 2 2 2 i) S = { 0, 5 } j) S = k) S= 0 , 11 2 0, 4 3

f) S= 4 3 , 43 3 3 g)S= 2 2 , 22 5 5 h) S = e) S= 1,3 3 4

L) S= 1 , 0 2

f) S = { 3 2 , 3 + 2 } 269

PROF. WELLINGTON BRITO

c) { 1 , 7 } d)S= 3- S= 1 , 1 3 2 2,1 2

g) S= {13, 1+ 3 } h) S = 1 + 3 2 , 1 32 2 2 4 - a) S = { 4,2} b) S = 1, 5 2

5 - a) = 36 , nenhuma b) = 0 , duas iguais c) = 72 , duas distintas 6- m1 7- k=4 8- m < 1 8 9- p > 1 3

d) = 9 , duas distintas e) = 0 , duas iguais f) = 7 , nenhuma 10- c 8 11- R = 2 ou R = 1

Funo Do 2 Grau

Dados os nmeros reais a, b e c, sendo a 0, podemos considerar a funo que a todo nmero real x faz corresponder o nmero ax2 + bx + c. f: R R, com f (x) = ax2 + bx + c. (x R) Esta funo denominada funo polinomial do 2 grau ( ou tambm funo quadrtica). O grfico uma curva denominada parbola. Exemplos 1) f(x) = x2 4x + 3, onde a = 1, b = 4 e c = 3. Temos: y s Nota: -O grfico a parbola desenhada na figura
-8 -7 -6 -5 -4 3 21ao lado. Observe que: D( f ) = R e Im ( f) = { y Ry 1} =[ 1;[. Dizemos que essa parbola tem concavidade para cima. A curva simtrica em relao reta s assinalada na figura. O ponto v onde o eixo de simetria s corta a curva denominada vrtice da parbola.

270

MATEMTICA PASSO A PASSO

-2 -1 0 1 2 3 4 - 1---- - v -2 Y 4 ----- V 3 --------210 -1 1 2 3 2 3 4 5

2) f(x) = x2 + 2x + 3, onde a = 1, b = 2 e c = 3. Temos: Nota:


O grfico a parbola desenhada na figura ao lado. Observe que D(f )=R e Im(f )={yRy 4} = ]; 4]. 4 x

-2

Dizemos que essa parbola tem concavidade para baixo.

Concavidade Na funo y = ax2 + bx + c,

quando a > 0, o grfico uma parbola de concavidade para cima; quando a < 0, o grfico uma parbola de concavidade para baixo.

Observe no exemplo 1 que a = 1, portanto a > 0 e a parbola cncava para cima.No exemplo 2 temos a = 1, portanto a < 0 e a parbola cncava para baixo. Vrtice Passo a Passo Quando vamos desenhar uma parbola importante que fique bem claro qual o vrtice da mesma. Por isso, interessante que saibamos previamente determinar o vrtice. y Clculo da abscissa do vrtice xv O fato da curva ser simtrica s eixo de simetria em relao reta s significa que se tomamos dois pontos y1=y2 ---------------------da parbola de abscissas yv -------------- v xv + k e xv k, k R, esses pontos tm a mesma ordenada y. b xv= 2a Clculo da ordenada do vrtice y v 271

xv 1

xv

xv + 1 x

PROF. WELLINGTON BRITO

O clculo de yv pode ser feito substituindo x por b temos: xv na funo. Para x = xv = 2a v y = 4a Exemplo Em y = x2 4x + 3 temos a = 1, b = 4 e c = 3. b ( 4) 4 xv = = = = 2 e yv = 22 4 . 2 + 3 = 4 8 + 3 = 1 2a 2.1 2 b2 + 4ac ( 4)2 + 4 . 1 . 3 4 Ou ento:, yv = = = = =1 4a 4a 4.1 4 Logo, V = ( 2; 1). EXERCCIO FUNO DO 2 GRAU ( I ) c) y = 4x2 + 2x + 1 d) y = x2 + 2x

1) Determine o vrtice de cada parbola. a) y = 3x2 6x 2 b) y = x2 + x _ 1 2 4

2) Para que valores de m o grfico de y = (m 4) x2 2x + m uma parbola cncava para cima? 3) O vrtice da parbola y = x2 + bx + c e o ponto V ( 3;1). Calcule b e c. 4) Sabe-se que a parbola y = x2 + bx + 2b passa pelo ponto (1;7). Qual o valor de b? 5) Calcule b e c sabendo que a parbola y = x2 + bx + c passa pelos pontos (1;1) e (2.6). Nos exerccios de 6 a 10 temos funes quadrticas f(x) = ax2 + bx + c. para cada uma delas diga se o grfico parbola cncava para cima ou para baixo, ache o vrtice e depois faa o grfico. 6) f(x) = x2 2x + 2 272

MATEMTICA PASSO A PASSO

7) f(x) = x2 2x 3 8) f(x) = x2 + 5x 3 9) f(x) = x2 2x + 1 10) f(x) = x2 + 4x Respostas: 1) a) V = ( 1; 5 )

c) V =

1 ;3 4 4

b) V = 1 ; 5 4 16 2) m > 4 3) b = 6 e c = 10

d) V= _ 2 ; 1 2 2 4) b = 2 5) b = 2 e c = 2

Valor Mximo ou Mnimo:

Vamos determinar a imagem da funo quadrtica f(x) = ax 2 + bx +c: Im = { y R (x R f(x) = y)} Devemos descobrir os valores de y para os quais existe x real satisfazendo f(x) = yv Temos: Caso a > 0, a funo Im 2 f(x) = ax + bx + c tem imagem Im = { y R y yv }. yv Observamos que, da esquerda para a direita, v os valores da funo vo diminuindo at chegarmos em x = xv e depois eles vo aumentando. Dizemos xv que f decrescente no intervalo ] ; xv ] e crescente b em [ xv ; + [ . Em x = xv = a funo tem o seu 2a valor mnimo, que f(x) = yv = .Tambm se diz 4a que x = xv ponto de mnimo de f, neste caso.

273

Caso a < 0, a funo

PROF. WELLINGTON BRITO

f(x) = ax2 + bx + c tem imagem Im = { y R y yv }.

y ---------v
v

xv

Neste caso a funo crescente no intervalo ] ; xv ] e descrescente em [ xv ; + [. b Em x = xv = a funo tem o seu valor mximo, 2a que f(x)= yv= .Tambm se diz , neste caso, que 4a x = xv, ponto de mximo de f.

Exemplos 1) Na funo f(x) 2x2 + 3x + 4 temos a = 2, portanto a > 0 (parbola cncava para cima). b 3 xv = = y 2a 4 v b2 + 4ac 23 yv = = = 4a 4a 8 32 + 4 . 2 . 4 23 = = _ 3 x 4.2 8 4 O ponto de mnimo de f x =_ 3 Im (f ) = y R y 23 o valor mnimo de f : 23 4 8 8 2) Na funo f(x) 3x2 + 2x + 1 temos a= 3, portanto a < 0 (parbola cncava para baixo) y xv = yv = = b 2a = 2 = 1 4 -= 4 3 1 274 x 6 3 b2 + 4ac =

= 4a 4a 22 + 4 . ( 3) . 1

MATEMTICA PASSO A PASSO

4 ( 3)

3 1 3

3 Im (f ) = y R y 4 3

O ponto de mximo de f x = o valor mximo de f : 4 / 3

Questo Comentada Determinar a rea mxima que pode ter um retngulo de permetro igual a 20 cm. Lembremos que a rea S do retngulo de base x e altura h S = x . h. Como o permetro 20 cm vem: x 2x + 2h = 20 x + h = 10 h = 10 x Logo, h h S = x (10 x ) = 10x x2 = x2 + 10x x Observe que a rea S uma funo quadrtica de x, com a = 1 < 0. Ento S tem um valor mximo que : b2 + 4ac 102 + 4 . ( 1) . 0 100 yv = = = = = 25 4a 4a 4 . ( 1) 4
A rea mxima 25 cm2. EXERCCIO FUNO DO 2 GRAU ( II ) Nos exerccios de 1 a 4 d o ponto de mximo ou de mnimo de f, o valor mximo ou mnimo de f e determine a imagem dela. 1) f(x) = 5x2 + x + 2 2) f(x) = x2 + 3x + 6 3) f(x) = 8 x + 2x2 4) f(x) = 6 x x2 5) Calcule m de modo que a funo f(x) = mx2 + 2x + 1 tenha um valor mnimo igual a 1 4 6) Para que valor de x na funo f(x) = 3 x 2 12x + 7 tem o seu valor mnimo? 7)Uma bola lanada para cima, verticalmente, tem sua altura h (medida em metros) dada em funo do tempo t decorrido aps o lanamento ( t medido em segundos) pela frmula h = 20t 5t 2. Calcule: a) o tempo decorrido at a bola chegar altura mxima; b) a altura mxima da bola; c) o tempo decorrido at a bola cair no solo (isto , at que se tenha novamente h=0) 275

PROF. WELLINGTON BRITO

8) Calcule a rea mxima que pode ter um retngulo de permetro igual a 40 cm. 9) Calcule o valor mximo que pode ter o produto pq sabendo que p e q so nmeros reais positivos e p + q = 1. Respostas.
1)O ponto mnimo _ 1 10 O valor mnimo de f 39 20 Im (f) = y R y 39 20 3 , 2) O ponto mximo 2 O valor mximo de f 33 , 4 Im (f) = y R y 33 4 3)O ponto mnimo 2, O valor mnimo de f 8. Im (f) = { y R y 8} 4)O ponto mximo _ 1 2 O valor mximo de f 25 4 Im (f) = y R y 25 4 5) m = 4 3 6) x = 2 8) 100 cm2 9) 1 4 7) a) 2 segundos b) 20 metros c)4segundos

Razes e sinais da funo quadrtica : Passo a Passo Para encontrar as razes da funo f(x) = ax2 + bx + c, a 0, devemos resolver a equao do 2 grau: ax2 + bx + c = 0 2 Sendo = b 4ac os seguintes casos podem ocorrer: 1) > 0 h duas razes reais distintas x e x Neste caso a parbola corta o eixo dos x nos pontos de abscissas x e x. Vejamos como so os sinais de f(x). Caso a > 0 y + x x + x y Caso a < 0 + x x x

( x < x ou x > x ) f(x) > 0 x" < x < x f(x) < 0 ( x = x ou x = x ) f(x) = 0 276

( x < x ou x > x ) f(x) < 0 x" < x < x f(x) > 0 ( x = x ou x = x ) f(x) = 0

MATEMTICA PASSO A PASSO

2) = 0 h duas razes reais iguais x = x Neste caso a parbola tangencia o eixo dos x no ponto de abscissa x. Vejamos os sinais de f(x). Caso a > 0 Caso a < 0 y y x

+ x x x f(x) > 0 x = x f(x) = 0

+ x x x f(x) < 0 x = x f(x) = 0

3) < 0 no h razes reais Neste caso a parbola no tem nenhum ponto comum com o eixo dos x.Vejamos os sinais de f(x). Caso a > 0 Caso a < 0 y y + + + x x

f(x) > 0, x R f(x) < 0, x R Exemplos 1) Estudar os sinais de f(x) = x2 6x + 8. Comeando calculando e determinando as razes de f(x). = ( 6) 2 4 . 1 . 8 = 4 x = 4 277 y

razes: x =

6 4 2

PROF. WELLINGTON BRITO

x = 2

+ 2 4

+ x

A parbola corta o eixo dos x nos pontos de abscissas 2 e 4. como a =1 > 0, ela tem concavidade para cima. Temos, ento: (x < 2 ou x > 4) f(x) > 0 (x = 2 ou x = 4) f(x) = 0 2 < x < 4 f(x) < 0 2) Estudar os sinais de f(x) = x2 + 2x 1 Temos: = 22 4 . ( 1) ( 1) = 0 =1 2 A parbola tangencia o eixo dos x no ponto de abscissa 1. Como a = 1< 0, ela tem concavidade para baixo, ento: x 1 f(x) < 0 x = 1 f(x) = 0 razes: x = 2 0

y 1 x

EXERCCIO FUNO DO 2 GRAU (III) Nos exerccios de 1 a 5 estude os sinais de f(x). 1) f(x) = 6x2 5x + 1 2) f(x) = x2 2x + 3 3) f(x) = x2 + 4x + 4 4) f(x) = x x2 5) f(x) = x2 9 6) Determine os valores de c para os quais temos: x2 + 4x + c > 0, x R. 7) Calcule os valores de m para os quais temos: 2x2 + 6x + (m 1) < 0, x R. Respostas 1) f > 0 x < f<0 1 3 1 3 ou x > 1 2 1 6) c > 4 7) m < _ 7 2 278

< x < 1

f=0x=

2 1 ou x =

MATEMTICA PASSO A PASSO

2) f > 0 3 < x < 1 f < 0 x < 3 ou x > 1 f = 0 x = 3 ou x = 1 3) f > 0 x - 2 f<0xR f=0x=2 4) f > 0 0 < x < 1 f < 0 x < 0 ou x > 1 f = 0 x = 0 ou x = 1 5) f > 0 x < 3 ou x > 3 f<0 3< x<3 f = 0 x = 3 ou x = 3

INEQUAO DO 2 GRAU Introduo: Chamamos inequaes do 2 grau s sentenas: ax + bx + c > 0, ax2 + bx + c 0, ax2 + bx + c < 0, ax2 + bx + c 0 onde a, b e c so nmeros reais conhecidos, a 0, e x a incgnita. Fazendo y = ax2 + bx + c, resolver cada inequao acima significa determinar para quais valores reais de x temos, respectivamente: y > 0, y 0, y < 0, y 0
2

Isto pode ser feito analisando-se os sinais da funo y = ax 2 + bx + c.

Questes Comentadas
1. Resolver a inequao 2x2 5x + 2 > 0. Comeamos estudando os sinais de y = 2x2 5x + 2. = ( 5)2 4 . 2 . 2 = 9 279 + + y

razes: x = 5 3 4

PROF. WELLINGTON BRITO

x = 2 1 2 2 x

x = 1 2 a = 2 > 0 parbola cncava para cima.

Resolvendo 2x2 5x + 2 > 0 significa dar os valores de x que tornam verdadeira a sentena y > 0. Observando o grfico notamos que y > 0 verdadeiro para x < 1 ou x > 2. 2 1 Resposta: S= x R x < ou x > 2 2 2. Resolver a inequao x2 6x 7 0
2

Fazemos y = x 6x 7 = ( 6)2 4. 1 . ( 7) = 64 + + 6 8 x = 7 razes:x = 2 x = 1 1 7 x a = 1 > 0 concavidade para cima. Temos y < 0 para 1 < x < 7, e y = 0 para x = 1 ou x = 7. Logo, y 0 para 1 x 7. S = { x R 1 x 7 }. Inequaes Produto e Quociente: Passo a Passo Novamente, a soluo ser feita analisando-se os sinais das funes y = ax + b e y = ax2 + bc + c.

Questes Comentadas 1. Resolver a inequao 3x2 2x 1 < 0. 1 4x2


1) Sinais de y1 = 3x2 2x 1 y + _ 1 3 + _ 1 x _ 1 2 + 2) Sinais de y2 = 1 4x2 y + _ 1 2 1 + 1 2 _ x

3) Sinais de y1 / y2 _ 1 _ 1 2 3 y1 + + y2 + + y1 / y2 + 280

MATEMTICA PASSO A PASSO


1 < 0 a resposta : y2 1 1 1 S = x R x < _ ou _ < x < ou x > 1 2 3 2 1 1 1 = ] ;_ [] ; [] 1; [ 2 3 2

Como queremos

2. Resolver a inequao (x2 + 2x 3 ) (4x 1) > 0. Faamos y1 = x2 + 2x 3 e y2 = 4x 1. Vamos estudar os sinais y1 e y2 separadamente e depois montar o quadro de sinais do produto y1 . y2 . 1) Sinais de y1 = 22 4. 1 . ( 3) = 16 1 razes: x = 2 4 2 3 a = 1 > 0 conc. para cima 2) Sinais de y2 = 1 4x2 raiz: 4x 1 = 0 x = a = 4 > 0 y2 1 4 _ 1 4 3) Sinais de y1 . y2 -3 y1 y2 y 1 . y2 + + 1 4 + 1 + + + + +

Como queremos y1. y2 > 0, a resposta : S = x R 3 < x < 1 4 ou x > 1 = ]3; 1 4 []1;+[

EXERCCIO INEQUAO DO 2 GRAU 1) Sendo U = R, resolva as inequaes do 2 grau: 281

a) x2 9x 10 > 0 b) 6x x2 > 0 c) x2 4 d) x2 + 36 > 12x e) x2 < x + 1 2 8

PROF. WELLINGTON BRITO

h) (x + 5 )2 20x i) 4m 3 3 + m(m 1) 2 4 j) (t + 1) ( t 1) 3 k) (a + 2)2 4(a2 + 2) L) x (x2 + 1) + (x + 2) (x 2) x2 ( 2 + x) m) r3 1 < (r 1) (r2 + 4 ) n) t2 < 5 h)


2

f) x2 + 2x 12x 1

g) m2 m + 1 < 2m2 + m + 2 a) ( x2 5x + 4 ) ( 2x2 x 1) > 0 b) ( x 4) (2x x ) 0


2 2

2) Determine o conjunto-soluo das inequaes abaixo,sendo U=R: x2 16 4 x2 x + 6x + 5


2 2

< 0 0 0

x + 4x + 3 i) j)

c) ( x + x2 ) ( 5x 1) < 0 d) x (x + 3x +2 ) 0 e) (2x 3) (x 3x 10) (1 x ) > 0


2 2

5x 2 x (x 9 )
2

f) x ( x + 1) (x 1 ) 0
2

g) 4x 3x 1 > 0 2x2 x 1
2

Respostas: 1)a) S= {x Rx < 1 ou x > 10 } b) S= {x R0 < x < 6} c) S= {x Rx 2 ou x 2} d) S= {x Rx 6} e) S= {x R1 5 < x < 1 + 5 } 2 2 1 f) S = _ 2 g) S = { m R m 1 } 2) a) S = x R x < _ 1 ou x > 4 2 282 h) S = { 5 } i) S = { 3/2} j) S = R k) S = l) S = m) S = { r R 1 < r < 3} n) S= { t R 5 < t < 5 }

b) S = { x R 2 x 0 ou x = 2 } c) S = x R x < 1 ou 0 < x < 1 5

MATEMTICA PASSO A PASSO

d) S = { x R x 2 ou 1 x 0 } e) S = x R x < 2 ou 1< x < 1 ou f) S = { x R x 0 ou x 1 } g) S = xRx < _ 1 2 ou _ 1 4 3 2 < x< 5

< x < 1 ou x > 1

h) S = { x R 4 < x < 1 ou 3 < x < 4 } i) S = { x R x < 5 ou 2 x < 1 ou x 2 } j) S = x R x < 3 ou 0 < x 2 5 ou x > 3

QUESTES DE CONCURSOS E VESTIBULARES 01-(PUC-SP)Os pares ordenados (2,3) , (3,3) e (1,4) so elementos do conjunto A x B. Ento: a) (1,3) , (2,4) e (3,4) esto necessariamente em A x B. b) (1,1) , (1,3) , (2,2), (2,4) e (3,4) esto necessariamente em A x B. c) (1,1) , (2,2) e (4,4) esto necessariamente em A x B. d) (3, 2) e (4, 1) esto necessariamente em A x B. e) Os elementos dados podem ser nicos de A x B. 02-(UFMT)Sejam os conjuntos A e B tais que: A x B = {( 1; 0),( 2 ; 0 ),( 1; 2),( 2 ; 2), (1;3),(2;3)} O nmero de elementos do conjunto A B : a) 0 b) 1 c)2 d)3 e)4 03-(U.E.C.E) Se P = { 1,2,5,7,8}, ento o nmero de elementos do conjunto W = {(x,y) p2; x < y} : a) 8 b) 9 c) 10 283 d) 11 04-(F.SANTANA) Seja a relao R, de A em A, definida por

(x;y) R

PROF. WELLINGTON BRITO

y = x , se x par y = x + 1, se x impar Se A = { 0, 1, 2, 3, 4, 5, 6, 7, 8, 9}, o nmero de pontos do grfico cartesiano de R : a) 5 b) 6 c) 8 d) 9 e) 10 05- (UF-Uberlndia) Dados os conjuntos A = {1; 0; 1; 2} e B {0; 1; 2; 3; 4} qual, entre as relaes seguintes, representa uma funo de A em B? a) {( 1; 0), (0;1), (1;2), (1;3),(2; 4) b) {(1; 1), (0;1), (1;0), (1;2)} c) {(0; 1), (1;0), (2;1), (4;2)} d) {( 1;1), (0;0), (1;1), (2;4)} e) {( 1; 1), (0;2), (0;3), (1;4), (2;4)} 06- (PUC-SP) Os conjuntos A e B possuem, respectivamente, 3 e 4 elementos. Quantas funes de A em B tm o conjunto imagem igual a B? a) nenhuma b)34 c)43 d)3! e) 4! 07- (U.F.PE) Dados os conjuntos A = { a, b, c, d} e B = {1,2,3,4,5}, assinale a nica alternativa que define uma funo de A em B. a){(a,1), (b,3), (c,2)} b){(a,3), (b,1), (c,5),(a,1)} c) {(a,1), (b,1), (c,1), (d,1)} 08- (U.F.PA) Sejam os conjuntos A ={1,2} e B = {0,1,2}. Qual das afirmativas abaixo verdadeira? a: : x 2x uma funo de A em B. b: : x x + 1 uma funo de A em B. c: : x x2 3x + 2 uma funo de A em B. d: : x x2 x uma funo de B em A. e: : x x 1 uma funo de B em A. 09 (U.F.PA) Dada a funo de A = { 0,1,2} em B = {-2,-1,0,1,2} definida por (x) = x 1, qual o conjunto imagem de ?. a) { 1, 0 , 1} c) {0,1,2 } 284 e){0, 1,2} d) {(a,1), (a,2), (a,3), (a,4),(a,5)} e) {(1,a), (2,b), (3,c), (4,d),(5,a)}

MATEMTICA PASSO A PASSO

b) { 2, 1, 0, 1, 2}

d) { 2, 1, 0}

10-(GV-SP) O nmero de unidades produzidas (y) de um produto, durante um ms, funo do nmero de funcionrios empregados (x) de acordo com a relao: y = 50x . Se 49 funcionrios esto empregados, podemos afirmar que: a) o acrscimo de um funcionrio aumenta a produo mensal em 50 unidades. b) o acrscimo de 15 funcionrios aumenta a produo mensal em 75 unidades. c) o acrscimo de 32 funcionrios aumenta a produo mensal em 100 unidades. d) o acrscimo de 51 funcionrios aumenta a produo mensal em 120 unidades.

11- (BB) Suponha-se que o nmero (x) de funcionrios necessrios para distribuir, em um dia, contas de luz entre x por cento de moradores, numa determinada cidade, seja dado pela funo. 300x (x) = 150 x Se o nmero de funcionrios necessrios para distribuir, em um dia, as contas de luz foi de 75, a porcentagem de moradores que as receberam : a)25 b) 30 c) 40 d) 45 e) 50 12 (BB) Em uma experincia realizada com camundongos, foi observado que o tempo requerido para um camundongo percorrer um labirinto, na ensima tentativa, era dada pela funo 12 minutos.Com relao a essa experincia (n) = 3 + n pode-se afirmar que o camundongo : a) consegue percorrer o labirinto em menos de trs minutos. b) gasta cinco minutos e 40 segundos para percorrer o labirinto na quinta tentativa. 285

PROF. WELLINGTON BRITO

c) gasta oito minutos para percorrer o labirinto na terceira tentativa. d) percorre o labirinto em quatro minutos na dcima tentativa. e) percorre o labirinto, numa das tentativas, em trs minutos e 30 segundos. 13 (CEF) A funo que representa o valor a ser pago aps um desconto de 3% sobre o valor de x de uma mercadoria : a) f(x) = x 3 b) f(x) = 0,97x c) f(x) = 1,3x d) f(x) = 3x e) f(x) = 1,03x

14 ( U.E.Londrina) Seja a funo : R tal que (x) = ax + b. Se os pontos (0; 3) e (2; 0) pertencem ao grfico de , ento a + b igual a: a) 9 b) 3 c) 2 d) _ 3 e) 1 2 3 2 15 (FGV) O grfico da funo (x)= mx + n passa pelos pontos (4,2) e ( - 1 , 6). Assim, o valor de m + n : a) _ 13 b) 22 c) 7 d) 13 e) 2,4 5 5 5 5 16 (PUC-SP) No conjunto dos nmeros reais, a equao ax = b, na incgnita x: a)no pode ter infinitas solues d)tem infinitas solues se b 0 b) sempre tem soluo e) tem soluo nica se a 0 c) s tem soluo se a 0 17 (PUC-MG) Uma funo do 1 grau tal que ( 1) = 5 e ( 3)= 3. Ento (0) igual a: a) 0 b) 2 c) 3 d) 4 e) 1 18-( UF-Viosa) Uma funo dada por (x) = ax + b, onde a e b so nmeros reais. Se ( 1) = 3 e ( 1) = 1, ento ( 3) o nmero: a) 1 b) 3 c) 3 d) 5 e) 5 19- (U.E.BA)A funo , de R em R,definida por (x) = (k2 1) .x + 3, crescente se, e somente se: a) k 1 e k 1 b) k = 1 ou k = 1 c) k > 0 d) 1 < k < 1 286

e) k < 1 ou k > 1

MATEMTICA PASSO A PASSO

20 (CEF) Para produzir um objeto, uma firma gasta $ 1,20 por unidade. Alm disso, h uma despesa fixa se $ 4.000,00, independentemente da quantia produzida. O preo de venda de $ 2,00 por unidade. Qual o nmero mnimo de unidades, a partir do qual a firma comea a ter lucro? a) 1 800 b) 2 500 c) 3 600 d)4 000 e) 5 000 21 (BB)Um botnico mede o crescimento de uma planta,em centmetros, todos os dias. Ligando os pontos colocados por ele num grfico, resulta a figura ao lado.Se for mantida sempre esta relao entre tempo e altura, a planta ter, no 30 dia, uma altura igual a: a) 5 cm altura em cm b) 6 cm 2 c) 3 cm d) 15 cm 1 e) 30 cm 5 10 tempo em dias 22-(BB) Duas funes importantes em finanas so: Receita Total: RT = P x Q e Custo Total: CT = CF + CVU x Q, onde : P = preo de venda unitrio: CF = Custo fixo; CVU = custo varivel unitrio; Q = quantidade produzida e vendida. A Metalrgica Atlas S.A. produz uma pea, para a qual so conhecidos os seguintes dados (mensais): P = $ 5.000,00 ; CF=$ 100.000,00;CVU=$ 2.000,00;Lucro= L = RT CT = 800.000,00 A Metalrgica Atlas, a fim de enfrentar seus concorrentes, decide reduzir em 20% o preo de venda unitrio (P), mas pretende obter o mesmo lucro, atravs do aumento em Q. Este aumento (em %) dever ser de: a) 20% b) 150% c) 40% d) 50% e) 10% 23- ( FUVEST) A tabela abaixo mostra a temperatura das guas do oceano Atlntico (ao nvel do equador) em funo da profundidade: Profundidade Temperatura Superfcie 27 C 100m 21C 500m 7 C 1 000m 4 C 3 000m 2,8 C

287

PROF. WELLINGTON BRITO

Admitindo que a variao da temperatura seja aproximadamente linear entre cada duas das medies feitas para a profundidade, a temperatura prevista para a profundidade 400m de: a) 16 C b) 14 C c) 12,5C d) 10,5 e) 8 C 24- (UF-GO) O menor mltiplo de 3 que satisfaz a inequao x + 5 < 2x 1 : a) 12 b) 9 c) 6 d) 3 e) 0 25-(UF-SE) Quantos nmeros inteiros, estritamente positivos, satisfazem a inequao: 3 x+ < 3x 4 ? 2 a) nenhum b) dois c) trs d) quatro e) infinitos 26-(PUC-SP) O menor nmero inteiro k que satisfaz a inequao 8 3(2k 1) < 0 : a) 2 b) 1 c) 0 d) 1 e) 2

27-(CESGRANRIO) Os valores positivos de x, para os quais (x 1) (x 2) (x +3) < 0, constituem o intervalo aberto: a) (1,3 ) b) (2,3) 4x a) 2 1+x b) 3 c) (0,3) d) (0, 1) e)(1,2)

28-(U.E.Londrina)Quantos nmeros inteiros satisfazem a inequao 0? c) 4 d) 5 x+3 2x 5 ; 3 5 ;+ 2 e) 6 0, em R ,

29-(UF-SE) O conjunto soluo da inequao : a) - 3; 5 2 b) - 3; 5 2 c) d) - 3; 5 2 ; 3 e)

288

MATEMTICA PASSO A PASSO

30- (UFC) O domnio da funo real g(x) = a) { x R ; x > 7} b) { x R ; x 2}

x 2 x7

c) { x R ; 2 x < 7 }

d) { x R ; x 2 ou x > 7} 31-(PUC-SP) Qual a funo do 2 grau cuja nica raiz 3 e cujo grfico passa pelo ponto A = ( - 2 ; 5) ? a) f(x) = 5x2 + 30x + 45 b) f(x) = _ 5 x2 _ 5 x + 15 4 4 2 c) f(x) = 5x2 20x 15 32- (F.Santana) sejam 5 e _ 3 , respectivamente, a soma e 2 2 o produto das razes da equao 2x2 + bx + c = 0. O valor de b + c : a) 8 b) 2 c) 1 d) 2 e) 8 d) f(x) = x2 + 10x + 21 e) f(x) = x2 + 9

33- (PUC-MG) O ponto extremo V da funo quadrtica (x) = x2 6x + 8 : a) um mximo, sendo b) um mnimo, sendo c) um mximo, sendo d) um mnimo, sendo e) um mnimo, sendo V = (3, - 1 ) V = (- 3, +1 ) V = (- 3, + 1 ) V = (3, + 1 ) V = (3, - 1 )

34 - (U.Fortaleza) Considere a funo : R R, definida por (x) = x2 2x + 5. Pode-se afirmar corretamente que: a) o vrtice do grfico de o ponto (1, 4). b) possui dois zeros reais distintos. c) atinge um mximo para x = 1 d) o grfico de tangente ao eixo das abscissas. 35-(UC-MG) O valor mximo da funo f(x) = x 2 + 2x + 2 : a) 2 b) 3 c) 4 d) 5 e)6 289

PROF. WELLINGTON BRITO

36-(GV-SP) O custo para se produzir x unidades de um produto dado por C = 2x2 100x + 5000. O valor do custo mnimo : a) 3 250 b) 3 750 c) 4 000 d) 4 500 e) 4 950 37-(UN-Fortaleza) ABCD um quadrado de rea igual a 1(um). So tomados dois pontos P AB e Q AD tais que PA + AQ = AD. Ento o maior valor da rea do tringulo APQ : D C Q A a) 1 2 b) 1 8 c) 1 4 e) 1 16 P B

38-(GV-SP) A equao da parbola : a) y = 2x2 + 4x 6 b) y = 2( x 3 ) (x 1) c) y = 2 (x + 3 ) ( x 1) d) y = 2(x + 3 ) ( x 1) + 6 e) y = 2x2 4x + 6 v 8 6 y

x 0 e) x 1 4

4x 1 39-(CESGRANRIO) Os valores de x tais que 2 so aqueles que satisfazem: x 2x + 1 a) x > 4 b) x 4 c) x 1 4 290 d) x 1

MATEMTICA PASSO A PASSO

Respostas: 01) A 02) B 03) C 04) B 05) D 06) A 07) C 08) C 09) A 10) C 11) B 12) E 13) B 14) D 15) B 16) E 17) C 18) E 19) E 20) E 21) B 22) D 23) D 24) B 25) E 26) E 27) D 28) A 29) C 30) D 31) A 32) A 33) E 34) A 35) B 36) B 37) B 38) E 39)C

JURO SIMPLES
Introduo Se A empresta a B a importncia de R$ 100 pelo prazo de um ano, comum que, ao final desse prazo, B devolva a A a importncia de R$ 100 acrescida, digamos, de R$ 36 como uma compensao financeira denominada juro. Designando por capital a quantia emprestada, temos: R$ 100 so o capital R$ 36 so o juro O estudo que vamos iniciar agora Matemtica Financeira , com todas as suas frmulas e fatores, feito em funo do crescimento de uma certa quantia em dinheiro aplicada com o tempo, isto , dos juros. Assim, podemos dizer que: Juro a remunerao, a qualquer ttulo, atribuda ao capital. 291

PROF. WELLINGTON BRITO

Juro Simples No regime de capitalizao a juro simples, por conveno, apenas o capital inicial rende juro, isto , o juro formado no fim de cada perodo a que se refere a taxa no incorporado ao capital para, tambm, render juro no perodo seguinte; dizemos, neste caso, que os juros no so capitalizados. Juro simples aquele calculado unicamente sobre o capital inicial . Clculo do Juro Simples: Passo a Passo Por definio, o juro simples diretamente proporcional ao capital inicial e ao tempo de aplicao, sendo a taxa de juro por perodo o fator de proporcionalidade. Assim, sendo: C - o capital inicial ou principal; j - o juro simples; n - o tempo de aplicao; i - a taxa de juro unitria. j=Cx i x n que a frmula de clculo do juro simples. Nota: importante observar que essa frmula s pode ser aplicada se o prazo de aplicao n expresso na mesma unidade de tempo a que se refere a taxa i considerada.

Questes Comentadas
1) Tomou-se emprestada a importncia de R$ 1.200, pelo prazo de 2 anos, taxa de 30% ao ano. Qual ser o valor do juro a ser pago? Resoluo: Temos: C = 1.200 n=2a i = 30% a.a. = 0,3 a.a. Como: j = C x i x n Temos: j = 1.200 x 0,3 x 2 j = 720 Logo, o juro a ser pago de: R$ 720

292

MATEMTICA PASSO A PASSO

2) Aplicou-se a importncia de R$ 3.000, pelo prazo de 3 meses, taxa de 1,2% ao ms. Qual o valor do juro a receber? Resoluo: Temos: C = 3.000 n = 3 me i = 1,2% a.m. = 0,012 a.m. Como: j = 3.000 x 0,012 x 3 j = 108O juro a receber de:R$ 108 Taxas Proporcionais Duas taxas so proporcionais quando seus valores formam uma proporo com os tempos a elas referidos, reduzidos mesma unidade. Dadas duas taxas (percentuais ou unitrias) i e i, relativas, respectivamente, aos tempos n e n, referidos mesma unidade, temos: i n = 1 i n Nota: As taxas i e i devem ser ambas percentuais ou ambas unitrias. Assim, as taxas de 18% ao ano e 1,5% ao ms, por exemplo, so proporcionais, pois: 18 12 0,18 12 = ou = (1 ano = 12 meses) 1,5 1 0,015 1

Questes Comentadas:
1) Calcule a taxa mensal proporcional a 30% ao ano. Resoluo: Lembrando que 1a = 12 me, temos: Ik= 30 12 2) Calcule a taxa mensal proporcional a 0,08% ao dia. Resoluo: Lembrando que 1me = 30 d, temos: i 0,08 = i = 0,08 x 30 = 2,4 Isto : 2,4% a.m. 30 Taxas Equivalentes 293 = 2,5 Isto : 2,5% a.m

PROF. WELLINGTON BRITO

Duas taxas so equivalentes quando,aplicadas a um mesmo capital, durante o mesmo perodo, produzem o mesmo juro. Vamos calcular o juro produzido pelo capital de R$ 2.000: taxa de 4% ao ms, durante 6 meses; taxa de 12% ao trimestre, durante 2 trimestres. No primeiro caso, temos: C = 2.000 n = 6 me i = 4% a.m. = 0,04 a.m.

Logo: j = 2.000 x 0,04 x 6 j = 480 isto , o juro produzido de R$ 480,00 No segundo caso, temos: C = 2.000 n = 2 trimestres i = 12% a.t. = 0,12 a.t.

Da: j = 2.000 x 0,12 x 2 j = 480 isto ,o juro produzido de R$ 480,00 Como os juros produzidos so iguais, podemos dizer que 4% a.m. e 12% a.t. so taxas equivalentes. Nota: Assim podemos concluir que: Em regime de juro simples, duas taxas proporcionais so Equivalentes

Questes Comentadas
1) Um capital de R$ 2.400 aplicado durante 10 meses, taxa de 25% ao ano.Determine o juro obtido. Resoluo: Temos: C = 2.400 n = 10 me i = 25% a.a. = 0,25 a.a. Como o tempo dado em meses e a taxa dada ao ano, antes de aplicarmos a frmula devemos determinar a taxa mensal proporcional dada: i = 0,25 a.a = (0,25 : 12) a.m. = Logo: j = 2.400 x 0,25 0,25 12 x 10 j = 500 Isto , o juro de:R$ 500 294 a.m

MATEMTICA PASSO A PASSO

12 2) Calcule o juro correspondente a um capital de R$ 18.500, aplicado durante 2 anos , 4 meses e 10 dias, taxa de 36% ao ano. Resoluo: Como o tempo foi dado sob a forma de numeral complexo, a primeira coisa a ser feita a obteno do nmero de dias correspondentes, lembrando que: 1a = 360 d e 1 me = 30 d

Assim: 2a 4 me 10 d = ( 2 x 360 + 4 x 30 + 10) d = 850 d* Temos, ento: C = 18.500 n = 850 d i = 36% a.a. = 36 % a.d = 0,1% a.d. = 0,001 a.d. 360 Da: j = 18.500 x 0,001 x 850 j = 15.725 Isto , o juro de: R$ 15.725

EXERCCIO JURO SIMPLES ( I )


1) Calcule a taxa mensal proporcional a: a) 9% a.t. a) 1,5% a.m. b) 24% a.s. b) 8% a.t c) 21% a.s. c) 0,04% a.d d) 0,05% a.d. 2) Calcule a taxa anual proporcional a: 3) Calcule a taxa anual proporcional a 8% ao trimestre. 4) Calcule o juro a ser pago por um emprstimo de R$ 9.200, taxa de 5% ao trimestre,durante 3 trimestres. 5) Um capital de R$ 56.800 foi empregado, taxa de 0,75% ao ms, durante 2,5 meses. Calcule o juro produzido. 6) Calcule o juro resultante de uma aplicao de R$ 32.500, taxa de 18% ao ano, durante 3 meses. 295

PROF. WELLINGTON BRITO

7) Calcule o juro de um capital de R$ 5.000, em regime de juro simples, durante 2 anos e 4 meses, taxa de 24% ao ano. Respostas: 1) a) 3% a.m b) 4% a.m. c) 1,2% a.m. 2) a) 18% a.a. b) 32% a.a. c) 42% a.a. 3) 32% a.a 5) R$ 1,065 4) R$ 1.380 6) R$ 1.463 Montante J vimos que o montante (ou valor nominal) igual soma do capital inicial (ou valor atual) com o juro relativo ao perodo de aplicao, isto : montante = capital inicial + juro ou valor nominal = valor atual + juro Assim, designando o montante por M, temos: M = C+j Lembrando que: j=Cxixn a frmula pode ser escrita assim: M = C + C x i x n ou, colocando C em evidncia: M = C (1 + in)

d) 18% a.a. 7) R$ 2.800

Questes Comentadas
1) Que montante receber um aplicador que tenha investido R$ 28.000 durante 15 meses, taxa de 3% ao ms? Resoluo: Temos: C = 28.000 n = 15 me i = 3% a.m. = 0,03 a.m. Lembrando que: M = C(1 + in) vem: M = 28.000 (1 + 0,03 x 15) = 28.000 x 1,45 = 40.600, Isto : M = R$ 40.600 Nota: 296

MATEMTICA PASSO A PASSO

A soluo deste problema tambm pode ser obtida do seguinte modo: j = 28.000 x 0,03 x 15 = 12.600 Como: M = C + j Vem : M = 28.000 + 12.600 = 40.600, Isto : M = R$ 40.600 2) Qual o capital inicial necessrio para se ter um montante de 14.800 daqui a 18 meses, a uma taxa de 48% ao ano, no regime de juro simples? Resoluo: Temos: M = 14.800 n = 18 me i = 48% a.a.= (48 : 12)%a.m.= 4%a.m. 0,04a.m. Substituindo esses valores na frmula do montante obtemos: 14800= C(1+ 18 x 0,04) ou: C (1 + 18 x 0,04) = 14.800 14.800 Da: 1,72 x C = 14.800 C = C = 8.604,65 1,72 Isto : C = R$ 8.605 3) Uma concessionria vende um automvel por R$ 15.000 vista. A prazo, vende por R$ 16.540, sendo R$ 4.000 de entrada e o restante aps 4 meses. Qual a taxa de juro mensal cobrada? Resoluo: Se o cliente resolver comprar a prazo, receber financiamento para apenas R$ 11.000 (15.000 4.000) . O fato se passa, ento, como se o cliente tivesse recebido R$ 11.000 emprestados com o compromisso de devolver R$ 12.540 (16.540 4.000) aps o prazo de 4 meses. Temos, ento: C = 11.000 n = 4 me Como: M = C ( 1 + in) 297 M = 12.540

PROF. WELLINGTON BRITO

Vem:

12.540 = 11.000 ( 1 + 4i)

12.540 ou: 11.000( 1 + 4i ) = 12.540 1 + 4i = 1 + 4i = 1,14 11.000 4i = 1,14 1 i = 0,14 i = 0,035 4 Isto : i = 0,035a.m. Logo a taxa de juro cobrada de: 3,5% a.m. 4) Uma pessoa aplica R$ 4.800 a 24% ao ano. Aps algum tempo, a taxa aumentada para 3% ao ms. Determine o prazo em que vigorou a taxa de 3% ao ms, sabendo que em 8 meses os juros totalizaram R$ 912. Resoluo: Temos: C = 4.800 i1 = 3% a.m.= 0,03 a.m. n1 = n C = 4.800 i2 = 24% a.a. = 2% a.m. = 0,02 a.m. n2 = 8 n

Logo: j1 = 4.800 x 0,03 x n = 144 n j2 = 4.800 x 0,02 x ( 8 n ) = 96 ( 8 n ) Como: j1 + j2 = 912 Vem : 144n + 96 ( 8 n ) = 912 144 n + 768 96 n = 912 144 n 96 n = 912 768 48 n = 144 n = 144 n = 3 48 Isto o prazo foi de 3 meses. EXERCCIO JURO SIMPLES ( II ) 1) Qual o valor do juro correspondente a um emprstimo de R$ 3.200, pelo prazo de 18 meses, sabendo que a taxa cobrada de 3% ao ms? 2) Qual a taxa de juro cobrada em um emprstimo de R$ 1.500 a ser resgatado por R$ 2.700 no final de 2 anos? 3) A que taxa o capital de R$ 24.000 rende R$ 1.080 em 6 meses? 4) Um capital de R$ 30.000, aplicado durante 10 meses, rende juro de R$ 6.000. determine a taxa correspondente. 5) Um capital emprestado a 24% ao ano rendeu, em 1 ano, 2 meses e 15 dias, o juro de R$ 7.830. Qual foi esse capital? 298

MATEMTICA PASSO A PASSO

6) Calcule o montante de uma aplicao de R$ 5.000, taxa de 2,5% ao ms durante 2 anos. 7) Uma pessoa aplicou R$ 90.000 no mercado financeiro e, aps 5 anos, recebeu o montante de R$ 180.000. Qual foi a taxa anual? 8) Qual o tempo em que um capital de R$ 96.480, a 25% ao ano, rende R$ 79.395 de juro? 9) Sabendo que o juro de R$ 120.000 foi obtido com uma aplicao de R$ 150.000 taxa de 8% ao trimestre, calcule o prazo. 10) Um capital emprestado a 1 3 % ao ms rendeu, em 1 ano, 5 1 ms e 10 dias, o juro de R$ 19.584. Qual foi esse capital? 11) Qual o capital que, taxa de 2,5% ao ms, rende juro de R$ 126.000 em 3 anos? 12) Uma pessoa sacou R$ 21.000 de um banco sob a condio de liquidar o dbito ao fim de 3 meses e pagar ao todo R$ 22.575. A que taxa de juro obteve aquele capital? 13) Por quanto tempo um capital deve ser empregado a 40% ao ano para que o juro obtido seja igual a 4 do capital? 5 14) Em quanto tempo um capital triplica de valor taxa de 20% ao ano? 15) Sabendo que um capital foi duplicado em 8 anos a juro simples, a que taxa foi empregado esse capital? 16) mais vantajoso empregar R$ 5.260 a 24% ao ano ou R % 3.510 a 22% ao ano e o restante a 28% ao ano? 17) Determine o montante de uma aplicao de R$ 5.000, taxa de 2% ao ms, durante 2 anos. 18) Empregam-se 2 de um capital a 24% ao ano e o restante a 3 32% ao ano, obtendo-se, assim, um ganho anual de R$ 8.640. Qual o valor desse capital? 19) Qual o prazo para que uma aplicao de R$ 200.000, a 2,5% ao ms, renda um montante de R$ 240.000? 299

PROF. WELLINGTON BRITO

20) O capital de R$ 7.812 foi dividido em duas partes. A primeira, colocada a 4% ao ms, rendeu durante 5 meses o mesmo juro que a segunda durante 8 meses a 2% ao ms. Calcule o valor de cada parte. Respostas: 1) R$ 1.728 2) R$ 40% a.a 3) 0,75% a.m 4) 2% a.m 5) R$ 27.000 6) R$ 8.000 7) 20% a.a. 8) 3 anos, 3me. 15d. 9) 2 anos 6 me 10) R$ 91.800 11) R$ 140.000 12) 2,5% a.m. 13) 2 anos 14) 10 anos 15) 12,5 % a.a. 16) indiferente 17) R$ 7.400 18) R$ 32.400 19) 8 meses 20) R$ 3.472 e R$ 4.340

DESCONTO SIMPLES
Introduo Se uma pessoa deve uma quantia em dinheiro numa data futura, normal que entregue ao credor um ttulo de crdito, que o comprovante dessa dvida. Todo ttulo de crdito tem uma data de vencimento; porm, o devedor pode resgat-lo antecipadamente, obtendo com isso um abatimento denominado desconto. O desconto uma das mais comuns aplicaes da regra de juro. Alm disso: dia do vencimento o dia fixado no ttulo para o pagamento (ou recebimento) da aplicao; 300

valor nominal o valor indicado no ttulo (importncia a ser paga no dia do vencimento); valor atual o lquido pago (ou recebido) antes do vencimento; tempo ou prazo o nmero de dias compreendido entre o dia em que se negocia o ttulo e o de seu vencimento, incluindo o primeiro e no o ltimo, ou, ento, incluindo o ltimo e no o primeiro. Assim: Desconto a quantia a ser abatida do valor nominal, isto , a diferena entre o valor nominal e o valor atual. O desconto pode ser feito considerando-se como capital o valor nominal ou valor atual. No primeiro caso, denominado desconto comercial; no segundo, desconto racional. Desconto Comercial: Passo a Passo Definio Chamamos de desconto comercial, bancrio ou por fora o equivalente ao juro simples, produzido pelo valor nominal do ttulo no perodo de tempo correspondente, taxa fixada.

MATEMTICA PASSO A PASSO

Valor do desconto comercial Chamando de: d o valor do desconto comercial N o valor nominal do ttulo A o valor atual comercial ou valor descontado comercial n o tempo i a taxa de desconto Temos, pela definio: d=Nxixn que o valor do desconto comercial. Valor atual comercial 301

PROF. WELLINGTON BRITO

O valor atual comercial ou valor descontado comercial dado por: A=Nd Substituindo d pelo seu valor obtido no desconto comercial, vem: A=NNxixn Da: A =N(1ixn) que o valor atual comercial. Nota: O desconto comercial s deve ser empregado para perodos curtos, pois para prazos longos o valor do desconto pode at ultrapassar o valor nominal do ttulo.

Questo Comentada
Um ttulo de R$ 6.000 vai ser descontado taxa de 2,1% ao ms. Faltando 45 dias para o vencimento do ttulo, determine: a) o valor do desconto comercial; b) o valor atual comercial. Resoluo: Temos: N = 6.000 n = 45 d i = 2,1% a.m. = 0,021 a.m. = 0,0007 a.d. Sabemos que:d = N x i x n Logo: d = 6.000x 0,0007x 45 d = 189, Isto , o desconto comercial de: R$ 189 Como: A = N - d Vem: A = 6.000 189 A = 5.811, Isto , o valor atual comercial de: R$ 5.811 Taxa de Juro Efetiva A taxa de juro que no perodo n torna o capital A igual ao montante N a taxa que realmente est sendo cobrada na operao de desconto. Essa taxa denominada taxa de juro efetiva. Assim, simbolizando a taxa efetiva p if , temos: C (1 + if x n) = M (valor do montante) Como: C = A e M = N Temos: A (1 + if x n ) = N Da: if = 302 d

MATEMTICA PASSO A PASSO

A x n

Questo Comentada
Um ttulo de 6.000 foi descontado taxa de 2,1% ao ms, faltando 45 dias para seu vencimento. Sabendo que o desconto comercial foi de R$ 189, calcule a taxa de juro efetiva. Resoluo: Temos: N = 6.000 d = 189 n = 45 d

Como: A = N d A = 6.000 189 A = 5.811 189 189 Vem: if = = if = 0,0007227 5.811x 45 261. 495 Isto : if = 0,000723 a.d. ou if = 0,0217 a.m. ou if =2,17% a.m. Nota: Assim, para que haja igualdade entre o capital empregado e o valor atual do ttulo, necessrio que a taxa de juro seja maior que a taxa de desconto, cuja relao nos dada pela frmula: if = d Axn

EXERCCIO DESCONTO SIMPLES ( I )


1) Uma duplicata cujo valor nominal de R$ 2.000, foi resgatada 2 meses antes do vencimento, taxa de 30% ao ano. Qual o desconto comercial? 2) Um ttulo no valor nominal de R$ 8.400, com vencimento em 18/10, resgatado em 20/07. Se a taxa de juro contratada foi de 54% ao ano, qual o valor comercial descontado? 3) Um titulo de R$ 4.800 foi resgatado antes de seu vencimento por R$ 4.476. Sabendo que a taxa de desconto comercial de 32,4% ao ano, calcule o tempo de antecipao do resgate. 303

PROF. WELLINGTON BRITO

4)

Uma duplicata de R$ 23.000 foi resgatada 112 dias antes de seu vencimento por R$ 21.068. Determine a taxa de desconto e a taxa efetiva. Quero substituir um ttulo de R$ 5.000, vencvel em 3 meses, por outro com vencimento em 5 meses. Sabendo que esses ttulos podem ser descontados taxa de 3,5% ao ms, qual o valor nominal comercial do novo ttulo?

5)

6)

Um ttulo de valor nominal igual a R$ 6.300 para 90 dias dever ser substitudo por outro para 150 dias. Calcule o valor nominal do novo ttulo, taxa de 2,5% ao ms. Respostas: 1) R$ 100 2) R$ 7.266 3) 2 meses e 15 dias 5) R$ 5.424 6) R$ 6.660 4) i = 2,25% a.m. if = 2,46% a.m. Desconto Racional Definio Chamamos de desconto racional ou por dentro o equivalente ao juro produzido pelo valor atual do ttulo numa taxa fixada e durante o tempo correspondente. Valor do desconto racional Chamando de: d r o valor do desconto racional A r o valor atual ou valor descontado racional Temos, pela definio: dr = Ar x i x n O valor do desconto racional em funo do valor nominal Como: A r = N dr Da : dr= Nxixn 1+ixn que o valor do desconto racional em funo do valor nominal do ttulo. Nota: Lembrando que d = N x i x n e substituindo em d r , vem 304

MATEMTICA PASSO A PASSO

dr =

d 1 + in

, o que nos permite concluir que o desconto racional menor que o desconto comercial.

Questo Comentada
Um ttulo de R$ 6.000 vai ser descontado taxa de 2,1% ao ms. Faltando 45 dias para o vencimento do ttulo, determine: a)o valor do desconto racional; b) o valor atual racional. Resoluo: Temos: N = 6.000 n = 45 d i = 2,1% a.m. = 0,07 a.d. = 0,0007 a.d. a. Como: d r = N x i x n 1+ixn vem: d r = 6.000 x 0,0007 x 45 = 189 1+ 0,0007 x 45 1,0315

d r = 183,22 isto dr = R$ 183,00 b. Como: A r = N d, vem: A r = 6.000183, isto A r = R$ 5.817 Nota: Comparando o valor do desconto racional (R$ 183) com o valor do desconto comercial obtido (R$ 189), comprovamos a afirmao de que o desconto racional menor do que o comercial.

EXERCCIO DESCONTO SIMPLES ( II )


1) Determine o desconto* de uma promissria de R$ 3.000, taxa de 40% ao ano, resgatada 75 dias antes do vencimento 2) Uma duplicata foi descontada pelo valor de R$ 234.375 cinqenta dias antes de seu vencimento, taxa de 45% ao ano. Qual o seu valor nominal? 3) Ao pagar um ttulo de R$ 3.600 com antecipao de 90 dias, recebo um desconto de R$ 486. Qual a taxa de desconto?

305

PROF. WELLINGTON BRITO

4) O valor atual de um ttulo de R$ 4.800 de R$ 4.380. Sabendo que a taxa bancria de desconto de 3,5% ao ms, qual o tempo de antecipao? 5) Uma duplicata de R$ 69.000 foi resgatada antes do seu vencimento por R$ 58.909.Sabendo que a taxa de desconto foi de 3 1 % ao ms, qual o tempo de antecipao? 4 6) Uma empresa possui um titulo cujo valor nominal de R$ 7.000, com vencimento daqui a 150 dias. Quantos dias antes do vencimento deve descont-lo, taxa comercial de 36% ao ano, para que possa adquirir mercadorias no valor de R$ 6.790? 7) Um comerciante vai a um banco e desconta uma nota promissria para 90 dias, taxa de 3% ao ms, mais 1,5% de comisso. Sabendo que o liquido creditado para o comerciante foi de R$ 17.900, qual o valor da promissria? 8) Um ttulo de R$ 27.000 foi descontado faltando 60 dias para o seu vencimento. Sabendo que o desconto foi de R$ 1.800, calcule a taxa de desconto e a taxa de juro efetiva ao ano. 9) Determine o valor do desconto e o valor atual racionais de um ttulo de R$ 50.000, disponvel dentro de 40 dias, taxa de 3% ao ms. 10) Calcule o valor nominal de um ttulo com vencimento para 60 dias, sabendo que a diferena entre os seus descontos comercial e racional, taxa de 3% ao ms, de R$ 408. * Neste texto,sempre que o desconto no for explicitado, voc deve subentender desconto comercial Respostas 1) R$ 250 5) 135 dias 9) dr = 1923 Ar = 48.077 2) R$ 250.000 6) 30 dias 10) R$ 120.133 3) 4,5% a.m 7) R$ 20.000 4) 75 dias 8) 40% a.a. e 42,86% a.a.

JURO COMPOSTO Definio


O regime de capitalizao que vamos estudar o mais comumente usado. Nele, o juro, a partir do segundo perodo, calculado sobre o montante do perodo anterior. Da afirmamos que neste regime o juro rede juros. 306

MATEMTICA PASSO A PASSO

Juro Composto aquele que em cada perodo financeiro, a partir do segundo, calculado sobre o montante relativo ao perodo anterior. Clculo Do Montante: Passo a Passo Consideremos, agora, um capital C, aplicado em regime de juro composto taxa i. Temos: Perodo 1 2 3 Juro j1 = C x i j2 = M1 x i j 3 = M2 x i Montante M1 = C + j1 = C + Ci M1 = C(1 + i ) M2 = M1+ j2 = M1 + M1 x i = M1 (1+i ) = C(1+i ) (1+ i) M2 = C(1+i)2 M3 = M2 + j3 = M2 + M2 x i = M2 (1+i ) = C(1+i ) (1+ i )2 M3 = C(1+ i)3 o que nos permite escrever, para o ensimo perodo: Mn = C( 1 + i )n Esta a frmula do montante em regime de juro composto, tambm chamada frmula fundamental do juro composto, para um nmero inteiro de perodos. O fator ( 1 + i )n denominado fator de capitalizao ou fator de acumulao de capital. Nota: Ainda aqui, como em juro simples, a unidade para a resoluo de um problema determinada pelo perodo financeiro a que se refere.

Questo comentada
Calcule o montante produzido por R$ 2.000, aplicados em regime de juro composto a 5% ao ms, durante 2 meses. Resoluo: Temos: C = 2.000 n = 2 me i = 5% a.m. = 0,05 a.m 307

PROF. WELLINGTON BRITO

Substituindo esses valores em Mn vem: M2 = 2.000 ( 1 + 0,05 )2 Logo: M2 = 2.000 x 1,052 = 2.000 x 1,1025 M2 = 2.205, Isto , o montante de: R$ 2.205 Tbua Financeira: Passo a Passo Para localizarmos nessa Tbua determinado valor de (1+ i ) n procuramos o quadro da taxa percentual correspondente a i (que, por questo didtica, designaremos por tabela) e na primeira coluna dessa tabela o valor de n. O valor de ( 1 + i )n aquele que figura na interseco da segunda coluna com a linha do nmero de perodos (n). Nessa Tbua, o nmero de perodos dado na unidade de tempo da taxa; assim, se a taxa anual, n o nmero de anos; se mensal, n o nmero de meses etc. Exemplos: 1) Temos: i = 20% a.a. = 0,2 a.a. n=5a Queremos determinar o valor de (1 + 0,2) 5 Localizamos, inicialmente, a tabela correspondente a i = 20%. Na primeira coluna procuramos o valor 5 de n. O valor de (1 + 0,2) 5 aquele que se encontra na interseco da quinta linha com a segunda coluna: 2,48832. Logo: ( 1 + 0,2)5 = 2,48832 2) Temos: i = 3% a.m. = 0,03 a.m. n = 1 a 4 me = 16 me

Logo, pela tabela correspondente a 3%, vem: ( 1 + 0,03)16 = 1, 60471 Questes Comentadas 1) Uma pessoa toma R$ 3.000 emprestados, a juro de 3% ao ms, pelo prazo de 10 meses,com capitalizao composta. Qual o montante a ser devolvido? Resoluo: Temos: C = 3.000 n = 10 me i = 3% a.m. = 0,03 a.m. Como: Mn = C( 1 + i )n vem: M10 = 3.000( 1 + 0,03 )10 308

MATEMTICA PASSO A PASSO

Consultando a Tbua Financeira, obtemos: (1 + 0,03)10 = 1,34392 Logo: M10 = 3.000 x 1,34392 M10 = 4.031,76, Isto , a quantia a ser devolvida de: R$ 4.032 2) Determine em que prazo um emprstimo de R$ 11.000 pode ser quitado em um nico pagamento de R$ 22.125, sabendo que a taxa contratada de 15% ao semestre em regime de juro composto. Resoluo: Temos: Mn = 22.125 C = 11.000 i = 15% a.s. = 0,15 a.s. Substituindo esses valores na frmula fundamental, vem: 22. 125 = 11.000 (1 + 0,15) n ou: 1,15n = 22.125 11.000 1,15 n = 2,01136

Tbua Financeira: Pesquisando na tabela correspondente taxa de 15%, na segunda coluna, verificamos que para n = 5 temos( 1 + i) n = 2,01136, Logo: n=5 isto , o prazo de: 5 semestres ou 2 anos e 6 meses

EXERCCIO JURO COMPOSTO 1) Calcule o montante de uma aplicao de R$ 8.000, taxa de 3% ao ms, pelo prazo de 14 meses. 2) Determine o juro de uma aplicao de R$ 20.000, a 5% ao ms, capitalizado mensalmente durante 8 meses. 3) Qual o montante produzido pelo capital de R$ 6.800, em regime de juro composto, aplicado durante 4 meses, taxa de 3,8% ao ms? 309

PROF. WELLINGTON BRITO

4) Em que prazo uma aplicao de R$ 100.000, em regime de juro composto, produzir um montante de R$ 146.853,00 taxa de 3% ao ms, capitalizado mensalmente? Respostas: 1) R$ 12.101,00 2)R$ 9.549,00 3)R$ 7.894,00 4)13 meses

QUESTES DE CONCURSOS E VESTIBULARES 01) (BNB) Em juros simples, a taxa de juros anual equivalente a 6% ao bimestre : a) 18% ao ano b) 72% ao ano c) 36% ao ano d) 12% ao ano e) 6% ao ano

02) (BNB) O valor futuro de um ttulo, contratado a juros simples, igual ao dobro do seu valor inicial. Sabe-se que a taxa de juros da operao foi de 12,5% ao ano. Qual o prazo de aplicao? a) 4 meses c) 1 ano e) 8 anos 310

MATEMTICA PASSO A PASSO

b) 4 anos

d) 8 meses

03) (TJ) Aplicado R$ 1.500,00 por um ano, obtive juros de R$ 405,00. No regime de juros simples, por quanto tempo eu devo aplicar o mesmo valor para obter uma renda de R$ 135,00? a) 4 meses e 15 dias b) 4 meses c) 2 meses e 20 dias d) 2 meses e 10 dias e) 3 meses e 15 dias

04) (TJ) Uma pessoa aplicou, a juros simples, 3/5 do seu capital a 7% ao ms e o restante a 66% ao ano. Passados 2 anos e 8 meses, recebeu um total de R$ 12.697,60 de juros. O capital aplicado por essa pessoa foi de: a) R$ 6.200,00 b) R$ 5.079,04 c) R$ 5.618,56 d) R$ 5.400,00 e) R$ 6.079,04

05) (SEFAZ) Um ttulo de valor nominal de R$ 9.500,00 sofreu um desconto bancrio taxa de 60% ao ano, 90 dias antes do seu vencimento. Sabendo-se que as taxas e comisses cobradas pelo banco importaram em 2,5% do valor nominal do ttulo, pode-se afirmar que o desconto bancrio foi de: a) R$ 1.453,20 c) R$ 1.662,50 e) R$ 1.834,60 b) R$ 1.574,00 d) R$ 1.728,30 06) (TTN) Jos descontou 2 duplicatas em um banco, no regime de juros simples comerciais, a uma taxa de juros anuais de 15%. O primeiro ttulo vencia em 270 dias e o segundo em 160 dias, sendo que o ltimo era de valor nominal 50% superior ao primeiro. Sabendo-se que os dois descontos somara o valor de R$ 382,50 o titulo que produziu o maior desconto, tinha valor nominal em reais, de: a) R$ 1.800,00 c) R$ 1.900,00 e) R$ 1.750,00 b) R$ 1.700,00 d) R$ 1.850,00 07) (BB) Jos vai receber os R$ 10.000,00 da venda de seu carro em duas parcelas de R$ 5.000,00, sendo a primeira dentro de 30 dias e a segunda, dentro de 60 dias. Considerando uma taxa de desconto de 2% ao ms, o valor atual, que Jos deveria receber hoje, com a certeza de estar recebendo o mesmo valor que ir receber no pagamento, de: a) R$ 9.709,65 c) R$ 9.729,65 e) R$ 9.749,65 b) R$ 9.719,65 d) R$ 9.739,65 08) (CEF) Suponha que R$ 20.000,00 sejam depositados numa caderneta de poupana que no possui correo monetria e 311

PROF. WELLINGTON BRITO

que rende 8% de juros ao semestre, compostos semestralmente. Se nenhuma retirada e nenhum deposito adicional foram feitos, ento o valor total dos juros creditados no final de 2 anos de: a) R$ 7.205,77 c) R$ 7.209,80 e) R$ 7.206,80 b) R$ 7.204,80 d) R$ 7.202,70 09) (CEF) Pretendendo guardar uma certa quantia para as festas de fim de ano, Jos depositou R$ 2.000,00 em 05/06/2003 e R$ 3.000,00 em 05/09/03. Se o banco pagou juros composto taxa de 10% ao trimestre, em 05/12/03 Jos tinha um total de: a) R$ 5.320,00 c) R$ 5.620,00 e) R$ 5.720,00 b) R$ 5.480,00 d) R$ 5.680,00 10) (CSD-SP) Para a srie de valores: 0, -1, -2, 5, 4, -3, -7, 2, -4, e 6: a)A mdia aritmtica 3,4 e a varincia 16; b)A mdia aritmtica zero e a varincia 4; c)A mdia aritmtica zero e a varincia 16; d)A mdia aritmtica 3,4 e a varincia 4; e)A mdia aritmtica zero mas a varincia impossvel calcular; 11) (T.C.U.) Os preos do pacote de caf (500g) obtidos em diferentes supermercados locais so: R$ 3,50, R$ 2,00, R$ 1,50 e R$ 1,00.Dados essas informaes, julgue os itens que se seguem: (1) O preo mdio do pacote de 500g de caf de R$ 2,00. (2) Se todos os preos tiverem uma reduo de 50%, o novo preo mdio ser de R$ 1,50; (3) A varincia dos preos igual a 0,625; (4) Se todos os preos tiverem um aumento de % R$ 1,00, o coeficiente de variao dos preos no se altera; (5) Se todos os preos tiverem um aumento de 50%, a nova varincia ser exatamente igual anterior, pois a disperso no ser afetada. 12) (T.T.N) Assinale a opo correta: a) A moda, a mediana e a mdia aritmtica so expressas na mesma unidade de medida da varivel a que se referem; b) A moda uma medida de posio que permite dividir a distribuio em duas partes de igual freqncia; c) A mdia harmnica a mdia geomtrica dos inversos das determinaes da varivel; d) A mdia aritmtica no influenciada pelos valores extremos da distribuio; 312

MATEMTICA PASSO A PASSO

e) A moda e a mediana so influenciadas pelos valores extremos da distribuio. Considere a distribuio de freqncia transcrita a seguir para responder s questes ; 13, 14, 15 e 16 Dimetro (cm) 4 6 6 8 8 10 10 12 12 14 13) (T.T.N) a) Mais de 85% as observaes tm dimetro no inferior a 6cm; b) 75% das observaes esto no intervalo 6 12; c) A soma dos pontos mdios dos intervalos de classe inferior soma das freqncias absolutas simples; d) e) 28% das observaes esto no quarto intervalo de classe; Menos de 25% das observaes tm dimetro abaixo de 10 cm. Freqncias simples absoluta 6 8 12 10 4

14) ( T.T.N) A mdia aritmtica da distribuio igual a: a) 8,90 cm b) 9,15 cm c) 9,00 cm d) 8,80 cm e) 8,70cm e) 9,6 cm 15) (T.T.N) A moda da distribuio igual a: a) 9,4 cm b) 9,5 cm c) 9,7 cm d) 9,3 cm

16) (T.T.N) A mediana da distribuio: a) Pertence a um intervalo de classe distinto do que contm a mdia aritmtica; b) eqidistante da mdia aritmtica e da moda; c) igual mdia aritmtica; d) inferior mdia aritmtica; e) Coincide com o ponto mdio de um intervalo de classe. Considere a distribuio de freqncia transcrita a seguir para responder s questes: 17, 18, 19 e 20. 313

PROF. WELLINGTON BRITO

Peso (kg) 2 4 4 6 6 8 10 12 8 10

Freqncias simples absoluta 9 12 6 2 1

17) (T.T.N) a) Menos de 20 das observaes tm peso igual ou superior a 4kg: b) A soma dos pontos mdios dos intervalos de classe inferior ao tamanho da amostra; c) 8% das observaes tm peso no intervalo de classe 8 10; d) 65% das observaes tm peso no inferior a 4kg e inferior a 10 kg; e) Mais de 65% das observaes tm peso maior ou igual a 4 kg. 18) (T.T.N) A mdia aritmtica da distribuio igual a: a) 5,21kg b) 5,19kg c) 5,30kg d) 5,27kg e) 5,24 kg

19) (T.T.N) A mediana da distribuio igual a: a) Menor que 5kg b) 5,10kg c) 5,20kg d) 5,30kg e) 5,00kg 20) (T.T.N) A moda da distribuio: a) maior do que a mediana e do que a mdia geomtrica; b) um valor inferior mdia aritmtica e mediana; c) Pertence a um intervalo de classe distinto do da mdia aritmtica; d Coincide com o limite superior de um intervalo de classe. e) Coincide com o ponto mdio de um intervalo de classe. Respostas: 1) C 2) E 3) B 4) A 12) A 13) B 14) A 15) D 314

MATEMTICA PASSO A PASSO

5) C 6) A 7) A 8) C 9) E 10) C 11) (1) certo (2) (3) (4) (5) errado

16) E 17) E 18) B 19) E 20) B

315

S-ar putea să vă placă și